Scrigroup - Documente si articole

     

HomeDocumenteUploadResurseAlte limbi doc
BulgaraCeha slovacaCroataEnglezaEstonaFinlandezaFranceza
GermanaItalianaLetonaLituanianaMaghiaraOlandezaPoloneza
SarbaSlovenaSpaniolaSuedezaTurcaUcraineana

AdministrationAnimalsArtBiologyBooksBotanicsBusinessCars
ChemistryComputersComunicationsConstructionEcologyEconomyEducationElectronics
EngineeringEntertainmentFinancialFishingGamesGeographyGrammarHealth
HistoryHuman-resourcesLegislationLiteratureManagementsManualsMarketingMathematic
MedicinesMovieMusicNutritionPersonalitiesPhysicPoliticalPsychology
RecipesSociologySoftwareSportsTechnicalTourismVarious

COST MANAGEMENT STUDY QUESTIONS

managements



+ Font mai mare | - Font mai mic













COST MANAGEMENT



STUDY QUESTIONS




CONTENTS


I. INTRODUCTION 1


II. MANUFACTURING ENVIRONMENT 3


III. COST BEHAVIOR 11


IV. COST VOLUME PROFIT 18


V. VARIABLE COSTING 28


VI. SEGMENT ANALYSIS 34


VII. DECISION MAKING 37


VIII. JOB ORDER COSTING 48


IX. PROCESS COSTING 58


X. BUDGETING 70


XI. FLEXIBLE BUDGETING 79


XII. STANDARD COSTS 85


XIII. JOINT COSTING 102


XV. SERVICE DEPARTMENT COST ALLOCATION 108


XV ANSWERS. 117



I. INTRODUCTION



1. The organization chart: a. depicts only line functions; b. is applicable only to profit‑oriented companies; c. shows that staff functions have authority over line functions; d. is used only in centralized organizations; e. none of these.


2. Accounting information: a. generally constitutes all the information received by a manager in directing the affairs of an organization; b. is used by few persons in an organization other than financial officers; c. is essential to all of the basic functions of management; d. Responses a, b, and c are all correct.


3. Financial and managerial accounting are similar in that: a. both record the financial history of an organization; b. both rely on the concept of responsibility or stewardship; c. both have a strong future orientation; d. both rely on generally accepted accounting principles; e. none of these.


4. Managerial accounting: a. has its primary emphasis on the future; b. is required by regulatory bodies such as the SEC; c. focuses on the organization as a whole, rather than on the organization's segments; d. Responses a, b, and c are all correct; e. Only responses a and c are correct.


5. Decentralization: a. is possible in both profit and service oriented organizations; b. is present in any company that has a well prepared organization chart; c. means the delegation of decision making authority to lower levels of management; d. Responses a and c are both correct; e. Responses a, b, and c are all correct.


6. The function of management that compares planned results against actual results is known as: a. planning; b. organizing and directing; c. controlling; d. decision making; e. none of these.


7. Which of the functions of management is an inseparable part of the other functions? a. Planning; b. Organizing and directing; c. Controlling; d. Decision making; e. none of these.


8. Which of the following persons would be in a line position at State University?

I. General manager of the campus bookstore

II. Dean of the law school

III. Head of the university's library

IV. Professor of Accounting


a. Only II; b. Only II and IV; c. Only II, III, and IV; d. I, II, III, and IV; e. Only I and III.


9. Obtaining feedback is generally identified most directly with which of the functions of management? a. Planning; b. Organizing and directing; c. Controlling; d. Decision making.


Which of the following persons would occupy a staff position at Valley View Hospital?

I. Chief physician, emergency room

II. Head, laboratory and testing

III. General manager, housekeeping and laundry

IV. General manager, outpatient clinic


a. Only I; b. Only I and II; c. Only II and III; d. Only III and IV; e. I, II, III, and IV.


Which of the following is not a characteristic of a controller? a. Occupies a line position; b. Is a member of top management; c. Exercises control outside the accounting department through the reporting and interpreting of data; d. Advises top management in special problem situations; e. All of the above are characteristics of a controller.


Which of the following statements are true regarding financial and managerial accounting?

I. Both are mandatory.

II. Both rely on the same accounting information system.

III. Both emphasize the segments of an organization, rather

than just looking at the organization as a whole.

IV. Both are geared to the future, rather than to the past.


a. I, II, III, and IV; b. Only II, III and IV; c. Only II and III; d. Only II; e. Only III.


Which of the functions of management involves the overseeing of day‑to‑day activities? a. Planning; b. Organizing and directing; c. Controlling; d. Decision making; e. none of these.


Which of the following statements are false concerning line and staff functions?

I. Persons occupying staff functions have authority

over persons occupying line functions.

II. Both line and staff functions are depicted on the

organization chart.

III. Line functions are directly related to the basic

objectives of an organization.


a. Only I; b. Only II; c. Only I and II; d. I, II, and III; e. All of the above statements are true.


Which of the following persons would occupy a line position in a department store?

I. Sales manager

II. Manager, furniture department

III. Manager, advertising department

IV. Manager, personnel department


a. Only I; b. Only I and II; c. Only I, II, III; d. I, II, III, IV; e. Only III and IV.


Which of the following persons would occupy a line position in a department store?

I. Sales manager

II. Manager, furniture department

III. Manager, advertising department

IV. Manager, personnel department


a. Only I; b. Only I and II; c. Only I, II, III; d. I, II, III, IV; e. Only III and IV.



II. MANUFACTURING ENVIRONMENT



1. Given the following information for the Duxbury Manufacturing Company:


Work in process‑ending $ 8,000

Work in process‑beginning 11,000

Cost of goods manufactured 70,000

Direct labor 25,000

Direct materials used 20,000


The manufacturing overhead is: a. $22,000; b. $25,000; c. $28,000; d. $36,000; e. none of these.


2. The annual premium payment for the factory manager's life insurance would be a: a. fixed, period, indirect cost; b. fixed, product, direct cost; c. variable, product, direct cost; d. fixed, product, indirect cost; e. none of these.


3. Each of the following would be classified as variable in terms of cost behavior except: a. prime costs; b. sales commissions; c. plant manager's salary; d. direct labor; e. none of these.


4. During the month of May, 19x8, Bennett Manufacturing Company purchased $43,000 of raw materials. The manufacturing overhead totaled $27,000 and the total manufacturing costs were $106,000. Assuming a beginning inventory of raw materials of $8,000 and an ending inventory of raw materials of $6,000, direct labor must have totaled: a. $34,000; b. $38,000; c. $36,000; d. $45,000; e. none of these.


5. During 19x8 there was no change in either the raw materials or the work in process beginning and ending inventories. However, finished goods, which had a beginning balance of $25,000, increased by $15,000. If the manufacturing costs incurred totaled $600,000 during 19x8, the goods available for sale must have been: a. $585,000; b. $600,000; c. $610,000; d. $625,000; e. none of these.


The following data relate to questions 6, 7 and 8.


Jun. 1, 19x7 Jun. 30, 19x7

Finished Goods $25,000 $ 17,000

Work in Process 0 10,000

Raw Materials 15,000 15,000

Other Data:

Indirect labor $ 4,000

Direct labor 30,000

Administrative expenses 10,000

Purchase of raw material 35,000

Sales salaries 12,000

Factory rent 5,000

Miscellaneous factory overhead 11,000

Other selling expenses 4,000

Sales 125,000


6. The raw materials used in production would be: a. $50,000; b. $35,000; c. $15,000; d. $ 0; e. none of these.


7. The cost of goods manufactured would be: a. $95,000; b. $85,000; c. $75,000; d. $93,000; e. none of these.


8. The gross margin would be: a. $50,000; b. $42,000; c. $32,000; d. $16,000; e. none of these.


9. During the first week of April, Gillian worked a total of 50 hours. Gillian is paid $15 per hour for regular time, and is paid time and a half for all hours in excess of a 40 hour week. The amount of Gillian's wages that should be charged to direct labor for the week is: a. $600; b. $675; c. $750; d. $825; e. none of these.


During the month of May, 19x7, Cassidy Manufacturers incurred $30,000, $40,000 and $20,000 of direct material, direct labor and factory overhead costs respectively. If the cost of goods manufactured was $95,000 in total and the ending work in process inventory was $15,000, the beginning inventory of work in process must have been: a. $10,000; b. $20,000; c. $110,000; d. $25,000; e. none of these.


Higgins Company's manufacturing overhead is 40% of its total conversion costs. If direct labor is $18,000 and if direct materials are $24,000, the manufacturing overhead will be: a. $30,000; b. $27,000; c. $16,000; d. $12,000; e. none of these.


The Lyons Company's cost of goods manufactured was $120,000 when its sales were $360,000 and its gross margin was $220,000. If the ending inventory of finished goods was $30,000, the beginning inventory of finished goods must have been: a. $10,000; b. $50,000; c. $130,000; d. $150,000; e. none of these.


This month, when 10,000 units of a product were manufactured, the cost per unit was $60. At this level of activity, variable costs are 50 percent of total unit costs. If 15,000 units are manufactured next month and cost behavior patterns remain unchanged the: a. total variable cost will remain unchanged; b. fixed costs will increase in total; c. variable cost per unit will increase; d. total cost per unit will decrease; e. none of these.


Indirect costs are also known as: a. differential costs; b. common costs; c. opportunity costs; d. sunk costs; e. none of these.


The following data relate to questions 15, 16 and 17.


May 1, 19x8 May 31, 19x8

Work in process inventory $ 7,000 $ 12,000

Raw materials inventory 15,000 ?

Finished goods inventory ? 20,000

Other data:

Raw material used 40,000

Sales 200,000

Cost of goods manufactured 135,000

Manufacturing overhead 60,000

Raw materials available 45,000

Administrative salaries 15,000

Gross Margin 60,000


The ending inventory of raw materials would be: a. $5,000; b. $10,000; c. $15,000; d. $20,000; e. none of these.


The beginning inventory of finished goods would be: a. $5,000; b. $15,000; c. $25,000; d. $30,000; e. none of these.


17. The direct labor for May, 19x8 would be: a. $35,000; b. $40,000; c. $30,000; d. none of these; e. cannot be determined from the data available.


Using the following data:


Cost of goods sold $70

Direct labor 20

Direct materials placed into production 15

Cost of goods manufactured 80

Work in process ending 10

Finished goods ending 15

Manufacturing overhead 30

18. The beginning work in process inventory is: a. $0; b. $10; c. $20; d. $30; e. none of these.


19. The gross margin for Cushing Company for the first quarter of 19x8 was $325,000 when sales were $700,000. The beginning inventory of finished goods was $60,000 and the ending inventory of finished goods was $85,000. The cost of goods manufactured would be: a. $300,000; b. $350,000; c. $400,000; d. $230,000; e. none of these.


20. Variable cost: a. increases on a per unit basis as the number of units produced increases; b. is constant if expressed on a per unit basis; c. remains the same in total as production increases; d. is not affected by changes in activity from period to period; e. none of these.


21. The property taxes on the factory building for a small manufacturer would be an example of:


Prime Cost Conversion Cost

a. No Yes

b. Yes No

c. Yes Yes

d. No No


22. During the month of July, McElroy Company's direct labor cost totaled $36,000, and direct labor cost was 60% of prime cost. If total manufacturing costs during July were $85,000, the manufacturing overhead was: a. $24,000; b. $25,000; c. $49,000; d. $60,000; e. none of these.


23. All of the following are examples of product costs except: a. depreciation on the company's retail outlets; b. salary of the plant manager; c. insurance on the factory equipment; d. rental costs of the factory facility; e. All of the above are examples of product costs.


24. During the last week in October, Harvey worked a total of 45 hours. Harvey is paid $10 per hour for regular time, and is paid time and a half for all hours in excess of 35 hours per week. Given this information: a. $350 should be charged to direct labor; b. $50 should be charged to manufacturing overhead; c. $150 should be charged to manufacturing overhead; d. $500 should be charged to direct labor; e. none of these.


The following data relate to questions 25, through 27.


July 1, 19x8 July 31, 19x8

Raw materials inventory 0 $ 5,000

Work in process inventory ? 4,000

Finished goods inventory $12,000 ?

Other data:

Cost of goods manufactured $105,000

Administrative expense 15,000

Raw materials used 40,000

Manufacturing overhead 20,000

Direct labor 39,000

Gross profit 100,000

Sales 210,000


25. The beginning inventory of work in process would be: a. $9,000; b. $14,000; c. $1,000; d. $4,000; e. none of these.


26. The ending inventory of finished goods would be: a. $17,000; b. $12,000; c. $7,000; d. $2,000; e. none of these.


27. The direct labor as a percentage of conversion costs would be: a. 25%; b. 50%; c. 66 2/3%; d.100%; e. none of these.


28. As the number of units produced in a manufacturing setting increases: a. the variable cost per unit remains the same; b. fixed costs in total remain the same; c. variable costs increase in total; d. all of the above; e. none of the above.


29. In the manufacture of a product, the direct labor costs combined with the direct materials costs are known as: a. period costs; b. conversion costs; c. prime costs; d. opportunity cost; e. none of these.


30. The cost associated with idle time should be: a. included as part of direct labor cost; b. treated as part of manufacturing overhead; c. added directly to cost of goods sold for the period; d. included as part of selling and administrative expenses; e. none of these.


31. The distinction between indirect and direct costs depends on: a. whether a cost is controllable or noncontrollable; b. whether a cost is variable or fixed; c. whether a cost is a product or a period cost; d. whether a cost can be conveniently and physically traced to a unit under consideration; e. none of these.


32. During the month of June 19x8, Reardon Company incurred $17,000 of direct labor, $8,500 of manufacturing overhead and purchased $15,000 of raw materials. Between the beginning and the end of the month, the raw materials inventory increased by $2,000, the finished goods inventory increased by $1,500, and the work in process inventory decreased by $3,000. The cost of goods manufactured would be: a. $38,500; b. $39,500; c. $41,500; d. $42,500; e. none of these.


33. During the month of April, 19x8, LTP Company incurred $30,000 of manufacturing overhead, $40,000 of direct labor, and purchased $25,000 of raw materials. Between the beginning and the end of the month, the raw materials and work in process inventories decreased by $4,000 and $3,000, respectively. The total manufacturing costs added to production during the month of April, 19x8 would be: a. $88,000; b. $91,000; c. $99,000; d. $102,000; e. none of these.


34. Which of the following should not be included as manufacturing overhead in the manufacture of a wooden chair? a. Glue in the chair; b. The amount paid to the individual who stains the chair; c. The workman's compensation insurance of the supervisor who oversees production; d. The factory utilities of the department in which production takes place; e. All of the above are manufacturing overhead items.


35. During 19x8, there was no change in the beginning or ending balance in the Raw Materials inventory account for the Devlin Company. However, the Work in Process inventory account increased by $15,000, and the Finished Goods inventory account decreased by $10,000. If purchases of raw materials were $100,000 for the year, direct labor cost was $150,000, and manufacturing overhead cost was $200,000, the cost of goods available for sale for the year would be: a. $435,000; b. $445,000; c. $465,000; d. $475,000; e. none of these.


36. In the preparation of the schedule of Cost of Goods Manufactured, the accountant included as part of manufacturing overhead the rental expense on the firm's retail facilities. This inclusion would: a. overstate the beginning work in process inventory; b. overstate the goods available for sale on the income statement; c. understate the cost of goods manufactured; d. have no effect on the cost of goods manufactured amount; e. none of these.


The following selected data for March, 19x8, taken from Rubenstein's financial statements, relate to questions 37 through 40.


Cost of goods available for sale $ 65,000

Manufacturing overhead 20,000

Cost of goods manufactured 51,000

Finished goods inventory ‑ ending 10,000

Direct materials used 15,000

Sales 105,000

Operating expenses 30,000

Direct labor 20,000

Work in process inventory ‑ beginning 0


37. The March 31, 19x8 gross margin would be: a. $55,000; b. $54,000; c. $50,000; d. $40,000; e. none of these.


38. The March 1, 19x8 finished goods inventory would be: a. $24,000; b. $9,000; c. $10,000; d. $14,000; e. none of these.


39. The March 31, 19x8 work in process inventory would be: a. $4,000; b. $8,000; c. $10,000; d. 0; e. none of these.


40. Assuming that 10,000 units were completed during March, and the Manufacturing Overhead cost is all fixed, the average unit cost would be: a. $5.50; b. $5.10; c. $3.10; d. none of these; e. cannot be determined from the data given.


41. Depreciation on a company's factory building would be classified as: a. period cost; b. opportunity cost; c. prime cost; d. product cost; e. none of these.


42. The cost of rent for a manufacturing plant is a:


Prime cost Product cost

a. No Yes

b. No No

c. Yes No

d. Yes Yes


43. Property taxes on a manufacturing plant are an element of:

Conversion cost Period cost

a. Yes No

b. Yes Yes

c. No Yes

d. No No


44. Factory supplies for a manufacturing plant are generally: a. prime costs; b. period costs; c. variable costs; d. excluded from product costs; e. none of these.


45. Prime cost and conversion cost share what common element of total cost? a. Direct labor; b. Direct materials; c. Variable overhead; d. Fixed overhead; e. none of these.


46. Indirect labor is a part of: a. nonmanufacturing cost; b. conversion cost; c. prime cost; d. period cost; e. none of these.


The following data relate to questions 47 through 49.


Inventories 4/1/x7 4/30/x7

Direct materials $18,000 $15,000

Work‑in process 9,000 6,000

Finished goods 27,000 36,000


Additional information for the month of April 19x7:


Direct materials purchased $42,000

Direct labor payroll 30,000

Direct labor rate per hour $ 7.50

Factory overhead rate per direct labor hour 10.00


47. For the month of April 19x7, prime cost incurred was: a. $75,000; b. $69,000; c. $45,000; d. $39,000; e. none of these.


48. For the month of April 19x7, conversion cost incurred was: a. $30,000; b. $40,000; c. $70,000; d. $72,000; e. none of these.


49. For the month of April 19x7, cost of goods manufactured was: a. $118,000; b. $115,000; c. $112,000; d. $109,000; e. none of these.


50. Which of the following is an element of prime cost?


Direct materials Indirect materials

a. Yes No

b. Yes Yes

c. No Yes

d. No No


51. Wages paid to a timekeeper in a factory are a


Prime cost Conversion cost

a. No No

b. No Yes

c. Yes No

d. Yes Yes


52. The fixed portion of the cost of electricity for a manufacturing plant is a:


Conversion Cost Product cost

a. No No

b. No Yes

c. Yes Yes

d. Yes No


53. The cost of fire insurance for a manufacturing plant is generally a: a. nonmanufacturing cost; b. period cost; c. variable cost; d. conversion cost; e. none of these.


54. Factory overhead includes: a. all manufacturing costs; b. all manufacturing costs, except direct materials and direct labor; c. indirect materials but not indirect labor; d. indirect labor but not indirect materials; e. none of these.


The following data relate to questions 55 and 56.


Morton Company's manufacturing costs for 19x4 were as follows:


Direct materials $300,000

Direct labor 400,000

Factory overhead:

Variable 80,000

Fixed 50,000


55. Prime cost totaled: a. $300,000; b. $380,000; c. $700,000; d. $830,000; e. none of these.


56. Conversion cost totaled: a. $400,000; b. $480,000; c. $530,000; d. $830,000; e. none of these.



III. COST BEHAVIOR



1. An example of a committed fixed cost for a manufacturing firm is: a. a development program for the foreman; b. factory clerical salaries with no annual salary or time guarantee; c. property taxes on the factory building; d. new product research and development; e. none of these.


2. In describing the equation y = a + bx, which of the following statements is correct? a. 'x' is the dependent variable; b. 'a' is the fixed component; c. In the high ‑low method, 'b' equals change in activity divided by change in costs; d. As 'x' increases 'y' decreases; e. none of these.


3. Which of the following statements is true when referring to fixed costs? a. Committed fixed costs arise from the annual decisions by management; b. As volume increases, unit fixed cost and total fixed cost will change; c. Fixed costs increase in total throughout the relevant range; d. Discretionary fixed costs can often be reduced to zero for short periods of time without seriously impairing the long‑run goals of the firm; e. none of these,


The following data relate to questions 4 and 5.


Cosco, Inc. has accumulated the following data for the cost of maintenance on its machinery for the last four months of 19x8:


Month Maintenance Cost Machine Hours

September $26,020 21,000

October $24,600 18,500

November $22,300 15,000

December $25,100 19,000


4. Assuming Cosco Company uses the high‑low method of analysis, the fixed cost of maintenance would be: a. $14,500; b. $5,020; c. $13,000; d. $12,320; e. none of these.


5. If machine hours are budgeted to be 20,000 hours during January, 19x9, the budgeted total maintenance cost would be: a. $25,400; b. $25,560; c. $23,700; d. $24,720; e. none of these.


6. Given the cost formula y = $12,500 + $5.00x, total cost for an activity level of 4,000 units would be: a. $20,000; b. $7,500; c. $16,000; d. $32,500; e. none of these.


7. An analysis of clerical costs in the billing department of Craig Company indicates that total unit (variable and fixed) processing costs will be $.50 per account processed at an activity level of 32,000 accounts. When only 22,000 accounts are processed, the total cost of processing is $12,500. Given these data, at a budgeted level of 25,000 accounts: a. processing costs will total $8,750; b. fixed processing costs will be $10,400; c. the variable processing costs will equal $.35 per account processed; d. processing costs will total $14,975; e. none of these.


8. Glory Company's gross margin exceeded its contribution margin by $25,000. If sales totaled $175,000 when net income equaled $20,000 and total selling and administrative expenses equaled $55,000, then the contribution margin equaled: a. $75,000; b. $80,000; c. $30,000; d. $50,000; e. none of these.



The following data relate to questions 9 through 12.


In the O'Donnell Manufacturing Company, at 80,000 machine hours total overhead costs equaled $223,000. Of this amount, utilities totaled $48,000 (all variable) and depreciation totaled $60,000 (all fixed). The balance of the overhead cost consisted of maintenance cost (mixed). At 100,000 machine hours, maintenance costs totaled $130,000.


9. At 100,000 hours, total overhead costs would equal: a. $250,000; b. $238,000; c. $278,750; d. $195,000; e. none of these.


By use of the high‑low method, the variable cost for maintenance per machine hour would equal: a. $1.30; b. $1.15; c. $.75; d. $1.35; e. none of these.


The total fixed overhead costs for O'Donnell would equal: a. $115,000; b. $130,000; c. $60,000; d. $55,000; e. none of these.


If 110,000 machine hours of activity are projected for next period, total overhead cost would equal: a. $256,000; b. $263,500; c. $306,625; d. $242,500; e. none of these.


Salaries of accounts receivable clerks when one clerical worker is needed for every 750 accounts receivable is an example of: a. fixed cost; b. a step‑ variable cost; c. mixed cost; d. curvilinear cost; e. none of these.


An analysis of the interest cost for Holms Company indicates that interest cost and fees will be $38,000 for a debt of $400,000 and $48,500 for a debt of $550,000. If $600,000 is borrowed by Holms Company, total interest cost and fees will equal (assume that all amounts are within the company's relevant range): a. $57,000; b. $52,800; c. $56,000; d. $52,000; e. none of these.


Quartz Manufacturing Company developed the following overhead cost formulas:


Cost Cost Formulas

Insurance $400

Depreciation $900

Maintenance materials $.15 per machine hour

Utilities $600 + $.20 per machine hour


The total overhead cost expected for Quartz Manufacturing Company if 1,500 hours are worked is: a. $1,825; b. $7,150; c. $6,650; d. $2,425; e. none of these.


The following data, relating to questions 16 and 17, were taken from the records of the IPS company.

Month Units Shipped Shipping Costs

March 9,500 $10,035

April 8,000 $ 8,550

May 7,000 $ 7,350

June 11,000 $11,550


Using the high‑low method, the fixed cost for shipping cost is: a. 0; b. $550; c. $440; d. $350; e. none of these.


17. If 10,000 units are shipped in July, the expected shipping cost would equal: a. $10,100; b. $10,550; c. $10,500; d. $10,750; e. none of these.


The following data relate to questions 18 and 19.


Month Meals served Utilities costs

December 5,500 $401.00

January 3,000 $360.00

February 2,500 $347.50

March 4,000 $385.50

April 6,000 $414.00


18. Using the high‑low method, the variable rate for utilities costs per meal served is: a. $.19; b. $.12; c. $.069; d. $.019; e. none of these.


19. Using the high‑low method, the fixed cost is: a. $66.50; b. $300.00; c. $303.00; d. $331.00; e. none of these.


20. An analysis of the cost for Elequis Company indicates that total maintenance costs (variable and fixed) will be $1.50 per hour at an activity level of 8,000 hours and $1.20 per hour at an activity level of 13,000 hours. Total maintenance cost for a budgeted activity level of 10,000 hours would be: a. $13,200; b. $12,000; c. $13,440; d. $14,400; e. none of these.


The following data relate to questions 21 through 25.


Comparative income statements for Boggs Sports Equipment Company for the last two months are presented below:

July August

Sales in Units 11,000 10,000


Sales Revenue $165,000 $150,000

Less Cost of Goods Sold 72,600 66,000

Gross Margin 92,400 84,000

Less Operating Expenses:

Rent 12,000 12,000

Sales Commissions 13,200 12,000

Maintenance Expenses 13,500 13,000

Clerical Expense 16,000 15,000

Total Operating Expenses 54,700 52,000

Net income $ 37,700 $ 32,000

21. Which of the following classifications describes the cost of goods sold? a. Fixed; b. Mixed; c. Variable; d. Process; e. none of these.


22. Which of the operating expenses is variable? a. Rent; b. Sales Commissions; c. Maintenance Expense; d. Clerical Expense; e. none of the above are variable.


23. The total monthly fixed costs for Boggs Sporting Equipment Company is: a. $12,000; b. $22,500; c. $25,000; d. $40,000; e. none of these.


24. If sales are projected to be 8,000 units in September, total operating expenses would be: a. $49,300; b. $41,600; c. $44,750; d. $46,600; e. none of these.


25. If sales are projected to be 8,000 units in September, total operating expenses would be: a. $49,300; b. $41,600; c. $44,750; d. $46,600; e. none of these.


26. Given the following information for the Evans Retail Stores, Inc., for the first quarter of 19x8:


Sales $350,000

Variable selling expense 35,000

Fixed selling expenses 25,000

Cost of goods sold 160,000

Fixed administrative expenses 55,000

Variable administrative expenses 15,000


The contribution margin of Evans Retail Stores, Inc. for the first quarter of 19x8 is: a. $300,000; b. $140,000; c. $210,000; d. $190,000; e. none of these.


27. At an activity level of 25,000 units, fixed costs totaled $75,000 and the variable cost per unit equaled $2.00. If total costs increase to $175,000, then: a. the activity level increases to 35,000 units; b. total unit cost equals $3.50; c. unit fixed cost equals $3.00; d. the activity level increases to 37,500 units; e. none of these.


The following data relate to questions 28 through 31.


The University Store, Inc. is the major supplier of books for the four area colleges. An income statement for the first quarter of 19x6 is presented below:


University Store, Inc.

Income Statement

For the Quarter Ended March 31, 19x6


Sales $800,000

Cost of Goods Sold 560,000

Gross Margin 240,000

Less operating expenses:

Selling $105,000

Administrative 105,000 210,000

Net income $ 30,000


On average, a book sells for $40.00. Variable selling expenses are $3.00 per book; the remaining selling expenses are fixed. The variable administrative expenses are 5 percent of sales; the remainder are fixed.


28. The contribution margin for the University Store for the first quarter of 19x6 is: a. $660,000; b. $700,000; c. $180,000; d. $140,000; e. none of these.


29. The net income computed using the contribution approach for the first quarter of 19x6 is: a. $30,000; b. $180,000; c. $140,000; d. $0; e. none of these.


30. The cost formula for operating expenses with 'x' equal to the number of books sold is: a. y = $105,000 + $3x; b. y = $105,000 + $5x; c. y = $110,000 + $5x; d. y = $110,000 + $33x; e. none of these.


31. If 25,000 books are sold during the second quarter of 19x6, the company's contribution margin would equal: a. $875,000; b. $300,000; c. $175,000; d. $65,000; e. none of these.


32. An analysis of a particular cost indicates it will be $.65 per hour at an activity level of 18,000 hours and $11,700 in total at an activity level of 20,000 hours. What will this cost be in total at an activity level of 16,000 hours? a. $9,360; b. $10,400; c. $11,250; d. $11,700; e. none of these.


33. At an activity level of 10,000 units, variable costs totaled $35,000 and fixed costs totaled $20,800. If 16,000 units are produced, then: a. total cost would equal $89,280; b. total unit cost would equal $4.80; c. fixed cost per unit would equal $2.08; d. total costs would equal $55,800; e. none of these.


The following data relate to questions 34 through 36.


April May

Sales in Units 3,200 4,500

Cost:

Cost S $6,400 $9,000

Cost T 5,600 5,600

Cost U 7,100 7,100

Cost V 4,480 6,300

cost W 3,950 5,250


34. Which of the following classifications describes Cost T? a. Variable; b. Curvilinear; c. Fixed; d. Mixed; e. none of these.


35. Which of the following classifications describes Cost V? a. Variable; b. Curvilinear; c. Fixed; d. Mixed; e. none of these.


36. Which of the following classifications describes Cost R? a. Variable; b. Curvilinear; c. Fixed; d. Mixed; e. none of these.


37. Sales for 19x8 for Shop Company were $250,000. Net income totaled $40,000 and cost of goods sold were $110,000. If Shop Company's contribution margin equaled $100,000, total variable selling and administrative expenses must equal: a. $40,000; b. $100,000; c. $60,000; d. $150,000; e. none of these.


The following data relate to questions 38 through 41.

Gasson Company is a merchandising firm. During the next month Gasson Company expects to sell 800 units of Product H. Given the following data for Product H:


Sales price per unit $40

Sales Commission 5 percent

Purchase price (cost) of Product H $18

Advertising expense $4,000 per month

Administrative expense $4,500 per month plus 15 percent of sales


38. The expected gross margin next month for Product H is: a. $17,600; b. $11,200; c. $14,400; d. $16,000; e. none of these.


39. The expected total administrative expenses next month relating to Product H is: a. $4,800; b. $13,300; c. $9,300; d. $14,900; e. none of these.


40. The expected contribution margin next month for Product H is: a. $17,600; b. $11,200; c. $14,400; d. $16,000; e. none of these.


41. The expected net income for Product H is: a. $7,500; b. $5,100; c. $2,700; d. $11,200; e. none of these.


42. Junk Food Imports determines that its shipping expense is $18,000 for 16,000 pounds shipped and $22,500 for 22,000 pounds shipped. If the company shipped 18,000 pounds during the current month, projected shipping cost should be: a. $18,500; b. $20,400; c. $19,500; d. $24,000; e. none of these.


43. Which costs will change with a decrease in volume? a. Total fixed costs and total variable cost; b. Unit fixed costs and total variable cost; c. Unit variable cost and unit fixed cost; d. Unit fixed cost and total fixed cost; e. Total fixed cost and total variable cost.


44. The Cost of Goods Sold in the Marin Company totaled $325,000. Fixed selling and administrative expenses totaled $115,000 and variable selling and administrative expenses were $210,000. If Marin Company's contribution margin totaled $590,000, then sales must have been: a. $1,125,000; b. $1,030,000; c. $915,000; d. $650,000; e. none of these.


45. Contribution margin means: a. what remains from total sales after deducting fixed expenses; b. what remains after deducting cost of goods sold to cover fixed and variable expenses; c. the sum of cost of goods sold and variable expenses; d. what remains from total sales after deducting all variable expenses; e. none of these.


46. When production levels are expected to decline within a relevant range, what effects would be anticipated with respect to each of the following?


Fixed costs Variable costs

per unit per unit

a. Increase Increase

b. Increase No change

c. No change No change

d. No change Increase

47. Within a relevant range, the amount of variable cost per unit: a. differs at each production level; b. remains constant at each production level; c. increases as production increases; d. decreases as production increases; e. none of these.


48. Brooks Company uses the following cost formula for the maintenance cost in department T:


Y = $7,200 + $0.60 per machine hour


The July 19x2 operating budget is based upon 20,000 hours of planned machine time. Maintenance cost expected to be incurred during the month is: a. $11,400; b. $12,000; c. $12,600; d. $19,200; e. none of these.


49. The following relationships pertain to activity and cost for two different years for Symthe Company:


Direct‑labor hours 300,000 400,000

Total costs $129,000 $154,000


What are the expected fixed costs for a year? a. $25,000; b. $54,000; c. $75,000; d. $100,000; e. none of these.




IV. COST VOLUME PROFIT



1. Street Company's fixed costs total $150,000, its variable cost ratio is 60 percent and its variable costs are $4.50 per unit. Based on this information, the break‑even point in units is: a. 50,000; b. 37,500; c. 33,333; d. 100,000; e. none of these.


2. Minist Company sells a single product at a selling price of $15.00 per unit. Last year, the company's sales revenue was $225,000 and its net income was $18,000. If fixed expenses totaled $72,000 for the year, the break‑even point in unit sales was: a. 15,000; b. 9,900; c. 14,100; d. 12,000; e. none of these.


3. Mason Company's selling price during 19x7 was $20.00 per unit. Fixed costs totaled $54,000, variable costs were $14.00 per unit, and the company reported a profit of $9,000 for the year. The break‑even point for Mason Company is: a. 10,500 units; b. 4,500 units; c. $171,000; d. $180,000; e. none of these.

4. Last year, Perry Company reported profits of $4,200. It's variable costs totaled $66,000 or $6 per unit. The unit contribution margin was $3.00. The break‑even point in units for Perry Company is: a. 11,000; b. 9,600; c. 22,000; d. 12,400; e. none of these.


5. Last year, Black Company reported sales of $640,000, a contribution margin of $160,000, and a net loss of $40,000. Based on this information, the break‑ even point was: a. $640,000; b. $480,000; c. $800,000; d. $960,000; e. none of these.


6. During 19x7, Flynn Company reported a profit of $70,000 when sales totaled $520,000 and the contribution margin ratio was 40 percent. If fixed costs increase by $10,000 during 19x8, what will sales have to be for the company to earn a profit of $80,000 (ignore taxes)? a. $600,000; b. $570,000; c. $562,500; d. $625,000; e. none of these.


7. The break‑even point can be expressed as the point where: a. sales equal total variable and fixed expenses; b. total contribution margin equals total fixed expenses; c. sales less fixed expenses equal variable expenses; d. all of the above; e. none of these.



The following data relate to questions 8 through 11.


Dorian Company manufactures and sells a single product, Product D. The product sells for $60 per unit and has a C/M ratio of 40 percent. The company's monthly fixed expenses are $28,800.


8. The variable cost per unit of Product D is: a. $31.20; b. $24.00; c. $36.00; d. $28.80; e. none of these.


9. The break‑even point for Product D is: a. $48,000; b. $72,000; c. 800 units; d. 1,000 units; e. none of these.


If Dorian Company desires a monthly income equal to 10 percent of sales, monthly sales will have to be (ignore taxes): a. 1,500 units; b. 760 units; c. 2,000 units; d. 1,600 units; e. none of these.


If the selling price is reduced by 5 percent, variable costs reduced by $1.00, and fixed costs increased to a total of $38,400, how many units would need to be sold to earn an income of $21,000 (ignore taxes)? a. 10,000; b. 27,000; c. 17,000; d. 29,500; e. none of these.


Scott Company's variable costs are 72 percent of sales. The company's break‑even point in sales is $2,450,000. If sales are $60,000 below the break‑ even point, the company would report a: a. $43,200 loss; b. $60,000 loss; c. $16,800 loss; d. cannot be determined from the data given; e. none of these.


Once the break‑even point is reached: a. the total contribution margin changes from negative to positive; b. net income will increase by the unit contribution margin for each item sold; c. variable costs will remain constant in total; d. the contribution margin ratio begins to decrease; e. none of these.


Green Company's variable costs are 75 percent of sales. At a sales level of $400,000, the company's degree of operating leverage is 8. At this sales level, fixed costs equal: a. $87,500; b. $100,000; c. $50,000; d. $75,000; e. none of these.


At a sales level of $90,000, Blue Company's contribution margin is $24,000. If the degree of operating leverage is 6 at a $90,000 sales level, net income must equal: a. $15,000; b. $11,000; c. $4,000; d. $20,000; e. none of these.



The following data relate to questions 16 through 19.


Given the following income statement for the most recent year for the Drake Company:


Sales (26,000 units) $650,000

Less: Variable Expenses 442,000

Contribution Margin 208,000

Less: Fixed Expenses 234,000

Net Loss $(26,000)

========


The unit contribution margin is: a. $17.00; b. $8.00; c. $1.00; d. $9.00; e. none of these.


17. The break‑even point in sales dollars is: a. $731,250; b. $676,000; c. $675,000; d. $720,000; e. none of these.


18. The sales manager is convinced that a $60,000 expenditure in advertising will increase unit sales by fifty percent. If the sales manager is correct, the company's net income would increase by: a. $44,000; b. $34,000; c. $18,000; d. $49,000; e. none of these.


19. Refer to the original data. If the company desires a net income of $20,000, the number of units needed to be sold is (ignore taxes): a. 28,500; b. 31,000; c. 31,750; d. 26,500; e. none of these.


20. The degree of operating leverage is: a. a measure of how a percentage change in sales volume will affect profits; b. equal to the contribution margin divided by the net income; c. greater at sales levels near the break even point; d. all of the above; e. none of these.


21. Brasher Company manufacturers and sells a single product. If the selling price and variable costs both decrease by 5 percent and fixed costs do not change, then: a. the unit contribution margin will decrease and the contribution margin ratio will decrease; b. the unit contribution margin will decrease and the contribution margin ratio will remain the same; c. the unit contribution margin will remain unchanged and the contribution margin ratio will decrease; d. neither the unit contribution margin nor the contribution margin ratio will change; e. none of these.


The following data relate to questions 22 and 23.

Gardner Furniture Company produces two kinds of chairs: an oak model and a chestnut wood model. The oak model sells for $60 and the chestnut wood model sells for $100. The variable expenses are as follows:


Oak Chestnut

Variable production costs per unit $30 $35

Variable selling expenses per unit $ 6 $ 5


Expected sales in units next year are: 5,000 oak chairs and 1,000 chestnut chairs. Fixed expenses are budgeted at $135,000 per year.


22. The yearly break‑even point in total sales for the sales mix expected is: a. $270,000; b. $300,000; c. $485,000; d. $500,000; e. none of these.


23. The company's overall contribution margin ratio for the sales mix expected is: a. 40 percent; b. 45 percent; c. 50 percent; d. 60 percent; e. none of these.


24. For 19x1, Twins Company reported $750,000 in sales (25,000 units) and a net income of $25,000. At the break‑end point, the company's total contribution margin equals $500,000. Based on this information, the company's: a. contribution margin ratio is 40 percent; b. break‑even point is 24,000 units; c. variable cost per unit is $9; d. variable costs are 60 percent of sales; e. none of these.


25. Frank Company manufacturers a single product that has a selling price of $20.00 per unit. Fixed costs total $45,000 per year, and the company must sell 5,000 units to break even. If the company desires to earn a profit of $13,500, sales in units must be: a. 6,000; b. 5,750; c. 6,500; d. 7,925; e. none of these.


26. Sinclair Company's single product has a selling price of $25 per unit. Last year the company reported a profit of $20,000 and variable expenses totaling $180,000. The product has a 40 percent contribution margin ratio. Because of competition, Sinclair Company will be forced in the current year to reduce its selling price by $2.00 per unit. How many units must be sold in the current year to earn the same profit as was earned last year? a. 15,000 units; b. 12,000 units; c. 16,500 units; d. 12,960 units; e. none of these.


27. Holt Company's variable costs are 70 percent of sales. At a $300,000 sales level, the degree of operating leverage is 10. If sales increase by $60,000, the degree of operating leverage will be: a. 12; b. 10; c. 6; d. 4; e. none of these.


The following data relate to questions 28 through 31.


Hurst Co. manufacturers and sells a single product. Price and cost data regarding this product are as follows:


Selling price per unit $ 40.00

Variable manufacturing costs per unit 20.00

Variable selling expenses per unit 6.00

Fixed manufacturing overhead $208,000.00

Fixed administrative costs 324,000.00


28. The break‑even point in units is: a. 15,200 units; b. 26,600 units; c. 38,000 units; d. 40,000 units; e. none of these.


29. How many units need to be sold to earn a net income equal to 10 percent of sales (ignore taxes)? a. 44,000 units; b. 53,200 units; c. 54,500 units; d. 47,500 units; e. none of these.


30. In the current year, the company sold 43,000 units. Due to competition, management will be forced to lower the selling price by 10 percent next year. How many units must be sold next year to earn the same income as was earned in the current year? a. 50,000 units; b. 53,200 units; c. 58,800 units; d. 60,200 units; e. none of these.


31. Refer to the original data. During the current year, the company reported a net income of $56,000. The market research department believes that a new deluxe package for the product, at an additional cost of $2.00 per unit, will have great appeal. The new package, combined with an increased advertising expenditure of $88,000, will result in a 50 percent increase in unit sales. If the marketing department's proposals are adopted, the net income will be: a. $136,000; b. $162,000; c. $182,000; d. $64,000; e. none of these.


32. Solen Company's break‑even‑point in sales is $900,000, and its variable costs are 75 percent of sales. If the company lost $32,000 last year, sales must have amounted to: a. $868,000; b. $804,000; c. $772,000; d. $628,000; e. none of these.


33. The break‑even point in sales for Rice Company is $360,000 and the company's contribution margin ratio is 30 percent. If Rice Company desires an income of $84,000, sales would have to total: a. $280,000; b. $640,000; c. $480,000; d. $560,000; e. none of these.


34. Spencer Company expects to sell 60,000 units of Product B next year. Variable production costs are $4 per unit, and variable selling costs are 10 percent of the selling price. Fixed costs are $115,000 per year, and the company desires a profit of $50,000 next year. Based on this information, the unit selling price next year should be: a. $7.00; b. $10.75; c. $7.50; d. $6.75; e. none of these.


35. Curtis Company anticipates selling 10,000 units next year. The company wants to earn a net income equal to 10 percent of sales. If variable costs are $12 per unit and fixed costs total $78,000 per year, what selling price must be established to achieve the desired level of net income? a. $19.80 per unit; b. $18.00 per unit; c. $21.78 per unit; d.$22.00 per unit; e. none of these.


36. Loren Company's single product has a selling price of $15 per unit. Last year the company reported total variable costs of $180,000, fixed costs of $90,000, and a net income of $30,000. A study by the sales manager discloses that a 15 percent increase in the selling price would reduce unit sales by 10 percent. If her proposal is adopted, net income would: a. increase by $45,000; b. increase by $37,500; c. increase by $7,500; d. increase by $28,500; e. none of these.


37. The contribution margin increases when sales volume remains constant and: a. variable cost per unit decreases; b. variable cost per unit increases; c. fixed costs decrease; d. both variable and fixed costs increase; e. none of these.


38. Black Company's sales for 19x8 are $600,000, its fixed costs are $150,000, and its variable costs are 60 percent of sales. Based on this information, the margin of safety is: a. $90,000; b. $190,000; c. $225,000; d. $240,000; e. none of these.


39. The margin of safety in the Flaherty Company is $24,000. If the company's sales are $120,000 and its variable costs are $80,000, its fixed costs must be: a. $8,000; b. $32,000; c. $24,000; d. $16,000; e. none of these.



The following data relate to questions 40 through 42.


Given the following income statement for Jeffrey Company for 19x7:


Sales (30,000 units) $600,000

Less Operating Expenses:

Variable $390,000

Fixed 140,000 530,000

Net income $ 70,000

========


40. The break‑even point for 19x7 is: a. 26,500 units; b. 17,500 units; c. $460,000; d. $400,000; e. none of these.


41. The Company's degree of operating leverage is: a. 3; b. 2; c. 4.28; d. 8.57; e. none of these.


42. The Company's margin of safety (rounded to the nearest whole percent) is: a. 33 percent; b. 50 percent; c. 12 percent; d. 67 percent; e. none of these.


43. At a break‑even point of 800 units sold, White Company's variable costs are $800 and its fixed costs are $400. What will the Company's net income be at a volume of 801 units? a. $1.50; b. $1.00; c. $.50; d. $2.00; e. none of these.

44. The break‑even volume in units will decrease if there is a(an): a. decrease in unit selling price; b. increase in total fixed costs; c. increase in unit variable costs; d. decrease in sales volume; e. none of these.


45. A $2.00 increase in a product's variable cost per unit accompanied by a $2.00 increase in its selling price per unit will: a. decrease the degree of operating leverage; b. result in a decrease in the contribution margin; c. have no effect on the break‑even volume; d. have no effect on the contribution margin ratio; e. none of these.


46. Gerber Company is planning to sell 200,000 units of product O for $2.00 a unit. The contribution margin is 25 percent. Gerber will break even at this level of sales. What would be the fixed costs? a. $100,000; b. $160,000; c. $200,000; d. $300,000; e. none of these.


47. How may the following be used in calculating the breakeven point in units?


Contribution

Fixed costs margin per unit

a. Denominator Numerator

b. Denominator Not used

c. Numerator Not used

d. Numerator Denominator


48. Marling Company is contemplating an expansion program based on the following data:


Expected sales $600,000

Variable costs 420,000

Fixed expenses 120,000


What is the amount of break‑even sales? a. $400,000; b. $420,000; c. $540,000; d. $660,000; e. none of these.


49. The contribution margin decreases when sales volume remains the same and: a. fixed costs increase; b. fixed costs decrease; c. variable cost per unit increases; d. variable cost per unit decreases; none of these.


50. How would the following be used in calculating sales necessary to realize a projected profit?


Combination

Projected profit margin ratio

a. Denominator Numerator

b. Denominator Not used

c. Numerator Numerator

d. Numerator Denominator


51. In using the unit contribution method to calculate a target sales level expressed in units, which of the following should be subtracted from fixed costs in the numerator? a. Predicted operating loss; b. Predicted operating profit; c. Unit contribution margin; d. Variable costs; none of these.


52. Korn Company sells two products, as follows:


Per Unit

Sales Variable

price costs

Product Y $120 $ 70

Product Z 500 200


Fixed costs total $300,000 annually. The expected sales mix in units is 60 percent for product Y and 40 percent for product Z. How much is Korn's expected breakeven sales in dollars? a. $300,000; b. $420,000; c. $475,000; d. $544,000; none of these.


53. The following information pertains to Nova Co.'s cost‑volume‑profit relationships:


Breakeven point in units sold 1,000

Variable costs per unit $ 500

Total fixed costs $150,000

How much will be contributed to profit before income taxes by the 1,001st unit sold? a. $650; b. $500; c. $150; d. $0; none of these.


54. The contribution margin ratio always increases when the: a. breakeven point increases; b. breakeven point decreases; c. variable costs as a percentage of net sales decrease; d. variable costs as a percentage of net sales increase; none of these.


55. To obtain the breakeven point stated in terms of dollars of sales, total fixed costs are divided by which of the following? a. Variable cost per unit; b. Variable cost per unit divided by sales per unit; c. Fixed cost per unit; d. (Sales price per unit ‑ variable cost per unit) divided by sales price per unit; none of these.


56. Breakeven analysis assumes over the relevant range that: a. total costs are unchanged; b. selling prices are unchanged; c. variable costs are nonlinear; d. fixed costs increase per unit; e. breakeven analysis assumes all of the above.


57. In planning its operations for 19x1 based on a sales forecast of $6,000,000, Wallace, Inc., prepared the following estimated data:


Cost and expenses

Variable Fixed

Direct materials $1,600,000

Direct labor 1,400,000

Factory overhead 600,000 $ 900,000

Selling expenses 240,000 360,000

Administrative expenses 60,000 140,000

$3,900,000 $1,400,000


What would be the amount of sales dollars at the breakeven point? a. $2,250,000; b. $3,500,000; c. $4,000,000; d. $5,300,000; e. none of these.


58. Information concerning Label Corporation's Product A is as follows:


Sales . $300,000

Variable costs . $240,000

Fixed costs . $ 40,000


Assuming that Label increased sales of Product A by 20 percent, what should the net income from Product A be? a. $20,000; b. $24,000; c. $32,000; d. $80,000; e. none of these.


59. Kent Co.'s 19x5 operating percentages were as follows:


Sales .. 100 Percent

Cost of sales:

Variable 50 Percent

Fixed 10 60

Gross profit . 40

Other operating expenses:

Variable 20

Fixed 15 35

Operating income 5 Percent

==

Kent's 19x5 sales totaled $2,000,000. At what 19x5 sales level would Kent break even? a. $1,900,000; b. $1,666,667; c. $1,250,000; d. $833,333; e. none of these.


60. Which of the following would cause the breakeven point to change? a. Sales increased; b. Total production decreased; c. Total variable costs increased as a function of higher production; d. Sales decreased; e. none of these.



The following data relate to questions 61 through 63.


Oslo Co.'s industrial photo‑finishing division, Rho, incurred the following costs and expenses in 19x5:


Variable Fixed

Direct materials $200,000

Direct labor 150,000

Factory overhead 70,000 $42,000

General, selling, and

administrative 30,000 48,000

Total $450,000 $90,000


During 19x5, Rho produced 300,000 units of industrial photo‑prints, which were sold for $2.00 each.


61. How many industrial photo‑print units did Rho have to sell in 19x5 to break even? a. 180,000; b. 120,000; c. 90,000; d. 60,000; none of these.


62. For the year ended December 31, 19x5, Rho's contribution margin was: a. $250,000; b. $180,000; c. $150,000; d. $60,000; none of these.


63. Based on Rho's 19x5 financial data, and an estimated 19x6 production of 350,000 units of industrial photo‑prints, Rho's estimated 19x6 total costs and expenses would be: a. $525,000; b. $540,000; c. $615,000; d. $630,000; none of these.


64. The following information pertains to Rica Company:


Sales (50,000 units) $1,000,000

Direct materials and direct labor 300,000

Factory overhead:

Variable 40,000

Fixed 70,000

Selling and general expenses:

Variable 10,000

Fixed 60,000


How much was Rica's break‑even point in number of units? a. 9,848; b. 10,000; c. 18,571; d. 26,000; none of these.



The following data relate to questions 65 and 66.


Taylor, Inc. produces only two products, Acdom and Belnom. These account for 60 percent and 40 percent of the total sales dollars of Taylor, respectively. Variable costs (as a percentage of sales dollars) are 60 percent for Acdom and 85 percent for Belnom. Total fixed costs are $150,000. There are no other costs.


65. What is Taylor's breakeven point in sales dollars? a. $150,000; b. $214,286; c. $300,000; d. $500,000; none of these.

66. Assuming that the total fixed costs of Taylor increase by 30 percent, what amount of sales dollars would be necessary to generate a net income of $9,000? a. $204,000; b. $464,000; c. $659,000; d. $680,000; none of these.



V. VARIABLE COSTING



The following data relate to questions 1 through 3.


Yankee Company manufactures a single product. Assume the following data for 19x7:


Variable costs per unit:

Selling and Administrative . $1

Production .. $4

Fixed costs in total:

Production .. $12,000

Selling and Administrative . $ 8,000


During 19x8, 4,000 units were produced and 3,500 units were sold.


1. Under direct costing, the cost of one unit of product would be: a. $4; b. $5; c. $7; d. $8; e. none of these.


2. The inventory carrying value of finished goods under direct costing would be: a. the same as under absorption costing; b. $1,500 higher than under absorption costing; c. $2,000 higher than under absorption costing; d. $2,000 less than under absorption costing; e. $1,500 less than under absorption costing.


3. Under absorption costing, the cost of goods sold for 19x8 would be: a. $28,000; b. $24,500; c. $17,500; d. $14,000; e. none of these.



The following data relate to questions 4 through 6.


Fahey Company manufactures a single product. Assume the following data for 19x7:


Fixed costs in total:

Selling and Administrative . $54,000

Production .. $72,000

Variable costs per unit:

Selling and Administrative . $3

Production .. $9


There were no units in inventory on January 1. During the year, 18,000 units were produced and 15,000 units were sold.


4. Under absorption costing, the cost of one unit of product would be: a. $9; b. $12; c. $13; d. $16; e. none of these.


5. Assume that the selling price of Fahey Company's product is $25 per unit. The company's net income for 19x7 under the direct costing method would be: a. $60,000; b. $81,000; c. $57,000; d. $69,000; e. none of these.


6. Assume that in the following year, 19x8, sales exceed production by 2,000 units. The net income under direct costing would be: a. the same as under absorption costing; b. $8,000 greater than under absorption costing; c. $6,000 less than under absorption costing; d. $12,000 greater than under absorption costing; e. none of these.


7. Which of the following statements is true for a firm that uses direct costing? a. The production cost per unit changes as a result of changes in the number of units manufactured; b. Both variable selling costs and variable production costs are included in the cost of a unit of product; c. Net income fluctuates directly with changes in sales; d. Net income is greatest in periods when production is highest; e. None of the above statements is true.


8. On January 1, 19x5, Welk Company had 16,000 units in its beginning inventory. During the year, the company's variable production costs were $6 per unit and its fixed overhead costs were $4 per unit. The company's net income for the year was $24,000 higher under absorption costing than it was under direct costing. Given these facts, the number of units of product in the ending inventory on December 31, 19x5 must have been: a. 22,000 units; b. 10,000 units; c. 6,000 units; d. 4,000 units; e. none of these.


9. During 19x4, Blake Company's income under absorption costing was $3,600 lower than its income under direct costing. The company sold 10,000 units during the year, and its variable costs were $9 per unit, of which $1 was variable selling expense. If production cost was $11 per unit under absorption costing, then how many units did the company produce during the year? a. 8,200 units; b. 8,800 units; c. 11,200 units; d. 11,800 units; e. none of these.



The following data relate to questions 10 through 13.


Blake Corporation

Income Statement‑Absorption Costing

For the month ended June 30, 19x6


Sales (9,500 units) .. $285,000

Cost of Goods Sold:

Beginning Inventory . $ 16,000

Cost of Goods Manufactured 160,000

Goods Available for Sale .. 176,000

Ending Inventory . 24,000

Cost of Goods Sold .. 154,000

Gross Margin . 131,000

Less Operating Expenses:

Fixed Administrative

Expense 75,000

Variable Selling Expense 19,000

Total Operating Expenses . 94,000

Net Income .. $ 37,000


During June, the company's variable production costs were $10 per unit and its fixed manufacturing overhead totaled $60,000.


The contribution margin per unit during June was: a. $20.; b. $18; c. $16; d. $14; e. none of these.


During June, the company's variable production costs were $10 per unit and its fixed manufacturing overhead totaled $60,000. The dollar value of the company's inventory on June 30 under the direct costing method would be: a. $10,000; b. $12,000; c. $15,000; d. $24,000; e. none of these.


Net income under the direct costing method for June would be: a. $34,000; b. $40,000; c. $53,000; d. $60,000; e. none of these.


The break‑even point in units for the month would be: a. 6,000 units; b. 6,750 units; c. 7,500 units; d. 9,000 units; e. none of these.


The following data relate to questions 14 through 17.


During 19x3, Hurlex Company manufactured 15,000 units and sold 12,000 units. Production costs for the year were as follows:


Fixed Overhead $210,000

Variable Overhead $135,000

Direct Labor .. $180,000

Direct Materials . $150,000


Sales totaled $840,000 for the year, variable selling expenses totaled $60,000, and fixed selling and administrative expenses totaled $180,000. There were no units in the beginning inventory.

Under absorption costing, the cost of one unit of product would be: a. $31; b. $35; c. $45; d. $28.75; e. none of these.


The contribution margin per unit would be: a. $25; b. $39; c. $34; d. $35; e. none of these.


Under absorption costing, the ending inventory for the year would be valued at: a. $135,000; b. $93,000; c. $105,000; d. $0; e. none of these.


17. Under direct costing, the company's net income for the year would be: a. $42,000 higher than under absorption costing; b. $30,000 higher than under absorption costing; c. $30,000 lower than under absorption costing; d. $42,000 lower than under absorption costing; e. none of these.


The following data relate to questions 18 through 20.


Crystal Company

Income Statement ‑ Direct Costing Method

For the month ended May 31, 19x2


Sales ($10 per unit) .. $900,000

Less Variable Costs:

Variable Cost of Goods Sold:

Beginning Inventory $125,000

Variable Cost of Goods

Manufactured .. 400,000

Goods available for Sale . $525,000

Ending Inventory 75,000

Variable Cost of Goods Sold . $450,000

Variable Selling Expense .. 90,000

Total Variable Costs .. 540,000

Contribution Margin $360,000

Fixed Costs:

Manufacturing $240,000

Selling and Administrative .. 90,000

Total Fixed Costs .. 330,000

Net Income $ 30,000


During May 19x2, 80,000 units were manufactured. Variable production costs have remained constant on a per unit basis over the past several months.


18. The dollar value of the company's inventory on May 31 under the absorption costing method would be: a. $120,000; b. $90,000; c. $75,000; d. 60,000; e. none of these.


19. Under absorption costing, for the month ended May 31, 19x2, the company would report a: a. $30,000 loss; b. $0 profit; c. $30,000 profit; d. $60,000 profit; e. none of these.


20. Crystal Company's total costs at the break‑even point would be: a. $900,000; b. $825,000; c. $495,000; d. $412,500; e. none of these.


21. For 19x1, Arco Company's net income computed by the absorption costing method was $6,400, and its net income computed by the direct costing method was $9,100. The company's unit cost was $17 under direct costing and $20 under absorption costing. If the ending inventory consisted of 2,100 units, the beginning inventory in units must have been: a. 1,200; b. 2,100; c. 3,000; d. 4,800; e. none of these.


22. Bosco Company was organized on January 2, 19x3, and manufactures a single product. If the number of units produced during 19x3 exceeds the number of units sold, then net income under direct costing for the year will: a. be equal to income under absorption costing; b. be greater than income under absorption costing; c. be less than income under absorption costing; d. none of these.


23. During 19x5, Silver Company's variable production costs totaled $7,500 and its fixed overhead costs totaled $4,500. The company produced 3,000 units during the year and sold 2,400 units. Assuming no units in the beginning inventory: a. under direct costing, the units in the ending inventory will be costed at $4 each; b. the net income under absorption costing for the year will be $900 lower than the net income under direct costing; c. the ending inventory under direct costing will be $900 lower than the ending inventory under absorption costing; d. none of these.


24. When sales are constant, but the production level fluctuates, net income determined by the absorption costing method will: a. tend to fluctuate in the same direction as fluctuations in the level of production; b. tend to remain constant; c. tend to fluctuate inversely with fluctuations in the level of production; d. none of these.


25. An allocated portion of fixed factory overhead is included in work in process inventory under:


Absorption Direct

costing costing

a. No No

b. No Yes

c. Yes No

d. Yes Yes


26. During May, Roy Co. produced 10,000 units of Product X. Costs incurred by Roy during May were as follows:


Direct materials .. $10,000

Direct labor 20,000

Variable manufacturing overhead .. 5,000

Variable selling and general .. 3,000

Fixed manufacturing overhead .. 9,000

Fixed selling and general .. 4,000

Total $51,000

Under absorption costing, Product X's unit cost was: a. $5.10; b. $4.40; c. $3.80; d. $3.50; e. none of these.


27. In an income statement prepared as an internal report using the direct costing method, variable selling and administrative expenses would: a. not be used; b. be used in the computation of the contribution margin; c. be used in the computation of operating income but not in the computation of the contribution margin; d. be treated the same as fixed selling and administrative expenses; e. none of these.


28. The absorption costing method includes in inventory:


Fixed factory Variable factory

overhead overhead

a. No No

b. No Yes

c. Yes Yes

d. Yes No


The following data relate to questions 29 and 30.


Osawa Inc. planned and actually manufactured 200,000 units of its single product in 19x2, its first year of operations. Variable manufacturing costs were $30 per unit of product. Planned and actual fixed manufacturing costs were $600,000 and selling and administrative costs totaled $400,000 in 19x2. Osawa sold 120,000 units of product in 19x2 at a selling price of $40 per unit.


29. Osawa's 19x2 operating income using absorption costing is: a. $200,000; b. $440,000; c. $600,000; d. $840,000; e. none of these.


30. Osawa's 19x2 operating income using direct costing is: a. $200,000; b. $440,000; c. $800,000; d. $600,000; e. none of these.


The following data relate to questions 31 and 32.


Lina Co. produced 100,000 units of Product Zee during the month of June. Costs incurred during June were as follows:


Direct materials used . $100,000

Direct labor used .. 80,000

Variable manufacturing overhead 40,000

Fixed manufacturing overhead 50,000

Variable selling and general expenses 12,000

Fixed selling and general expenses 45,000

$327,000


31. What was Product Zee's unit cost under absorption costing? a. $3.27; b. $2.70; c. $2.20; d. $1.80; e. none of these.


32. What was Product Zee's unit cost under direct costing? a. $2.82; b. $2.70; c. $2.32; d. $2.20; e. none of these.


33. In an income statement prepared as an internal report using the direct costing method, fixed selling and administrative expenses would: a. not be used; b. be used in the computation of the contribution margin; c. be used in the computation of operating income but not in the computation of the contribution margin; d. be treated the same as variable selling and administrative expenses; e. none of these.


VI. SEGMENT ANALYSIS



The following data relate to questions 1 through 5.


Ring, Incorporated's income statement for the most recent month is given below.


Total Store P Store Q

Sales $600,000 $200,000 $400,000

Variable Expenses 384,000 144,000 240,000

Contribution Margin . 216,000 56,000 160,000

Traceable Fixed Costs .. 152,000 42,000 110,000

Segment Margin 64,000 $ 14,000 $ 50,000

Common Fixed Costs .. 34,000

Net Income .. $ 30,000


For each of the following questions, refer back to the original

data.


1. If Store Q sales increase by $30,000 with no change in fixed costs, the overall company net income will: a. increase by $3,750; b. increase by $7,500; c. increase by $12,000; d. increase by $18,000; e. none of these.


2. The marketing department believes that a promotional campaign at Store P costing $5,000 will increase sales by $15,000. If the campaign is adopted, overall company net income will: a. decrease by $800; b. decrease by $5,800; c. increase by $5,800; d. increase by $10,000; e. none of these.

3. A proposal has been made that will lower variable costs in Store P to 65 percent of sales. However, this reduction can only be accomplished by a $16,000 increase in Store P's traceable fixed costs. If this proposal is implemented and sales remain constant, overall company net income will: a. remain the same; b. decrease by $2,000; c. increase by $2,000; d. increase by $14,000; e. none of these.


4. If sales in Store Q increase by $30,000 as a result of a $7,000 increase in traceable fixed costs: a. Store Q's contribution margin will increase by $18,000; b. Store Q's segment margin will increase by $12,000; c. Store Q's contribution margin will increase by $11,000; d. the overall company net income will increase by $11,000; e. Store Q's segment margin will increase by $5,000.


5. Currently the sales clerks receive a salary of $17,000 per month in Store Q. A proposal has been made to change from a fixed salary to a sales commission of 5 percent. Assume that this proposal is adopted, and that as a result sales in Store Q increase by $40,000. The new segment margin for Store Q will be: a. $47,000; b. $61,000; c. $85,000; d. $44,000; e. none of these.


6. Channing Company has two divisions, S and T. The company's overall contribution margin ratio is 30 percent when sales in the two divisions total $750,000. If variable costs are $450,000 in Division S, and if Division S's contribution margin ratio is 25 percent, then sales in Division T must be: a. $75,000; b. $150,000; c. $225,000; d. $300,000; e. none of these.


7. All other things being equal, if a division's traceable fixed expenses increase: a. the division's contribution margin ratio will decrease; b. the division's segment margin ratio will remain the same; c. the division's segment margin will decrease; d. the overall company profit will remain the same; e. none of these.


8. Koen Company consists of two divisions, C and D. In 19x1, Koen Company reported a contribution margin of $50,000 for Division C. Division D had a contribution margin ratio of 30 percent and sales in Division D were $250,000. Net income for the company was $30,000 and traceable fixed expenses for the two divisions totaled $50,000. Lyons Company's common fixed expenses for 19x1 were: a. $95,000; b. $75,000; c. $45,000; d. $40,000; e. none of these.


9. Reardon Retail Company consists of two stores, A and B. Store A had sales of $80,000 during March, a contribution margin ratio of 30 percent, and a segment margin of $11,000. The company as a whole had sales of $200,000, a contribution margin ratio of 36 percent, and segment margins for the two stores totaling $31,000. If net income for the company was $15,000 for the month, the traceable fixed costs in Store B must have been: a. $16,000; b. $20,000; c. $31,000; d. $28,000; e. none of these.


Leis Retail Company has two Stores, M and N. Store N had sales of $180,000 during March, a segment margin of 30 percent, and traceable fixed costs of $26,000. The company as a whole had a contribution margin ratio of 25 percent and $120,000 in total contribution margin. Based on this information, total variable costs in Store M for the month must have been: a. $140,000; b. $260,000; c. $300,000; d. $360,000; e. none of these.


Sturr Market has 3 stores: P, Q, and R. During 19x8, Store P had a contribution margin of $24,000 and a contribution margin ratio of 30 percent. Store Q had variable costs of $48,000 and a contribution margin ratio of 40 percent. Store R had variable costs of $84,000, which represented 70 percent of sales in the store. For 19x8, Sturr Market's total sales were: a. $320,000; b. $360,000; c. $440,000; d. $280,000; e. none of these.


Fenway Market has two stores, F and G. During 19x8, Store F had a segment margin of $10,000, traceable fixed costs of $26,000, and variable costs equal to 55 percent of sales. Fenway Market as a whole had a segment margin of 15 percent, a contribution margin ratio of 40 percent, and total sales of $180,000 for the year. Based on this information, the traceable fixed costs in Store G for the year were: a. $19,000; b. $17,000; c. $30,000; d. $36,000; e. none of these.


Insider Company has two divisions, J and K. During 19x2, the contribution margin in J was $30,000. The contribution margin ratio in K during 19x2 was 40 percent, its sales were $125,000, and its segment margin was $32,000. The common fixed expenses in the company were $40,000, and the company's net income for the year was $18,000. The segment margin for Division J for 19x2 was: a. $26,000; b. $32,000; c. $8,000; d. $58,000; e. none of these.


Denner Company has two divisions, A and B. Assume the following data for the two divisions for 19x3:


Division A Division B

Sales .. $90,000 $150,000

Variable cost as a

percentage of sales . 70 Percent 60 Percent

Segment margin .. $ 2,000 $ 23,000


If common fixed costs were $31,000 during 19x3, total fixed costs for the year would be: a. $31,000; b. $62,000; c. $93,000; d. $52,000; e. none of these.



The following data relate to questions 15 and 17.


Higgins Company sells three products, Product A, Product B, and Product C. Sales during June 19x4 totaled $1,500,000 in the company. The company's overall contribution margin ratio was 38 percent, and its fixed costs totaled $525,000 for the year. Sales by product were: Product A, $750,000; Product B, $450,000; and Product C, $300,000. Traceable fixed costs were: Product A, $180,000; Product B, $150,000; and Product C, $90,000. The variable costs were: Product A, $450,000; Product B, $270,000; and Product C, $ ?___.


The net income for the company as a whole for June was: a. $45,000; b. $105,000; c. $150,000; d. $570,000; e. none of these.


The contribution margin ratio for Product C for June was: a. 0 percent; b. 30 percent; c. 38 percent; d. 70 percent; e. none of these.


17. Common fixed costs for Higgins Company for June were: a. $45,000; b. $420,000; c. $150,000; d. $105,000; e. none of these.



The following data relate to questions 18 through 21.


Ieso Company has two stores: J and K. During 19x5, Ieso Company reported a net income of $30,000 and sales of $450,000. The contribution margin in Store J was $100,000, or 40 percent of sales. The segment margin in Store K was $30,000, or 15 percent of sales. Traceable fixed costs are $60,000 in Store J, and $40,000 in Store K.


18. For 19x4, sales in Store J totaled: a. $400,000; b. $250,000; c. $150,000; d. $100,000; e. none of these.


19. For 19x4, variable costs in Store K totaled: a. $70,000; b. $110,000; c. $200,000; d. $130,000; e. none of these.


20. Ieso Company's total fixed costs for 19x4 were: a. $40,000; b. $100,000; c. $140,000; d. $170,000; e. none of these.


21. For 19x4, the segment margin ratio in Store J was: a. 16 percent; b. 24 percent; c. 40 percent; d. 60 percent; e. none of these.


22. Sullivan Retailers has two stores: R and T. During 19x6, Store R had a segment margin of $26,000, traceable fixed costs of $34,000, and a contribution margin ratio of 20 percent. Store T had sales of $180,000, a contribution margin ratio of 40 percent, and a segment margin ratio of 5 percent. For 19x6, variable costs for the company as a whole totaled: a. $132,000; b. $240,000; c. $312,000; d. $348,000; e. none of these.



VII. DECISION MAKING



1. Consider the following production and cost data for two products, X and Y:


Product X Product Y

Contribution margin per unit $24 $18

Machine‑hours needed per unit .. 3 hours 2 hours


The company has 15,000 machine hours available each period, and there is unlimited demand for each product. What is the largest possible total contribution margin that can be realized each period? a. $120,000; b. $125,000; c. $135,000; d. $150,000; e. none of these.



The following data relate to questions 2 through 4.


The Bernard Company has a piece of equipment with an original cost of $126,000. It is now December 31 and this equipment has accumulated depreciation of $90,000 and a salvage value of $15,000. Bernard is now pondering the possibility of buying a new piece of equipment to replace the existing equipment. This new machine would cost $154,000 and have no salvage value at the end of its useful life of five years. The annual operating cost of the new equipment is $18,000.


Alternatively, Bernard can continue using the existing equipment for the next five years at an annual operating cost of $45,000; the old equipment will have no salvage value at the end of five years.


Bernard Company is trying to determine whether to keep the existing equipment for five years or buy the new equipment and use it for five years.


2. Given the data above, how much should Bernard regard as a sunk cost? a. $90,000; b. $126,000; c. $0; d. $36,000; e. none of these.


3. If Bernard chooses to buy the new equipment, the increase or decrease in total company net income over the next five years will be; a. $5,000 increase; b. $19,000 decrease; c. $4,000 decrease; d. $71,000 decrease; e. none of these.


4. At what price for the new equipment would Bernard be economically indifferent between keeping the existing equipment or purchasing the new equipment? a. $150,000; b. $160,000; c. $170,000; d. $135,000; e. none of these.


5. Consider the following statements:


I. The integrated firm is more dependent on its suppliers.

II. Many firms feel they can control quality better by making their own parts.

III. The integrated firm realizes profits from the parts it is 'making' instead of 'buying' as well as profits from its regular operations.


Which of the above statements represent advantages to a firm that is vertically integrated? a. Only I; b. Only III; c. Only I and II; d. Only II and III; e. I, II and III.


6. The Garey Company has 3,000 circuit boards (all alike) which are out of date and are carried in inventory at a total cost of $216,000. The circuit boards can be reworked and upgraded at a total cost of $63,000 and then sold for $110,000. As an alternative, the company can sell these circuit boards to an outside buyer for $48,000. If Garey chooses to upgrade the circuit boards rather than sell them to the outside buyer, the opportunity cost to Garey is: a. $48,000; b. $1,000; c. $27,000; d. $116,000; e. none of these.


The following data relate to questions 7 through 11.


The Rodgers Company makes 27,000 units of a certain component each year for use on its production line. The cost per unit for the component at this level of activity is as follows:


Direct materials .. $4.20

Direct labor $12.00

Variable factory overhead .. $5.80

Fixed factory overhead .. $6.50


Rodgers has received an offer from an outside supplier who is willing to provide 27,000 units of this component each year at a price of $25 per component.


7. Assume that there is no other use for the facilities now being used to produce the component. If Rodgers Company continues to make the components, how much higher or lower will net income be than if the components are purchased from the outside supplier? a. $94,500 lower; b. $81,000 higher; c. $237,600 higher; d. $124,000 lower; e. none of these.


8. Assume that the facilities now being used to make the component could be rented to another company for $64,800 per year if the component were purchased from the outside supplier. Under these conditions, if Rodgers continued to make the component, how much higher or lower will net income be than if the component were purchased from the outside supplier? a. $29,700 lower; b. $21,500 lower; c. $16,200 higher; d. $18,100 higher; e. none of these.


9. Suppose the conditions are the same as in the previous question. At what price per unit charged by the outside supplier would Rodgers be economically indifferent between making the component or buying it from the outside? a. $24.40; b. $22.50; c. $25.00; d. $19.60; e. none of these.


Assume that if the component is purchased from the outside supplier, $35,100 of annual fixed factory overhead could be avoided and the facilities now being used to make the component could be rented to another company for $64,800 per year. If Rodgers chooses to buy the component from the outside supplier under these circumstances, then the change in annual net income due to accepting the offer is: a. $18,900 decrease; b. $18,900 increase; c. $21,400 decrease; d. $21,400 increase; e. none of these.


Suppose the conditions are the same as in the previous question and the number of units of this component used each year can change. What would be the quantity of units of this component used per year that would make Rodgers economically indifferent between making or buying the component? a. 21,600 units; b. 27,000 units; c. 28,500 units; d. 33,300 units; e. none of these.


Consider the following statements:


I. A firm that produces all of its own parts runs the risk of destroying long‑run relationships with suppliers which may be harmful to the firm.

II. The integrated firm is less dependent on its suppliers.

III. Changing technology often makes continued production of one's own parts more costly than buying them from the outside.


Which of these statements indicate hazards to a firm that arise from being vertically integrated? a. Only I; b. Only II; c. Only I and II; d. Only I and III; e. I, II and III.


Consider a decision facing a firm of either accepting or not accepting a special offer for one of its products. A cost that is not relevant to a decision of this type is: a. direct materials; b. direct labor; c. variable factory overhead; d. fixed factory overhead that will be avoided if the special offer is accepted; e. fixed factory overhead that will continue even if the special offer is not accepted; e. none of these.


The following data relate to questions 14 through 16.


The Tolar Company has 400 obsolete desk calculators that are carried in inventory at a total cost of $26,800. If these calculators are upgraded at a total cost of $10,000, they can be sold for a total selling price of $30,000. As an alternative, the calculators can be sold in their present condition for $11,200.


The sunk cost in this situation is: a. $10,000; b. $26,800; c. $11,200; d. $0; e. none of these.


What is the net advantage or disadvantage to the company from upgrading and selling the calculators? a. $8,800 advantage; b. $18,000 disadvantage; c. $20,000 advantage; d. $8,000 disadvantage; e. none of these.


Assume that Tolar decides to upgrade the calculators. At what selling price per unit would the company be as well off as if it just sold the calculators in their present condition? a. $8; b. $30; c. $53; d. $67; e. none of these.


17. The Milham Company has two divisions ‑ East and West. The divisions have the following revenues and expenses:

East West

Sales . $720,000 $350,000

Variable costs . 370,000 240,000

Direct fixed costs 130,000 80,000

Allocated corporate costs .. 110,000 50,000

Net income (loss) . 110,000 (20,000)


The management at Milham is pondering the elimination of the West division. If the West division were eliminated, its direct fixed costs could be avoided. Given these data, the elimination of the West Division would result in an overall company net income of: a. $110,000; b. $80,000; c. $130,000; d. $60,000; e. none of these.


18. Two or more different products that are produced from a common input are known as: a. substitute products; d. main products; b. by‑products; c. joint products; d. scrap; e. none of these.


The following data relate to questions 19 through 23.


The Varone Company makes a single product called a Hom. The company has the capacity to produce 40,000 Homs per year. Per unit costs to product and sell one Hom at the 40,000 ‑ unit activity level are as follows:


Direct materials .. $20

Direct labor 10

Variable factory overhead .. 5

Fixed factory overhead .. $ 7

Variable selling expense 8

Fixed selling expense 2


The regular selling price for one Hom is $60. A special order has been received at Varone from the Fairview Company to purchase 8,000 Homs next year at 15 percent off the regular selling price. If this special order were accepted, the variable selling expense would be reduced by 25 percent. However, Varone would have to purchase a specialized machine to engrave the Fairview name on each Hom in the special order. This machine would cost $12,000 and it would have no use after the special order was filled.


19. If Varone can expect to sell 32,000 Homs next year through regular channels and the special order is accepted, the change in net income next year at Varone due to accepting this order is: a. $52,000 increase; b. $80,000 increase; c. $24,000 decrease; d. $68,000 increase; e. none of these.


20. If Varone can expect to sell 32,000 Homs next year through regular channels, at what special order price from Fairview would Varone be economically indifferent to either accepting or not accepting this special order? a. $51.00; b. $48.20; c. $42.50; d. $39.60; e. none of these.


21. If Varone has an opportunity to sell 37,960 Homs next year through regular channels and the special order is accepted, the change in net income next year at Varone due to accepting the special order is: a. $33,320 decrease; b. $33,320 increase; c. $35,480 decrease; d. $35,480 increase; e. none of these.


Assume that Varone has an opportunity to sell 37,960 homes next year through regular channels. Also assume that the number of units in the special order can be adjusted as needed. At what number of Homs in the special order would Varone be economically indifferent between accepting or rejecting the special order? a. 2,040 units; b. 3,240 units; c. 4,160 units; d. 5,250 units; e. none of these.


23. Suppose that instead of offering to pay 15 percent off the regular selling price, Fairview offers to pay a fixed fee of $3 per Hom plus all production costs (both fixed and variable) associated with the special order. All other given data remain the same. If Varone has an opportunity to sell 32,000 Homs through regular channels and the special order is accepted, the change in net income next year at Varone due to accepting this order is: a. $12,000 decrease; b. $24,000 decrease; c. $32,000 decrease; d. $32,000 increase; e. none of these.


24. Consider the following statements:


I. Assemble all costs associated with each alternative being considered.

II. Eliminate those costs that are sunk.

III. Eliminate those costs that differ between alternatives.


Which of the above statements does not represent a step in identifying the relevant costs in a decision problem? a. Only I; b. Only II; c. Only III; d. Only I and III; e. Only II and III.


The following data relate to questions 25 through 29.


Austin Wool Products purchases raw wool and processes it into yarn. The spindles of yarn can then be sold directly to stores or they can be used by Austin Wool Products in the manufacture of afghans. Each afghan requires the use of one spindle of yarn for completion. Current cost and revenue data for the spindles of yarn and for the afghans are as follows:


Data for one spindle of yarn:

Selling price .. $12

Variable production cost 8

Fixed production cost (based on 4,000 spindles of yarn produced) .. 2


Data for one afghan:

Selling price .. $32

Production cost per spindle of yarn . 10

Variable production cost to process the yarn into an afghan . 9

Avoidable fixed production cost to process the yarn

into an afghan (based on 4,000 afghans produced) . 5


Each month 4,000 spindles of yarn are produced which can either be sold outright or processed into afghans.


25. If Austin chooses to produce 4,000 afghans each month, the change in the monthly net income as compared to selling 4,000 spindles of yarn is: a. $24,000 decrease; b. $24,000 increase; c. $16,000 decrease; d. $16,000 increase; e. none of these.


26. What is the lowest price Austin should be willing to accept for one afghan before it chooses to sell spindles of yarn to the outside market? a. $32; b. $30; c. $28; d. $26; e. none of these.


27. Suppose now that it takes 1.25 spindles of yarn to make one afghan instead of the given information. This means that the cost of yarn for one afghan is $12.50 instead of $10 and that 3,200 afghans can be produced each month from 4,000 spindles of yarn. If Austin chooses to produce 3,200 afghans each month, the change in monthly net income as compared to selling 4,000 spindles of yarn is: a. $34,400 decrease; b. $15,200 increase; d. $10,800 decrease; c. $5,600 increase; e. none of these.


28. Suppose the conditions are the same as in the previous question. What is the lowest price Austin should be willing to accept for one afghan before it chooses to sell spindles of yarn to the outside market? a. $30.25; b. $31.65; c. $32.90; d. $39.40; e. none of these.


29. Suppose it takes 1.25 spindles of yarn to make one afghan instead of the given information and that the number of spindles of yarn produced each month can vary, with the cost data adjusted accordingly. At what quantity of spindles of yarn produced each month would Austin be economically indifferent to processing the yarn into afghans or selling the spindles of yarn to the outside market? a. 1,087; b. 2,900; c. 3,125; d. 3,450; e. 3,615.


30. Consider the following statements:


I. The division's net income, after deducting both direct and allocated costs, is negative.

II. The division's direct fixed costs exceed its contribution margin.

III. The division's direct fixed costs plus its allocated corporate costs exceed its contribution margin.


Which of the above statements give an economic reason for eliminating the division? a. Only I; b. Only II; c. Only III; d. Only I and II; e. I, II and III.


31. Landor Appliance Company makes and sells electric fans. Each fan regularly sells for $42. The following cost data per fan is based on a full capacity of 150,000 fans produced each period.


Direct materials $ 8

Direct labor . 9

Factory overhead (70% variable and 30% unavoidable fixed) . 10


A special order has been received by Landor for a sale of 25,000 fans to an overseas customer. The only selling costs that would be incurred on this order would be $4 per fan for shipping. Landor is now selling 120,000 fans through regular channels each period. What should Landor use as a minimum selling price per fan in negotiating a price for this special order? a. $28; b. $27; c. $31; d. $24; e. none of these.



The following data relate to questions 32 through 35.


The Clemson Company reported the following results last year for the manufacture and sale of a product known as a Tam.


Sales ‑ 6,500 Tams at $130 each $845,000

Variable cost of sales 390,000

Variable distribution costs . 65,000

Fixed advertising expense 275,000

Salary of line manager 25,000

Fixed general factory OH (allocated on the basis of MH) 145,000

Net loss .. (55,000)


Clemson Company is trying to determine whether or not to discontinue the manufacture and sale of Tams. The operating results reported above for last year are expected to continue in the foreseeable future if the product is not dropped.


32. Assume that discontinuing the manufacture and sale of Tams will have no effect on the sale of other product lines. If the company discontinues the Tam product line, the change in annual net income due to this decision will be: a. $55,000 decrease; b. $65,000 decrease; c. $90,000 decrease; d. $70,000 increase; e. none of these.


33. Suppose that the price of Tams next year drops to $110 and all other conditions stay the same. If Clemson continues making and selling the Tams next year the change in net income next year due to this decision as compared to discontinuing the line would be: a. $40,000 decrease; b. $90,000 increase; c. $48,000 increase; d. $65,000 decrease; e. none of these.


34. Refer to the original data. Assume that discontinuing the Tams line would result in a $120,000 increase in the contribution margin of other product lines. If Clemson chooses to discontinue the Tams line, then the change in net income next year due to this action as compared to continuing the line would be: a. $30,000 decrease; b. $30,000 increase; c. $90,000 increase; d. $50,000 increase; e. none of these.


35. Refer to the original data. Assume that discontinuing the Tams line would result in a $120,000 increase in the contribution margin of other product lines. How many Tams would have to be sold next year for the company to be as well off as if it just dropped the line and enjoyed the increase in contribution margin from other products? a. 5,000 units; b. 6,000 units; c. 6,500 units; d. 7,000 units; e. none of these.


The following data relate to questions 36 and 37.


The Flint Fan Company is considering the addition of a new model fan, the F‑27, to its current product lines. The expected cost and revenue data for the F‑27 fan are as follows:


Annual sales .. 4,000 units

Unit selling price .. $58

Unit variable costs:

Production . $34

Selling . $4

Avoidable direct fixed costs per year:

Production . $20,000

Selling . $30,000


If the F‑27 model is added as a new product line, it is expected that the contribution margin of other product lines at Flint will drop by $7,000 per year.


36. If the F‑27 product line is added next year, the change in net income resulting from this decision would be: a. $30,000 increase; b. $5,000 decrease; c. $23,000 increase; d. $15,000 increase; e. none of these.


37. What is the lowest unit selling price that could be charged for the F‑27 model and still make it economically desirable for Flint to add the new product line? a. $52.25; b. $50.50; c. $55.75; d. $49.00; e. impossible to determine from the information given.


38. Manor Company plans to discontinue a department that has a contribution margin of $24,000 and $48,000 in fixed costs. Of the fixed costs, $21,000 cannot be eliminated. The effect of this discontinuance on Manor's profit would be a(an): a. decrease of $3,000; c. decrease of $24,000; b. increase of $3,000; d. increase of $24,000; e. none of these.


The following data relate to questions 39 and 40.


Elly Industries is a multi‑product company that currently manufactures 30,000 units of Part MR24 each month for use in production. The facilities now being used to produce Part MR24 have a fixed monthly cost of $150,000 and a capacity to produce 84,000 units per month. If Elly were to buy part MR24 from an outside supplier, the facilities would be idle, but its fixed costs would continue at 40 percent of their present amount. The variable production costs of Part MR24 are $11 per unit.


39. If Elly Industries continues to use 30,000 units of Part MR24 each month, it would realize a net benefit by purchasing Part MR24 from an outside supplier only if the supplier's unit price is less than: a. $14.00; b. $11.00; c. $16.00; d. $13.00; e. none of these.


40. If Elly industries is able to obtain Part MR24 from an outside supplier at a unit purchase price of $12.875, the monthly usage at which it will be indifferent between purchasing and making Part MR24 is: a. 30,000 units; b. 32,000 units; c. 80,000 units; d. 48,000 units; e. none of these.


41. At December 31, 19x4, Zar Co. had a machine with an original cost of $84,000, accumulated depreciation of $60,000, and an estimated salvage value of zero. On December 31, 19x4, Zar was considering the purchase of a new machine having a five‑year life, costing $120,000, and having an estimated salvage value of $20,000 at the end of five years. In its decision concerning the possible purchase of the new machine, how much should Zar consider as sunk cost at December 31, 19x4? a. $120,000; b. $100,000; c. $24,000; d. $4,000; e. none of these.


42. The opportunity cost of making a component part in a factory with no excess capacity is the: a. variable manufacturing cost of the component; b. fixed manufacturing cost of the component; c. total manufacturing cost of the component; d. cost of the production given up in order to manufacture the component; e. net benefit foregone from the best alternative use of the capacity required.


43. The opportunity cost of making a component part in a factory with excess capacity for which there is no alternative use is: a. the variable manufacturing cost of the component; b. the total manufacturing cost of the component; c. the total variable cost of the component; d. the fixed manufacturing cost of the component; e. zero.


44. The following standard costs pertain to a component part manufactured by Bor Co.:


Direct materials .. $ 4

Direct labor 10

Factory overhead (60 percent fixed) . 40

Standard cost per unit . $54


The fixed overhead cost will continue unchanged regardless of any 'make or buy' decision. It would cost $49 per unit to buy the part from an outside supplier. In the decision to 'make or buy,' what is the relevant unit cost figure to compare against the outside supplier's price? a. $54; c. $30; b. $38; d. $5; e. none of these.


45. Gata Co. plans to discontinue a department that has a $48,000 contribution margin and $96,000 of fixed costs. Of these fixed costs, $42,000 cannot be eliminated. What would be the effect of this discontinuance on Gata's profit? a. Increase of $48,000; c. Increase of $6,000; b. Decrease of $48,000; d. Decrease of $6,000; e. none of these.


The following data relate to questions 46 through 48.


Condensed monthly operating income data for Cosmo Inc. for November 19x4 is presented below. Additional information regarding Cosmo's operations follows the statement.

Mall Town

Total Store Store

Sales .. $200,000 $80,000 $120,000

Less variable costs 116,000 32,000 84,000

Contribution margin $ 84,000 $48,000 $ 36,000

Less traceable fixed expenses .. 60,000 20,000 40,000

Store segment margin .. $ 24,000 $28,000 $ (4,000)

Less common fixed expenses .. 10,000 4,000 6,000

Operating income $ 14,000 $24,000 $(10,000)


One‑fourth of each store's traceable fixed expenses would continue through December 31, 19x5, if either store were closed. Cosmo allocates common fixed expenses to each store on the basis of sales dollars. Management estimates that closing the Town Store would result in a ten percent decrease in Mall Store sales, while closing the Mall Store would not affect Town Store sales. The operating results for November 19x4 are representative of all months.


46. A decision by Cosmo Inc. to close the Town Store would result in a monthly increase (decrease) in Cosmo's operating income during 19x5 of: a. $4,000; b. $(10,800); c. $(800); d. $(6,000); e. none of these.


47. Cosmo is considering a promotional campaign at the Town Store that would not affect the Mall Store. Increasing annual promotional expenses at the Town Store by $60,000 in order to increase Town Store sales by ten percent would result in a monthly increase (decrease) in Cosmo's operating income during 19x5 of: a. $(16,800); b. $3,400; c. $7,000; d. $(1,400); e. none of these.


48. One‑half of Town Store's dollar sales are from items sold at variable cost to attract customers to the store. Cosmo is considering the deletion of these items, a move that would reduce the Town Store's traceable fixed expenses by 15 percent and result in the loss of 20 percent of the remaining Town Store's sales volume. This change would not affect the Mall Store. A decision by Cosmo to eliminate the items sold at cost would result in a monthly increase (decrease) in Cosmo's operating income during 19x5 of: a. $(6,000); b. $(1,200); c. $2,600; d. $2,400; e. none of these.


49. Peluso Company, a manufacturer of snowmobiles, is operating at 70 percent of plant capacity. Peluso's plant manager is considering making the headlights now being purchased for $11.00 each, a price that is not expected to change in the near future. The Peluso plant has the equipment and labor force required to manufacture the headlights. The design engineer estimates that each headlight requires $4.00 of direct materials and $3.00 of direct labor. Peluso's plant overhead rate is 200 percent of direct labor dollars, and 40 percent of the overhead is fixed cost. A decision by Peluso Company to manufacture the headlights will result in a gain (loss) for each headlight of: a. $(2.00); b. $1.60; c. $.40; d. $2.80; e. none of these.


The following data relate to questions 50 and 51.


Atway Company has met all production requirements for the current month and has an opportunity to produce additional units of product with its excess capacity. Unit selling prices and unit costs for three models of one of its product lines are as follows:

Plain Regular Super

Model Model Model

Selling price $60 $65 $80

Direct material . 18 20 19

Direct labor . 10 15 20

Variable overhead .. 8 12 16

Fixed overhead .. 16 5 15


Variable overhead is applied on the basis of direct labor dollars while fixed overhead is applied on the basis of machine hours. There is sufficient demand for the additional production of any model of the product line.


50. If Atway Company has excess machine capacity and can add more labor as needed (i.e., neither machine capacity nor labor is a constraint), the excess production capacity should be devoted to producing: a. the Plain Model; b. the Regular Model; c. the Super Model; d. an equal number of Regular and Super Models; e. an equal number of Plain and Regular Models.


51. If Atway has excess machine capacity but a limited amount of labor time available, the excess production capacity should be devoted to producing: a. the Plain Model; b. the Regular Model; c. the Super Model; d. an equal number of Regular and Super Models; e. an equal number of each model.


VIII. JOB ORDER COSTING



The following data relate to questions 1 through 3.


Killian Company began operations on January 1, 19x8. The predetermined overhead rate for 19x8 was computed to be $6.00 per direct labor hour. Debits to Work in Process for the year totaled $550,000; credits to Work in Process totaled $480,000, which represents the cost of the goods manufactured for the year. Analysis of the Company's records indicate that direct labor cost totaled $250,000 for the year, which represents 20,000 hours of direct labor time.

1. The direct materials used in production during 19x8 totaled: a. $180,000; b $240,000; c. $130,000; d. $120,000; e. none of these.


2. If the actual manufacturing overhead cost for the year totaled $145,000, then overhead was: a. overapplied by $25,000; b. overapplied by $10,000; c. underapplied by $25,000; d. underapplied by $10,000; e. none of these.


3. The Company's ending work in process inventory consisted of one job, Job 42. The job had been charged with $28,000 of direct labor cost, which consisted of 2,000 actual labor hours. The direct materials cost in Job 42 totaled: a. $33,000; b. $42,000; c. $17,000; d. $30,000; e. none of these.


The following information relates to questions 4 through 6.


The Tse Manufacturing Company closes any balance in the Manufacturing Overhead account to Cost of Goods Sold. For 19x9, manufacturing overhead was overapplied by $4,500. During the year, the company's Finished Goods inventory account was debited for $125,000, and credited for $110,000. The ending balance in the Finished Goods inventory account was $28,000.


4. The balance in the Finished Goods inventory account at the beginning of the year was: a. $28,000; b. $13,000; c. $17,500; d. $8,500; e. none of these.


5. If the estimated manufacturing overhead for 19x9 was $24,000, and the applied overhead was $26,500, the actual manufacturing overhead cost for the year was: a. $19,500; b. $22,000; c. $28,500; d. $31,000; e. none of these.


6. If sales for 19x9 totaled $210,000, the gross margin for Tse Company for the year was: a. $80,500; b. $95,500; c. $89,500; d. $104,500; e. none of these.


The following information for Company C and Company D, for 19X2, relates to questions 7 and 8.


Company C Company D

Estimated direct labor costs $240,000 $84,000

Estimated machine hours 20,000 12,000

Estimated manufacturing overhead $180,000 $42,000


The predetermined overhead rate for Company C is based on direct labor cost and for Company D it is based on machine hours.


7. If the actual overhead for 19x5 was $192,000 in Company C, the predetermined overhead rate would be: a. 110% of direct labor costs; b. 80% of direct labor costs; c. 75% of direct labor costs; d. 125% of direct labor costs; e. none of these.


8. During 19x5, Company D recorded direct labor cost of $77,000 and worked 10,000 machine hours. Overhead was overapplied by $2,000 for the year. Actual overhead cost for the year must have been: a. $33,000; b. $37,000; c. $38,500; d. $35,000; e. none of these.


9. In a job order cost system, the entry to record the application of overhead cost to jobs would include: a. a credit to the Manufacturing Overhead account; b. a credit to the Work in Process inventory account; c. a debit to Cost of Goods Sold; d. a debit to the Manufacturing Overhead account; e. none of these.


10. In a job order cost system, the entry to record depreciation on manufacturing equipment would include: a. a credit to the Work in Process inventory account; b. a debit to the Depreciation Expense account; c. a debit to the Manufacturing Overhead account; d. a credit to the Work in Process inventory account; e. none of these.


11. For 19x1, Feiner Company estimated that it would incur $200,000 in manufacturing overhead costs, $250,000 in direct labor costs, and that it would work 25,000 direct labor hours. The company applies overhead cost to production on the basis of direct labor costs. During 19x1, actual direct labor costs totaled $260,000, actual manufacturing overhead cost totaled $208,000, and the company worked 25,500 direct labor hours. Given this information, overhead for the year would be: a. overapplied by $10,000; b. overapplied by $8,000; c. underapplied by $4,000; d. neither over nor underapplied; e. none of these.


12. During 19x2, Darrow Company worked 10,000 direct labor hours and incurred $80,000 of actual overhead cost. Overhead cost is applied to products on a basis of direct labor hours. The company had estimated that it would work 10,500 direct labor hours during the year and incur $84,000 in direct labor cost. If overhead was underapplied by $2,000 during 19x2, the predetermined overhead rate for the company for the year must have been: a. $7.80; b. $8.00; c. $8.20; d. $8.40; e. none of these.


The following data relates to questions 13 through 15.


Flaherty Company employs a job order costing system. Only two jobs (111, 112) were worked on during June, 19x1. Job 112 was completed during the month and Job 111 remained in process. (The predetermined overhead rate was calculated to be 150% of direct labor costs.) Job cost sheet information is given below:


Job 111 Job 112

May June May June

Costs Costs Costs Costs

Direct Materials $6,000 $1,000 0 $11,000

Direct Labor $3,000 $4,000 0 $ 8,000

Overhead $4,500 ? 0 $12,000


13. The Work in Process inventory: a. totaled $42,000 on June 1, 19x1; b. increased by $11,000 between June 1, 19x1 and June 30, 19x1; c. totaled $31,000 on June 30, 19x1; d. was debited for a total of $31,000 during June; e. none of these.


14. The debit to finished goods for June, 19x1 was for: a. $31,000; b. $24,500; c. $24,000; d. $42,000; e. none of these.


15. The company incurred $17,500 in actual overhead cost during June. Therefore, overhead cost was: a. overapplied by $500; b. underapplied by $1,000; c. underapplied by $5,500; d. overapplied by $5,000; e. none of these.


The following data relate to questions 16 through 18.


Code Company began operations on January 1, 19x3. During 19x3, Work in Process was debited for a total of $195,000. Of this amount, $60,000 was for direct material and $75,000 was for direct labor. Overhead is applied to production on the basis of direct labor cost. During the year, Work in Process was credited for $160,000, which represented the cost of goods transferred to Finished Goods for the year.


16. The predetermined overhead rate in Code Company for 19x3 was: a. 125% of direct labor cost; b. 80% of direct labor cost; c. $1.25 per direct labor hour; d. cannot be determined from the data; e. none of these.


17. If the Work in Process inventory consisted of one job, and if this job has $15,000 of labor cost and $12,000 of applied overhead, then the cost of direct materials in the job would be: a. $52,000; b. $33,000; c. $10,000; d. $8,000; e. none of these.


18. If overhead was underapplied by $4,000 during 19x3, the actual overhead cost incurred during the year must have been: a. $56,000; b. $60,000; c. $64,000; d. $71,000; e. none of these.


19. During 19x4, the Sawyer Manufacturing Company worked 57,000 actual direct labor hours and incurred $345,000 of actual manufacturing overhead cost. The Company had estimated that it would work 55,000 direct labor hours during the year and incur $330,000 of manufacturing overhead cost. If overhead cost is applied to products on the basis of direct labor hours, the company's overhead cost for the year was: a. $15,000 underapplied; b. $15,000 overapplied; c. $3,000 overapplied; d. $3,000 underapplied; e. none of these.


20. On January 1, 19x5, Harper Company's Raw Materials inventory account contained a balance of $20,000. During the year, the company purchased $80,000 in raw materials, and issued $75,000 of materials to production. 80% of the materials issued were direct materials and 20% were indirect. Based on this information: a. Manufacturing Overhead was credited for $15,000 during the year; b. the ending balance in Raw Materials would be $32,000; c. Work in Process was debited for $60,000 during the year; d. Raw Materials was debited for $75,000 during the year; e. none of these.


21. In the Vasquez Company, any over or underapplied overhead is closed out to Cost of Goods Sold. During May 19x6, the company incurred $27,000 in actual manufacturing overhead cost, and applied $29,000 of overhead cost to production. There was no change in the beginning or ending balance of Finished Goods, and the Company's cost of goods manufactured for the year totaled $71,000. Given this information, cost of goods sold for the year must have been: a. $98,000; b. $73,000; c. $71,000; d. $69,000; e. none of these.


The balance in White Company's Work in Process inventory account was $15,000 on August 1, 19x7, and $18,000 on August 31, 19x7. The company incurred $30,000 in direct labor cost during August and requisitioned $25,000 in raw materials (all direct material). If the sum of the debits to the Manufacturing Overhead account total $28,000 for the month, and if the sum of the credits totaled $30,000, then: a. Finished Goods was debited for $82,000 during the month; b. Finished Goods was credited for $83,000 during the month; c. Manufacturing Overhead was underapplied by $2,000 at the end of the month; d. Finished Goods was debited for $85,000 during the month; e. none of these.


23. If manufacturing overhead cost is underapplied, then: a. actual overhead cost is less than estimated overhead cost; b. the amount of overhead cost applied to Work in Process is less than the actual overhead cost incurred; c. the predetermined overhead rate is too high; d. the Manufacturing Overhead account will have a credit balance at the end of the year; e. none of these.


24. In a job order cost system, the entry to record the requisition of indirect materials for use in production would include: a. a debit to the Raw Materials inventory account; b. a debit to the Manufacturing Overhead account; c. a debit to the Work in Process inventory account; d. a credit to the Manufacturing Overhead account; e. none of these.


25. Able Company uses a job order costing system. In reviewing its records at the end of the year, the company has discovered that $2,000 of raw materials has been drawn from the storeroom and used in the production of Job 110, but that no entry has been made in the accounting records for the use of these materials. Job 110 has been completed but it is unsold at year end. This error will cause: a. Work in Process to be understated by $2,000 at year end; b. Cost of Goods manufactured to be overstated by $2,000 for the year; c. Finished Goods to be understated by $2,000 at year end; d. Cost of Goods Sold to be overstated by $2,000 for the year; e. none of these.



The following data relate to questions 26 through 29.


The direct labor rate in Brent Company is $9.00 per hour, and overhead cost is applied to products at a rate of $6.00 per direct labor hour. During May 19x8, the company purchased $60,000 in raw materials (all direct materials) and worked 3,200 direct labor hours. The Raw Materials inventory (all direct materials) decreased by $3,000 between the beginning and end of May. The Work in Process inventory on May 1 consisted of one job which had been charged with $4,000 in direct materials and on which 300 hours or direct labor time had been worked. There was no Work in Process inventory on May 31.


26. The balance in the Work in Process inventory account on May 1, 19x8 was: a. $0; b. $6,700; c. $4,500; d. $8,500; e. none of these.


27. The debit to Work in Process for the cost of direct materials used during May was: a. $63,000; b. $61,000; c. $57,000; d. $67,000; e. none of these.


28. The debit to Work in Process for direct labor cost during May was: a. $21,000; b. $26,100; c. $28,800; d. $31,500; e. none of these.


29. If overhead was underapplied by $2,500 during May, the actual overhead cost for the month must have been: a. $16,700; b. $21,700; c. $18,500; d. $23,500; e. none of these.


30. In computing its predetermined overhead rate, Marple Company inadvertently left its indirect labor costs out of the computation. This oversight will cause: a. manufacturing overhead to be overapplied; b. the cost of goods manufactured to be understated; c. the debits to the Manufacturing Overhead account to be understated; d. the ending balance in Work in Process to be overstated; e. none of these.


31. In a job order cost system, the amount of overhead cost that has been applied to a job that remains incomplete at the end of a period: a. is deducted on the income statement as overapplied overhead; b. is closed to cost of goods sold; c. is transferred to finished goods at the end of the period; d. is part of the ending balance of the Work in Process account; e. none of these.


32. In a job order cost system, the cost of a completed but unsold job is: a. closed to cost of goods sold; b. part of the Work in Process inventory balance; c. adjusted to exclude any applied overhead; d. part of the Finished Goods inventory balance; e. none of these.



The following data relate to questions 33 through 36.


Meyers Company had the following inventory balances at the beginning and end of November, 19x9:


11/1/x9 11/30/x9

Raw Materials (all direct) $17,000 $20,000

Finished Goods $50,000 $44,000

Work in Process $ 9,000 $11,000


During November, $39,000 in raw materials (all direct materials) was drawn from inventory and used in production. The company's predetermined overhead rate for 19x9 was $8 per DLH, and it paid its direct labor workers $10 per hour. Some 300 hours of direct labor time had been expended on the jobs in the beginning Work in Process inventory account. The ending Work in Process inventory account contained $4,700 of direct materials cost. The Company incurred $28,000 of actual manufacturing overhead cost during the month and applied $26,400 in manufacturing overhead cost.


33. The raw materials purchased during November 19x9 totaled: a. $42,000; b. $45,000; c. $36,000; d. $39,000; e. none of these.


34. The direct materials cost in the November 1 Work in Process inventory account totaled: a. $6,600; b. $6,000; c. $3,600; d. $3,000; e. none of these.


35. The actual direct labor hours worked during November totaled: a. $2,800 hours; b. 3,300 hours; c. 3,500 hours; d. 3,600 hours; e. none of these.


36. The amount of direct labor cost in the November 30 Work in Process inventory was: a. $2,800; b. $3,300; c. $3,500; d. $6,300; e. none of these.


37. Given the following information about Fisher Company's Work in Process inventory account for the month of May, 19x1:


May 1 balance $ 26,000

Debits During May:

Direct Materials $ 40,000

Direct Labor $ 50,000

Manufacturing Overhead $ 37,500


During the month, Fisher Company's Work in Process inventory account was credited for $120,500, which represented the cost of goods manufactured for the month. The company applies overhead to production on the basis of direct labor cost. Only one job remained in process on May 31; this job had been charged with $9,600 of applied overhead cost. The amount of direct materials cost in the unfinished job would be: a. $10,600; b. $16,700; c. $12,800; d. $23,400; e. none of these.


38. For 19x1, Sharp Company's records show that overhead was overapplied by $10,000. This overapplied overhead was closed out to the Cost of Goods Sold account at the end of the year. In trying to determine why overhead was overapplied by such a large amount, the company has discovered that $6,000 of depreciation on factory equipment was charged to administrative expense in error. Given the above information, which of the following statements is true? a. Manufacturing overhead was actually overapplied by $16,000 for the year; b. The company's net income is understated by $6,000 for the year; c. Under the circumstances posed above, the error in recording depreciation would have no effect on net income for the year; d. The $6,000 in depreciation should have been charged to Work in Process rather than to administrative expense; e. None of the above statements is true.


The following information applies to Collins Company for the current year:


Direct labor‑hours:

Estimated for the year 24,000

Actual hours worked 19,500

Direct labor cost:

Estimated for the year $300,000

Actual cost incurred 210,000

Manufacturing overhead:

Estimated for the year $240,000

Actual cost incurred 185,000


39. If overhead cost is applied to production on a basis of direct labor cost, the under or overapplied overhead cost for the current year will be: a. $10,000 overapplied; b. $17,000 underapplied; c. $15,000 underapplied; d. $10,000 underapplied; e. none of these.


40. For 19x8, Dowan Company incurred $156,600 in actual manufacturing overhead cost. The Manufacturing Overhead account showed that overhead was underapplied by $12,600 for the year. If the predetermined overhead rate is $6.00 per direct labor hour, how many hours did the company work during the year? a. 26,000 hours; b. 24,000 hours; c. 28,200 hours; d. 25,000 hours; e. none of these.


41. In reviewing the accounting records at year end, Garff Company's accountant has determined that the following items and amounts were debited to the Manufacturing Overhead account during the year:


Factory supervisor's salary $8,000

Sales commissions 7,000

Vacation pay for the materials storeroom clerk 2,000


Including the items listed above, the debits to the Manufacturing Overhead account totaled $245,000 for the year. Credits to the account totaled $240,000 for the year. Based on this information, if all entries had been made correctly during the year the Manufacturing Overhead account would have been: a. overapplied by $4,000; b. overapplied by $12,000; c. underapplied by $5,000; d. overapplied by $2,000; e. none of these.


The following data relates to questions 42 through 45.


Dillon Company has only Job 211 in process on May 1, 19x2. The job had been charged with $2,000 of direct material, $4,000 of direct labor, and $3,000 of manufacturing overhead. The company applies overhead cost to jobs at a predetermined rate of 75% of direct labor cost. Any under or overapplied overhead cost is closed out to Cost of Goods Sold at the end of the month. During May, the following activity and amounts were recorded by the company:


Raw materials (all direct materials):

Purchased during the month $38,000

Used in production 35,000

Labor:

Direct labor hours worked during the month 3,150

Direct labor cost incurred $30,000

Indirect labor cost incurred 6,000


Manufacturing overhead cost incurred (total) $24,500

Inventories:

Raw materials (all direct), May 31 $ 8,000

Work in process, May 31 12,000*


*Contains $4,400 in direct labor cost.


42. The balance on May 1 in the Raw Materials inventory was: a. $11,000; b. $5,000; c. $7,000; d. $9,000; e. none of these.


43. The amount of direct materials cost in the May 31 Work in Process inventory account was: a. $7,600; b. $2,000; c. $6,300; d. $4,300; e. none of these.


44. The entry to dispose of the under or overapplied overhead cost for the month would include: a. a debit of $2,000 to the Manufacturing Overhead account; b. a credit of $2,500 to the Manufacturing Overhead account; c. a debit of $2,000 to Cost of Goods Sold; d. a credit of $2,500 to Cost of Goods Sold; e. none of these.


45. The Cost of Goods Manufactured for May 19x2 was: a. $84,500; b. $95,000; c. $75,500; d. $81,500; e. none of these.


46. Luna Co.'s 19x5 manufacturing costs were as follows:


Direct materials and direct labor $500,000

Depreciation of manufacturing equipment 70,000

Depreciation of factory building 40,000

Janitor's wages for cleaning factory premises 15,000


How much of these costs should be inventoried for external reporting purposes? a. $625,000; b. $610,000; c. $585,000; d. $500,000; e. none of these.


47. Lucas Co. has a job order cost system. For the month of April 19x6, the following debits (credits) appeared in the general ledger account, Work in Process:


April

1 Balance $ 24,000

30 Direct materials 80,000

30 Direct labor 60,000

30 Factory overhead 54,000

30 To finished goods (200,000)


Lucas applies overhead to production at a predetermined rate of 90% based on direct labor cost. Job No. 100, the only job still in process at the end of April, has been charged with factory overhead of $4,500. The amount of direct materials charged to Job No. 100 was: a. $18,000; b. $8,500; c. $5,000; d. $4,500; e. none of these.


48. In a job order cost system, the application of factory overhead would usually be reflected in the general ledger as an increase in: a. raw materials inventory; b. finished goods inventory; c. work in process inventory; d. cost of goods sold; e. none of these.


The following data relate to questions 49 through 51.


Farber company employs a job‑order cost system. The information below is from the financial records of the company for 19x5: Total manufacturing costs were $2,500,000; Cost of goods manufactured was $2,425,000; Applied factory overhead was 30 percent of total manufacturing costs; Factory overhead was applied to production at a rate of 80 percent of direct labor cost; Work in process inventory at January 1, 19x5, was 75 percent of work in process inventory at December 31, 19x5.


49. Farber Company's total direct labor cost in 19x5 is: a. $750,000; b. $600,000; c. $900,000; d. $937,500; e. none of these.


50. Total cost of direct material used by Farber Company in 19x5 is: a. $750,000; b. $812,500; c. $850,000; d. $1,150,000; e. none of these.


51. The carrying value of Farber Company's work in process inventory at December 31, 19x5, is: a. $300,000; b. $225,000; c. $100,000; d. $75,000; e. none of these.


52. Heller Cannery, Inc., estimated that it would incur $510,000 in manufacturing overhead during 19x1 and that it would work 100,000 machine hours. The company actually worked 105,000 machine hours and incurred $540,000 in actual manufacturing overhead cost. By how much was manufacturing overhead under or overapplied for the year? a. $30,000 underapplied; b. $4,500 underapplied; c. $25,500 overapplied; d. $30,000 overapplied; e. none of these.


53. In a job order cost system, direct labor costs usually are recorded initially as an increase in: a. manufacturing overhead; b. cost of goods sold; c. finished goods; d. work in process; e. none of these.


54. In a job order cost system, the use of indirect materials would usually be reflected in the general ledger as an increase in: a. raw materials; b. work in process; c. manufacturing overhead; d. finished goods; e. none of these.


55. In a job order cost system, the incurrence of indirect labor costs would usually be included in the general ledger as a charge to: a. manufacturing overhead; b. salaries and wages expense; c. work in process; d. salaries and wages payable; e. none of these.


56. Avery Co. uses a predetermined overhead rate based on direct labor hours. For the month of October, Avery's estimated manufacturing overhead cost was $300,000 based on an estimated activity level of 100,000 direct labor hours. Actual overhead amounted to $325,000 with actual direct labor hours totaling 110,000 for the month. How much was the overapplied or underapplied overhead? a. $30,000 overapplied; b. $30,000 underapplied; c. $5,000 overapplied; d. $5,000 underapplied; e. none of these.


57. Marc Corp. has a job order cost system. The following debits (credits) appeared in the work in process account for the month of May 19x6:


May Description Amount

1 Balance $ 10,000

31 Direct materials 60,000

31 Direct labor 40,000

31 Factory overhead 32,000

31 To finished goods (120,000)


Marc applies overhead to production at a predetermined rate of 80% based on direct labor cost. Job No. 23, the only job still in process at the end of May 19x6, has been charged with direct labor of $5,000. The amount of direct materials charged to Job No. 23 was: a. $6,250; b. $7,500; c. $13,000; d. $17,000; e. none of these.


58. Woodman Company applies factory overhead on the basis of direct labor hours. Estimated and actual data for direct labor and overhead for 19x3 are as follows:

Estimated Actual

Direct labor hours 600,000 550,000

Factory overhead costs $720,000 $680,000


The factory overhead for Woodman Company in 19x3 is: a. overapplied by $20,000; b. overapplied by $40,000; c. underapplied by $20,000; d. underapplied by $40,000; e. neither underapplied nor overapplied.



IX. PROCESS COSTING


The following data relate to questions 1 through 4.


1. The activity in Nolan Company's blending department for the month of April is given below:

Number Labor and Overhead

of Units Percent Completed

Work in process inventory, April 1 8,000 50 percent

Started into process during the month 50,000

Work in process inventory, April 30 10,000 70 percent


All materials are added at the beginning of processing in the blending department. The equivalent units of production for material for the month, using the FIFO method, are: a. 50,000 units; b. 58,000 units; c. 54,000 units; d. 60,000 units; e. none of these.


2. The equivalent units of production for labor and overhead for the month, using the FIFO method, are: a. 47,000 units; b. 51,000 units; c. 5,000 units; d. 54,000 units; e. none of these.


3. The equivalent units of production for material for the month, using the weighted‑average method, are: a. 48,000 units; b. 50,000 units; c. 58,000 units; d. 52,000 units; e. none of these.


4. The equivalent units of production for labor and overhead for the month, using the weighted‑average method, are: a. 50,000 units; b. 51,000 units; c. 47,000 units; d. 55,000 units; e. none of these.


5. Mark Company completed 18,000 equivalent units as to conversion costs for March 19x8. The company uses the FIFO method. The Company's beginning inventory consisted of 5,000 units, 60 percent complete as to conversion costs. 20,000 units were started into production during the month, and 15,000 units were transferred to finished goods. Based on this information, the Company's ending inventory consisted of: a. 10,000 units, 60 percent complete as to conversion costs; b. 5,000 units, 10 percent complete as to conversion costs; c. 10,000 units, 50 percent complete as to conversion costs; d. 5,000 units, 60 percent complete as to conversion costs; e. none of these.


6. Black Company's Work in Process inventory consists of 5,000 units, 80 percent complete as to materials and 50 percent complete as to labor and overhead. If the total dollar value of the inventory is $60,000 and the equivalent unit cost for labor and overhead is $8.00, the equivalent unit cost of materials must be: a. $5.00; b. $10.00; c. $8.00; d. $4.00; e. none of these.


7. The Gasson Company uses a process cost system. The ending Work in Process inventory consists of 10,000 units, 100 percent complete as to materials and 70 percent complete as to labor and overhead. If the production costs per equivalent unit for the period are $4.50 for the materials and $2.00 for labor and overhead, the balance of the ending Work in Process inventory account would be: a. $44,500; b. $50,500; c. $59,000; d. $65,000; e. none of these.

The following data relate to questions 8 through 12.


The information below was obtained from the records of Bapst Manufacturing Company for the month of May. The company has a process costing system in operation and uses the FIFO method to account for cost flows.


Number Labor and Overhead

of Units percent Completed

Work in process Inventory, May 1 3,000 30 percent

Started into production during the month 20,000

Work in process inventory, May 31 4,000 40 percent


Costs for the month:

Labor &

Materials Overhead Total

Work in process inventory, May 1 $ 27,000 $ 8,000 $ 35,000

Added to production during the month $137,000 $200,000 $337,000


All materials are added at the beginning of the manufacturing process.


8. The equivalent units of production for material for the month are: a. 21,600 units; b. 22,500 units; c. 16,000 units; d. 20,000 units; e. none of these.


9. The equivalent units of production for labor and overhead for the month are: a. 17,600 units; b. 18,500 units; c. 19,700 units; d. 21,600 units; e. none of these.


10. Do not use the equivalent units calculated above for material. If you assume that the company had 20,000 equivalent units for material, the cost per equivalent unit for material, under FIFO, is: a. $1.35; b. $6.85; c. $8.20; d. $1.75; e. none of these.


11. Do not use the equivalent units figure calculated above for labor and overhead. If you assume that the Company had 20,000 equivalent units for labor and overhead, the cost per equivalent unit for labor and overhead, under FIFO, is: a. $10.40; b. $16.85; c. $10.00; d. $.40; e. none of these.


12. Do not use the cost per equivalent units figures calculated above. If you assume that the cost per equivalent unit for material is $6.00, and that the cost per equivalent unit for labor and overhead is $8.00, then the ending work in process inventory for Bapst Manufacturing Company is: a. $266,000; b. $24,400; c. $29,400; d. $36,800; e. none of these.


13. The Morgan Company has a process cost system in operation and uses the weighted‑average method to account for cost flows. The company completed 54,000 equivalent units as to conversion costs for March 19x1. There were 7,500 units in the beginning inventory, two thirds complete as to conversion costs. During March, 52,500 units were started and 50,000 were completed and transferred out. Based on this information, the ending work in process inventory: a. consisted of 5,000 units; b. consisted of 2,500 units; c. was 65 percent complete as to conversion costs; d. was 40 percent complete as to conversion costs; e. none of these.


14. The Nichols Company has a process cost system in operation and uses the FIFO method to account for cost flows. The company completed 25,500 equivalent units for labor and overhead for November 19x2. There were 6,000 units in the ending inventory on November 30, 19x2, 75 percent complete as to labor and overhead. If the November 1, 19x2, inventory consisted of 8,000 units, 50 percent complete and 25,000 units were completed and transferred out, the number of units started during November was: a. 24,500 units; b. 23,000 units; c. 27,000 units; d. 21,000 units; e. none of these.

The following data relate to questions 15 and 16.


Higgins Labs, Inc. uses a process costing system. The following data are available for one department for October, 19x3:


Percent Complete

Units Materials Conversion

Work in process (Beginning) 10,000 60 percent 30 percent

Work in process (Ending) 5,000 80 percent 70 percent


The department started 45,000 units into production during the month and completed and transferred 50,000 units to the next department.


15. Assuming a weighted‑average method of accounting for units and costs, the equivalent units for material for October, 19x3, would be: a. 54,000 units; b. 50,000 units; c. 48,000 units; d. 44,000 units; e. none of these.


16. Assuming a FIFO method of accounting for units and costs, the equivalent units for conversion costs for October, 19x3, would be: a. 50,500 units; b. 46,500 units; c. 44,500 units; d. 48,000 units; e. none of these.


17. The following information pertains to the month of July for Steven Company:


Inventory method: FIFO process costing

Beginning work in process inventory: $17,500 (10,000 units; 100 percent complete for materials, and 60 percent complete for labor and overhead)

Units completed: 60,000 units

Cost per equivalent unit: Material, $2.50; Labor and Overhead, $2.00


Given the data above, the cost of units transferred out during the month is: a. $260,000; b. $242,500; c. $254,500; d. $250,500; e. none of these.


18. The following information pertains to the month of August for the Steven Company.


Inventory method: FIFO process costing

Beginning Work in Process inventory: 15,000 units ($52,000); 100 percent complete as to materials; 40 percent complete as to labor and overhead

Units started: 70,000

Units completed: 80,000

Ending Work in Process inventory (100 percent complete as to materials; 80 percent complete as to labor and overhead)

Equivalent unit cost: Materials $3.00; Labor and Overhead $1.25


Given the data above, the cost of the units in the ending Work in Process inventory is: a. $17,000; b. $20,000; c. $10,000; d. $22,500; e. none of these.


19. Trapp Corporation has a process costing system in operation and uses the weighted‑average method. The following information relates to the month of September:


Beginning work in process inventory: $10,000 (3,000 units; 70 percent complete for materials, and 60 percent complete for labor and overhead)

Units completed: 9,000

Cost per equivalent unit: Material, $2.00; Labor and overhead, $3.00.


Given the data above, the cost of units transferred out during the month is: a. $39,600; b. $45,000; c. $45,400; d. $35,400; e. none of these.


The following data relate to questions 20 and 21.


The Pert Company has a process cost system and uses the weighted‑average method. The following information is applicable for the month of September:


Beginning work in process inventory: 5,000 units (100 percent complete as to materials, totaling $10,000; 30 percent complete as to labor and overhead, totaling $4,500)

Units started in September: 25,000

Units completed in September: 22,000

Ending work in process inventory: 8,000 units (100 percent complete as to materials; 50 percent complete as to labor and overhead)

Costs added during September: Materials $47,000; Labor and Overhead $73,500


20. Based on this information the cost of the units completed is: a. $135,000; b. $113,600; c. $107,800; d. $120,250; e. none of these.


21. The cost of the ending Work in Process inventory is: a. $27,200; b. $39,200; c. $21,400; d. $32,400; e. none of these.


The following data relate to questions 22 through 26.


L.A. Company is a manufacturer of small widgets. The company has a process costing system in operation and uses the weighted‑average method to account for cost flows. The following information for the assembly department was obtained from the accounting records for September (all materials are added at the beginning of the process):

Number Labor and Overhead

of Units percent Complete

Work in process inventory, Sept. 1 40,000 25 percent

Transferred in during the month 100,000

Work in process inventory, Sept. 30 20,000 50 percent


September costs:

Transferred Labor &

In Materials Overhead

Beginning work in process inventory $ 80,000 $ 37,600 $ 5,000

Cost added during the month $251,000 $122,400 $66,500


The 'Total cost to be accounted for' section of the production report for the month will show an amount equal to: a. $562,500; b. $231,500; c. $439,900; d. $520,000; e. none of these.


23. The equivalent units for material for the month is: a. 100,000 units; b. 120,000 units; c. 140,000 units; d. 160,000 units; e. none of these.


24. The unit cost for labor and overhead for the month is: a. $.50; b. $.125; c. $.52; d. $.55; e. none of these.


25. Do not calculate the cost per equivalent unit. Assume the following cost per equivalent unit figures: transferred in, $2.00; material, $1.50; labor and overhead, $.60. Given these figures, the cost of the ending work in process inventory is: a. $82,000; b. $76,000; c. $36,000; d. $70,000; e. none of these.


26. Using the same unit cost data as in the previous question, the cost of the units completed and transferred out during the month is: a. $410,000; b. $252,000; c. $492,000; d. $486,500; e. none of these.


The following data relate to questions 27 through 31.


The information below was obtained from the records of the Cushing Company for the month of August, 19x3. The company uses the FIFO method of accounting for cost flows.

Number Labor and Overhead

of Units Percentage Completed

Units started 85,000

Units completed and transferred 75,000

Work in process ‑ August 1 10,000 25 percent

Work in process ‑ August 31 20,000 50 percent


Labor

Costs for August: Materials and Overhead

Beginning inventory $ 16,000 $ 7,500

Costs added during the month $144,000 $247,500


All materials are added at the beginning of the process.


27. The equivalent units of production for materials for the month of August are: a. 75,000 units; b. 85,000 units; c. 87,500 units; d. 95,000 units; e. none of these.


28. The equivalent units of production for labor and overhead for the month of August are: a. 85,000 units; b. 95,000 units; c. 87,500 units; d. 82,500 units; e. none of these.


29. Do not use the equivalent units figure calculated above for materials. If you assume 80,000 equivalent units for materials, then the cost per equivalent unit for materials is: a. $1.78; b. $1.80; c. $1.60; d. $2.00; e. none of these.


30. Assume that the equivalent unit cost for materials is $2.00 and that the equivalent unit cost for labor and overhead is $3.00. Based on this information, the total costs assigned to the units transferred out would be: a. $347,500; b. $358,000; c. $371,000; d. $375,000; e. none of these.


31. Assuming the same equivalent unit costs as in question 45 above ($2.00 for materials and $3.00 for labor and overhead), the value of the Work in Process ending inventory would be: a. $70,000; b. $100,000; c. $50,000; d. $55,000; e. none of these.


32. Banden Company has a process cost system using the FIFO cost flow method. All materials are introduced at the beginning of the process in Department One. The following information is available for the month of May, 19x4:

Units

Work in process 5/1/x4 (60 percent complete as to conversion costs) 1,500

Started in May 7,500

Transferred to Department Two during May 8,000

Work in process 5/31/x4 (30 percent complete as to conversion costs) 1,000


What are the equivalent units of production for the month of May, 19x4?


Materials Conversion Cost

a. 9,000 units 8,300 units

b. 9,000 units 7,400 units

c. 7,500 units 7,400 units

d. 7,500 units 7,700 units


33. Department A is the first stage of Laurie Company's production cycle. The following information is available for conversion costs for the month of May, 19x6:

Units

Work in process, beginning (25 percent complete) 8,000

Started in May 40,000

Completed in May and transferred to Department B 38,000

Work in process, ending (60 percent complete) 10,000


Using the FIFO method, the equivalent units for conversion costs for the month are: a. 42,000 units; b. 38,000 units; c. 44,000 units; d. 36,000 units; e. none of these.


34. Barnett Company adds materials at the beginning of the process in Department M. Conversion costs were 75 percent complete as to the 4,000 units in work in process at May 1, 19x8, and 50 percent complete as to the 6,000 units in work in process at May 31. During May, 12,000 units were completed and transferred to the next department. An analysis of the costs relating to work in process at May 1 and to production activity for May is as follows:


Costs

Materials Conversion

Work in process 5/1 $13,800 $ 3,740

Costs added during May 42,000 26,260


Using the weighted‑average method, the total cost per equivalent unit for May was: a. $5.02; b. $5.10; c. $5.12; d. $5.25; e. none of these.


35. On November 1, 19x8, Riser Company had 5,000 units of work in process in Department No. 1 which were 100 percent complete as to material costs and 20 percent complete as to conversion costs. During November, 32,000 units were started in Department No. 1 and 34,000 units were completed and transferred to Department No. 2. The work in process on November 30, 19x8, was 100 percent complete as to material costs and 40 percent complete as to conversion costs. By what amount would the equivalent units for conversion costs for the month of November differ if the first‑in, first‑out method were used instead of the weighted‑average method? a. 1,000 decrease; b. 3,000 decrease; c. 1,500 decrease; d. 2,200 decrease; e. none of these.


36. Carrington Company produces canned vegetable soup and is budgeting sales of 300,000 units for the month of January, 19x9. Actual inventory units at January 1 and budgeted inventory units at January 31 are as follows:


Units

Actual inventory at January 1:

Work in process None

Finished goods 75,000

Budgeted inventory at January 31:

Work in process (75 percent processed) 24,000

Finished goods 60,000


How many equivalent units of production is Carrington budgeting for January, 19x9? a. 285,000 units; b. 303,000 units; c. 300,000 units; d. 309,000 units; e. none of these.


37. Colby Company adds materials at the beginning of the process in the Molding Department, which is the first of two stages of its production cycle. Information concerning the materials used in the Molding Department during March, 19x9 is as follows:

Materials

Units Costs

Work in process at March 1, 19x9 22,000 $11,000

Units started during March 90,000 $46,120

Completed and transferred to next department during March 97,000


Using the weighted‑average method, what was the materials cost of the work in process inventory at March 31, 19x9? a. $11,220; b. $7,500; c. $5,100; d. $7,650; e. none of these.


38. Richardson Company computed the flow of physical units completed for department M for the month of June, 19x1 as follows:


Units completed:

From work in process on June 1, 19x1 5,000

Started and completed during June 35,000

Total completed units 40,000


Materials are added at the beginning of the process. 10,000 units were in work in process at June 30, 19x1; these units were 80 percent complete as to conversion costs. The work in process at June 1, 19x1, was 40 percent complete as to conversion costs. Using the FIFO method, the equivalent units for June conversion costs were a. 39,000 units. b. 45,000 units; c. 46,000 units; d. 48,000 units; e. none of these.


39. The ending Work in Process inventory consists of 10,000 units, 60 percent complete as to materials. If the total dollar value of this inventory is $38,000, the equivalent unit cost for materials is $5.00, and the equivalent unit cost of labor and overhead is $4.00, the inventory with respect to labor and overhead is: a. 10 percent complete; b. 20 percent complete; c. 38 percent complete; d. 30 percent complete; e. none of these.


40. The beginning Work in Process inventory consisted of 6,000 units, two‑ thirds complete as to conversion costs. During the month, 42,000 units were started and 40,000 units were completed and transferred. If the company had 40,000 equivalent units for conversion costs (computed by the FIFO method) for the month, the ending inventory consists of: a. 8,000 units, 25 percent complete; b. 0 units; c. 8,000 units, 50 percent complete; d. 4,000 units, 100 percent complete; e. none of these.


41. Equivalent units for a process costing system using the weighted average method would be equal to: a. units completed during the period and transferred out; b. units completed during the period plus equivalent units in the ending work in process inventory; c. units needed to complete the beginning inventory; d. units completed during the month less the equivalent units in the beginning work in process inventory; e. none of these.


42. The equivalent units for conversion costs equaled 47,500 units. The beginning inventory consisted of 15,000 units, 60 percent complete. The ending inventory consisted of 10,000 units, 75 percent complete. Assuming a FIFO basis of computing equivalent units, the number of units started during the month was: a. 41,500 units; b. 31,000 units; c. 25,000 units; d. 34,000 units; e. none of these.


43. The equivalent units for conversion costs equaled 47,500 units. The beginning inventory consisted of 15,000 units, 60 percent complete. The ending inventory consisted of 10,000 units, 75 percent complete. Assuming a weighted‑ average basis of computing equivalent units, the number of units started during the month was: a. 25,000 units; b. 34,000 units; c. 35,000 units; d. 40,000 units; e. none of these.


44. The beginning inventory consists of 80,000 units, 100 percent complete as to materials and 25 percent complete as to conversion costs. The total dollar value of this inventory is $226,000. During the month, 150,000 units are transferred out and the equivalent unit cost for the month for materials is $2.00 and for conversion costs is $3.50. ]f the weighted‑average method was used in computing equivalent costs, the value of the units completed and transferred is: a. $681,000; b. $765,000; c. $821,000; d. $825,000; e. none of these.


45. The beginning inventory consisted of 15,000 units, 100 percent complete as to materials and 40 percent complete as to conversion costs. The total dollar value of this inventory was $31,000. 40,000 units were transferred out during the month. The equivalent unit cost was computed to be $1.30 for materials and $2.20 for conversion costs under the FIFO method. Given this information, the value of the units completed and transferred is: a. $140,000; b. $138,300; c. $131,700; d. $118,500; e. none of these.


46. Assuming that there was no beginning work in process inventory, and the ending work in process inventory is 50 percent complete as to conversion costs, the number of equivalent units as to conversion costs would be: a. the same as the units placed in process; b. the same as the units completed; c. less than the units placed in process; d. less than the units completed; e. none of these.


47. The following information pertains to Top Co.'s Division D for the month of May:

Number Cost of

of units materials

Beginning work in process 30,000 $11,000

Started in May 80,000 36,000

Units completed 85,000

Ending work in process 25,000


All materials are added at the beginning of the process. Using the weighted‑ average method, the cost per equivalent unit for materials is: a. $0.43; b. $0.45; c. $0.55; d. $0.59; e. none of these.


48. Glo Co., a manufacturer of combs, planned sales of 125,000 units for the month of April 19x7. The following additional information is provided:


Number

of units

Actual inventory at April 1:

Work in process None

Finished goods 37,500

Planned inventory at April 30:

Work in process (75 percent completed) 8,000

Finished goods 30,000


How many equivalent units of production did Glo plan for April 19x7? a. 126,500; b. 125,500; c. 123,500; d. 117,500; e. none of these.


49. Bart Co. adds materials at the beginning of the process in Department M. The following information pertains to Department M's work in process during April:

Units

Work in process, April 1 (60 percent complete as to conversion cost) 3,000

Started in April 25,000

Completed 20,000

Work in process, April 30 (75 percent complete as to conversion cost) 8,000


Under the weighted average method, the equivalent units for conversion cost are: a. 26,000; b. 25,000; c. 24,200; d. 21,800; e. none of these.


50. In the computation of manufacturing cost per equivalent unit, the weighted‑ average method of process costing considers: a. current costs only; b. current costs plus cost of ending work in process inventory; c. current costs plus cost of beginning work in process inventory; d. current costs less cost of beginning work in process inventory; e. none of these.


51. Kew Co. had 3,000 units in work in process at April 1, 19x6, which were 60 percent complete as to conversion cost. During April, 10,000 units were completed. At April 30, 4,000 units remained in work in process which were 40 percent complete as to conversion cost. Direct materials are added at the beginning of the process. How many units were started during April? a. 9,000; b. 9,800; c. 10,000; d. 11,000; e. none of these.


52. In developing a predetermined overhead rate for use in a process costing system, which of the following could be used as the base in computing the rate? a. Actual factory overhead; b. Estimated factory overhead; c. Actual direct labor hours; d. Estimated direct labor hours; e. Any of the above could be used as the base.


53. During March 19x4 Bly Co.'s Department Y equivalent unit product costs, computed under the weighted‑average method, were as follows:


Materials $1

Conversion 3

Transferred‑in 5


Materials are introduced at the end of the process in Department Y. There were 4,000 units (40 percent complete as to conversion cost) in work in process at March 31, 19x4. The total costs assigned to the March 31, 19x4, work in process inventory should be: a. $36,000; b. $28,800; c. $27,200; d. $24,800; e. none of these.


54. Assuming that there was no beginning work in process inventory, and the ending work in process inventory is 100 percent complete as to material costs, the number of equivalent units as to material costs would be: a. the same as the units placed in process; b. the same as the units completed; c. less than the units placed in process; d. less than the units completed; e. none of these.


55. Roy Company manufactures product X in Departments A and B. Materials are added at the beginning of the process in Department B. Roy uses the weighted‑ average method. Conversion costs for Department B were 50 percent complete as to the 6,000 units in the beginning work in process and 75 percent complete as to the 8,000 units in the ending work in process. 12,000 units were completed and transferred out of Department B during February 19x1. An analysis of the costs relating to work in process (WIP) and production activity in Department B for February 19x1 is as follows:


Transferred In Materials Conversion

WIP, February 1:

Costs attached $12,000 $2,500 $1,000

February activity:

Costs added 29,000 5,500 5,000


The total cost per equivalent unit transferred out for February 19x1 of product X, rounded to the nearest penny, was: a. $2.75; b. $2.78; c. $2.82; d. $2.85; e. none of these.



X. BUDGETING



1. Which of the following is not included as one of the four major sections of a cash budget? a. The receipts section; b. The disbursements section; c. the financing section; d. The selling and administrative expense section; e. The cash excess or deficiency section.


2. The Mahar Company uses a 'Just In Time' (JIT) inventory system in maintaining its direct materials inventory of Tebs. Under its JIT system, the company keeps a zero inventory level of Tebs. Five Tebs are needed to complete one unit of finished product. The budgeted production of finished product is 2,450 units for the month of February. Given these data, the budgeted purchases of Tebs in February is: a. 490; b. 2,450; c. 12,250; d. 15,175; e. impossible to determine from the information given.


3. Which of the following is not true about a self‑imposed budget? a. It contains its own unique system of control in that if people can't meet budget specifications, they have only themselves to blame; b. An individual is more apt to work at fulfilling a budget he has set himself; c. The person in direct contact with an activity is in the best position to make budget estimates; d. The views of persons below middle management on budget matters are not considered by top management; e. It is also called a participative budget.


The following data relate to questions 4 through 6.


Barley Enterprises has budgeted sales in units for the next four months as follows:


October . 4,800 units

November 5,800 units

December 6,400 units

January . 5,200 units


Past experience has shown that the ending inventory for each month must be equal to 15 percent of the next month's sales in units. The inventory on September 30 contained 600 units. The company needs to prepare a Production Budget for the last quarter of the year.


4. The total units to be produced in October is: a. 4,530; b. 5,070; c. 5,670; d. 5,890; e. none of these.


5. The total units needed in November is: a. 5,800; b. 7,180; c. 7,520; d. 6,760; e. none of these.


6. The desired ending inventory for December is: a. 960; b. 870; c. 780; d. 690; e. impossible to determine from the information given.



The following data relate to questions 7 through 9.


Marty's Merchandise has budgeted sales as follows for the second quarter of the year:


April . $30,000

May $60,000

June .. $50,000


Cost of Goods Sold in the company is equal to 70 percent of sales. The company wants to maintain a monthly ending inventory equal to 120 percent of the Cost of Goods Sold for the following month. The inventory on March 31 is $22,000. The company is now preparing a Merchandise Purchases Budget for April, May, and June.


7. The total merchandise needs for the month of April is: a. $63,500; b. $102,000; c. $71,400; d. $41,000; e. none of these.


8. The desired beginning inventory for June is: a. $42,000; b. $35,000; c. $50,000; d. $38,000; e. impossible to determine from the information given.


9. The budgeted purchases for May are: a. $49,400; b. $50,400; c. $60,000; d. $33,600; e. none of these.


10. Which of the following represents the proper sequence in which the budgets below are prepared? a. Production, Cash, Direct Materials; b. Direct Materials, Cash, Production; c. Cash, Direct Materials, Production; d. Direct Materials, Production, Cash; e. none of these.


11. Which of the following factors is not considered in making a sales forecast? a. Market share; b. General economic conditions; c. Advertising and product promotion; d. Availability of clerical staff for credit and collections; e. Unfilled order backlogs.


12. The Segal Company makes and sells a single product called a Wib. The budgeted sales for November are $165,000. The gross margin is budgeted at 40 percent of sales dollars. If the net operating income for November is budgeted at $28,000 the budget for selling and administrative expenses for November is: a. $66,000; b. $38,000; c. $32,000; d. $30,000; e. none of these.



The following data relate to questions 13 through 15.


Hamway Products, Inc. makes and sells a single product called a Wob. It takes two yards of material A to make one Wob. Budgeted production of Wobs for the next four months is as follows:


April . 12,000 units

May 13,500 units

June .. 12,400 units

July .. 11,200 units


The company wants to maintain monthly ending inventories of material A equal to 10 percent of the following month's production needs. On March 31, 1,500 yards of material A were on hand. The cost of material A is $.90 per yard. The company wants to prepare a Direct Materials Purchases Budget for the second quarter.


13. The total cost of material A to be purchased in April is: a. $22,680; b. $25,200; c. $26,750; d. $26,780; e. none of these.


14. The total needs of material A for the month of May is: a. 14,740 yards; b. 25,860 yards; c. 29,480 yards; d. 27,000 yards; e. none of these.


15. The desired ending inventory of material A for the month of June is: a. 2,480 yards; b. 1,120 yards; c. 1,870 yards; d. 2,240 yards; e. none of these.


The following data relate to questions 16 through 18.


The Gerald Company makes and sells a single product called a Clop. Each Clop requires the use of 1.1 hours of direct labor time in its manufacture. The planned cost of direct labor time is $8.20 per hour. The company wishes to prepare a Direct Labor Budget for the first quarter of the year.


16. If the company has budgeted to produce 20,000 Clops in January, then the budgeted direct labor cost for January is: a. $164,000; b. $180,400; c. $172,200; d. $195,600; e. none of these.


17. If the budgeted direct labor cost for February is $162,360, then the budgeted production of Clops for February is: a. 23,200 units; b. 21,000 units; c. 19,800 units; d. 18,000 units; e. none of these.


18. The budgeted direct labor cost per Clop is: a. $7.45; b. $8.20; c. $9.02; d. $9.76; e. none of these.


19. Which of the following statements is not correct? a. A direct labor budget is a detailed plan showing labor requirements over some specific time period; b. Responsibility accounting is a system of accounting in which costs are assigned to various managerial levels according to where control of the costs is deemed to rest, with managers being held responsible for the difference between actual and budgeted results; c. A sales budget is a detailed schedule showing expected sales over some specified time period; d. A production budget is a detailed plan showing the production costs, other than direct materials and direct labor, that will be incurred in attaining the output budgeted for a period; e. A master budget is an interdependent network consisting of the various budgets in an organization.


20. The Victor Company makes and sells a single product. The company reported the following data in its Ending Finished Goods Inventory Budget as of December 31:

PRODUCTION COSTS FOR ONE UNIT


Quantity Cost

Direct materials .. 7 pounds $.80 per pound

Direct labor 0.6 hours $8.00 per hour

Manufacturing overhead .. 0.6 hours ? per hour


Finished Goods on December 31 (4,000 units): $54,800


Manufacturing overhead is applied to products on a basis of direct labor hours. The manufacturing overhead cost per unit of finished product is: a. $3.30; b. $4.80; c. $5.50; d. $13.70; e. none of these.



The following data relate to questions 21 through 23.


The Culver Company is preparing its Manufacturing Overhead Budget for the third quarter of the year. The budgeted variable factory overhead rate is $3.00 per direct labor hour; the budgeted fixed factory overhead is $75,000 per month, with $16,000 of this amount being factory depreciation.


21. If the budgeted direct labor time for July is 6,000 hours, then the total budgeted factory overhead for July is: a. $77,000; b. $82,000; c. $85,000; d. $90,000; e. none of these.


If the budgeted direct labor time for August is 5,000 hours, then the total budgeted factory overhead per direct labor hour is: a. $15; b. $18; c. $20; d. $22; e. none of these.


23. If the budgeted cash disbursements for factory overhead for September are $80,000, then the budgeted direct labor hours for September must be: a. 7,400 hours; b. 6,200 hours; c. 6,500 hours; d. 7,000 hours; e. none of these.



The following data relate to questions 24 through 27.


The Panza Company makes and sells only one product called a Deb. The company is in the process of preparing its Selling and Administrative Expense Budget for the first half of the year. The following budget data are available:


Monthly Fixed Variable Cost

Item Cost Per Deb Sold

Sales Commissions . ‑‑ $0.75

Freight‑Out . ‑‑ $1.30

Advertising . $30,000 $0.20

Executive Salaries $25,000 ‑‑

Depreciation on Office

Equipment . $15,000 ‑‑

Other . $ 7,000 ‑‑


All of these expenses (except depreciation) are paid in cash in the month they are incurred.


24. If the company has budgeted to sell 18,000 Debs in January, then the total budgeted variable selling and administrative expenses for January will be: a. $13,500; b. $23,400; c. $37,900; d. $40,500; e. none of these.


25. If the company has budgeted to sell 16,000 Debs in February, then the total budgeted fixed selling and administrative expenses for February is: a. $36,000; b. $77,000; c. $62,000; d. $98,000; e. none of these.


26. If the company has budgeted to sell 20,000 Debs in March, then the total budgeted selling and administrative expenses per unit sold for March is: a. $2.25; b. $5.35; c. $5.80; d. $6.10; e. none of these.


27. If the budgeted cash disbursements for selling and administrative expenses for April total $116,000, then how many Debs does the company plan to sell in April? a. 17,333 units; b. 18,250 units; c. 24,000 units; d. 26,800 units; e. none of these.


The following data relate to questions 28 and 29.


The LaPann Company has obtained the following sales forecast data:


July August September October

Cash sales $ 80,000 $ 70,000 $ 50,000 $ 60,000

Credit sales $240,000 $220,000 $180,000 $200,000


The regular pattern of collection of credit sales is 20 percent in the month of sale, 70 percent in the month following the month of sale, and the remainder in the second month following the month of sale. There are no bad debts.


28. The budgeted accounts receivable balance on September 30 is: a. $126,000; b. $148,000; c. $161,000; d. $190,000; e. none of these.


29. The budgeted cash receipts for October are: a. $188,000; b. $248,000; c. $226,000; d. $278,000; e. none of these.



The following data relate to questions 30 through 32.


The Ellis Company has budgeted its activity for September according to the following information:


I. Sales are budgeted at $392,000 and all sales are for cash.

II. All purchases of merchandise inventory are for cash. Merchandise inventory was $150,000 on August 31 and the planned merchandise inventory on September 30 is $140,000; all merchandise is marked to sell at 40 percent above invoice cost.

III. The budgeted depreciation for the month is $12,000.

IV. The selling and administrative expenses are budgeted at $80,000 for the month and are paid for in cash.

V. The cash balance on September 1 is $15,000.


30. The budgeted net income for September is: a. $20,000; b. $143,200; c. $112,000; d. $64,800; e. none of these.


31. The budgeted cash receipts for September are: a. $235,200; b. $156,800; c. $392,000; d. $407,000; e. none of these.


32. The budgeted cash disbursements for September are: a. $140,000; b. $270,000; c. $350,000; d. $362,000; e. none of these.


33. Which of the following is not a benefit of budgeting? a. It uncovers potential bottlenecks before they occur; b. It coordinates the activities of the entire organization by integrating the plans and objectives of the various parts; c. It provides guidance on the proper methods of keeping the accounting records to comply with generally accepted accounting principles; d. It provides managers with a way to formalize their planning efforts; e. It provides definite goals and objectives that serve as benchmarks for evaluating subsequent performance.


34. Which of the following represents the proper sequence in which the budgets below are prepared? a. Sales, Balance Sheet, Income Statement; b. Balance Sheet, Sales, Income Statement; c. Sales, Income Statement, Balance Sheet; d. Income Statement, Sales, Balance Sheet; e. none of these.



The following data relate to questions 35 through 37.


The Gilbert Company makes and sells a product called a Ciff. Each Ciff sells for $15 and has a unit variable cost of $10. The company has budgeted the following data for March:


I. Sales of $347,250, all for cash.

II. A cash balance on March 1 of $30,000.

III. Cash disbursements (other than interest) during March of $363,200.

IV. A minimum cash balance on March 31 of $25,000.


If necessary, the company will borrow cash from a bank. The borrowing will be in multiples of $1,000 and will bear interest at 1 percent per month. All borrowing will take place at the beginning of the month. The March interest will be paid in cash during March.


35. The number of Ciffs (in units) budgeted to be sold during March is: a. 34,725; b. 23,150; c. 69,450; d. 42,650; e. none of these.


36. The cash excess or deficiency before considering any cash borrowing or any interest charges for March is budgeted to be: a. $14,050 excess; b. $10,950 deficiency; c. $15,950 deficiency; d. $12,150 excess; e. none of these.


37. The amount of cash needed to be borrowed on March 1 to cover all cash disbursements and obtain the desired March 31 cash balance is: a. $11,000; b. $12,000; c. $13,000; d. $14,000; e. none of these.


38. The Willsey Merchandise Company has budgeted $40,000 in sales for the month of December. The company's gross margin is 70 percent of sales. If the company has budgeted to purchase $18,000 in merchandise during December, then the budgeted change in inventory levels over the month of December is: a. $6,000 increase; b. $10,000 decrease; c. $22,000 decrease; d. $15,000 increase; e. impossible to determine from the information given.


39. Manning Department Store's cost of goods sold is 45 percent of sales. For May, the budgeted net operating income is $12,000, and the budgeted selling and administrative expenses are $32,000. The budgeted sales for May are: a. $97,778; b. $80,000; c. $74,850; d. $71,000; e. none of these.


40. For May 19x1, Young Company has budgeted its cash receipts at $125,000 and its cash disbursements at $138,000. The company's cash balance on May 1 is $17,000. If the desired May 31 cash balance is $20,000, then how much cash must the company borrow during the month (before considering any interest payments)? a. $4,000; b. $8,000; c. $12,000; d. $16,000; e. none of these.


The following data relate to questions 41 and 42.


Esplanade Company has the following historical pattern on its credit sales:


70 percent collected in month of sale

15 percent collected in the first month after sale

10 percent collected in the second month after sale

4 percent collected in the third month after sale

1 percent uncollectible


The sales on open account have been budgeted for the last six (6) months of 19x6 as shown below.


July . $ 60,000

August .. 70,000

September .. 80,000

October . 90,000

November 100,000

December 85,000


41. The estimated total cash collections during October 19x6 from accounts receivable would be: a. $63,000; b. $84,400; c. $89,100; d. $21,400; e. none of these.


42. The estimated total cash collections during the fourth calendar quarter from sales made on open account during the fourth calendar quarter would be: a. $172,500; b. $275,000; c. $230,000; d. $251,400; e. none of these.


The following data relate to questions 43 and 44.


Pardise Company budgets on an annual basis for its fiscal year. The following beginning and ending inventory levels (in units) are planned for the fiscal year of July 1, 1989 through June 30, 1990.


July 1, 1989 July 30, 1990

Raw material* 40,000 50,000

Work in process . 10,000 10,000

Finished goods .. 80,000 50,000


*Two (2) units of raw material are needed to produce each unit of finished product.


43. If Pardise Company plans to sell 480,000 units during the 1989‑90 fiscal year, the number of units it would have to manufacture during the year would be: a. 440,000 units; b. 480,000 units; c. 510,000 units; d. 450,000 units; e. none of these.


44. If 500,000 finished units were to be manufactured during the 1989‑90 fiscal year by Pardise Company, the units of raw material needed to be purchased would be: a. 1,000,000 units; b. 1,020,000 units; c. 1,010,000 units; d. 990,000 units; e. none of these.


The following data relate to questions 45 through 47.


Information pertaining to Noskey Corporation's sales revenue is presented in the following table.


November 19x5 December 19x5 January 19x6

(Actual) (Budget) (Budget)

Cash sales . $ 80,000 $100,000 $ 60,000

Credit sales .. 240,000 360,000 180,000

Total sales . $320,000 $460,000 $240,000

======== ======== =========

Management estimates that five percent of credit sales are uncollectible. Of the credit sales that are collectible, 60 percent are collected in the month of sale and the remainder in the the month following the sale. Purchases of inventory each month are 70 percent of the next month's projected total sales. All purchases of inventory are on account; 25 percent are paid in the month of purchase, and the remainder are paid in the month following the purchase.


45. Noskey's budgeted cash collections in December 19x5 from November 19x5 credit sales are: a. $144,000; b. $136,800; c. $96,000; d. $91,200; e. none of these.


46. Noskey's budgeted total cash receipts in January 19x6 are: a. $240,000; b. $294,000; c. $299,400; d. $239,400; e. none of these.


47. Noskey's budgeted total cash payments in December 19x5 for inventory purchases are: a. $405,000; b. $283,500; c. $240,000; d. $168,000; e. none of these.


The following data relate to questions 48 through 51.


Kelly Company is a retail sporting goods store that uses accrual accounting for its records. Kelly's sales are budgeted at $220,000 for December 1989 and $200,000 for January 1990. Collections are expected to be 60 percent in the month of sale and 38 percent in the month following the sale. Two percent of sales are expected to be uncollectible. Gross margin is 25 percent of sales. A total of 80 percent of the merchandise for resale is purchased in the month prior to the month of sale and 20 percent is purchased in the month of sale. Payment for merchandise is made in the month following the purchase. Other expected monthly expenses to be paid in cash are $22,600. Annual depreciation is $216,000. Kelly Company's Statement of Financial Position at the close of business on November 30, 1989 is reproduced below.


Kelly Company

Statement of Financial Position

November 30, 1989


Assets

Cash . $ 22,000

Accounts receivable (net of $4,000 allowance

for uncollectible accounts) 76,000

Inventory .. 132,000

Property, plant and equipment

(net of $680,000 accumulated

depreciation) .. 870,000

Total assets $1,100,000

==========


Liabilities and Stockholders' Equity


Accounts payable $ 162,000

Common stock . 800,000

Retained earnings .. 138,000

Total liabilities and

stockholders' equity $1,100,000

==========


48. The budgeted cash collections for December 1989 are: a. $208,000; b. $132,000; c. $203,600; d. $212,000; e. none of these.


49. The pro forma income (loss) before income taxes for December 1989 is: a. $32,400; b. $28,000; c. $14,400; d. $10,000; e. none of these.


50. The projected balance in accounts payable on December 31, 1989 is: a. $162,000; b. $204,000; c. $153,000; d. $160,000; e. none of these.


51. The projected balance in inventory on December 31, 1989 is: a. $160,000; b. $120,000; c. $153,000; d. $150,000; e. none of these.


The following data relate to questions 52 and 53.


Berol Company plans to sell 200,000 units of finished product in July of 19x1 and anticipates a growth rate in sales of five percent per month. The desired monthly ending inventory in units of finished product is 80 percent of the next month's estimated sales. There are 150,000 finished units in the inventory on June 30, 19x1. Each unit of finished product requires four pounds of direct material at a cost of $1.20 per pound. There are 800,000 pounds of direct material in the inventory on June 30, 19x1.


52. Berol's production requirement in units of finished product for the three‑ month period ending September 30, 19x1 is: a. 712,025 units; b. 630,000 units; c. 664,000 units; d. 665,720 units; e. none of these.


53. Without prejudice to your answer to the previous question, assume Berol plans to produce 600,000 units of finished product in the three‑month period ending September 30, 19x1, and have direct materials inventory on hand at the end of the three month period equal to 25 percent of the use in that period. The estimated cost of direct materials purchases for the three‑month period ending September 30, 19x1 is: a. $2,200,000; b. $2,400,000; c. $2,640,000; d. $2,880,000; e. none of these.


XI. FLEXIBLE BUDGETING



1. If the Master Budget Formula for the year is:


Net income = $21.80 contribution margin per unit times number of units sold, less $445,000 fixed costs per year, then a budgeted annual net income of $34,600 corresponds to what number of units sold? a. 20,413 units; b. 18,000 units; c. 26,000 units; d. 22,000 units; e. none of these.


2. At the end of the year, a company's Manufacturing Overhead account contained the following data:


Manufacturing Overhead

­­ ­­­­ ­­­­ ­­­­ ­­­­ ­­­­ ­­­­ ­­­­ ­­‑‑‑

Actual $82,140 | Applied $78,260

­­ ­­­­ ­­­­ ­­­­ ­­­­ ­­­­ ­­­­ ­­­­ ­­­­ ­­

$3,880 |


If the denominator activity for the year was 40,000 machine hours, and if 36,400 machine hours were allowed for the year's production, then the predetermined overhead rate per machine hour was: a. $2.15; b. $1.96; c. $2.26; d. $2.05; e. none of these.


The following data relate to questions 3 through 6.


The Blew Company is interested in preparing a Master Flexible Budget for April. The following budgeted data are available for the month:


Sales in units . 8,000

Sales in dollars .. $960,000

Fixed costs (total):

Manufacturing overhead 160,000

Selling and admin. . 50,000


Variable costs (total):

Direct materials .. 160,000

Manufacturing overhead .. 48,000

Selling & admin. .. 16,000


3. The budgeted direct materials cost per unit sold is: a. $20; b. $.375; c. $45; d. $6.60; e. none of these.


4. The budgeted variable cost (total) per unit sold is: a. $26.25; b. $73.00; c. $99.25; d. $71.00; e. none of these.


5. If 7,000 units, rather than 8,000 units, are sold during a month, then the budgeted amount of total fixed cost for the month would be: a. $183,750; b. $210,000; c. $794,000; d. $511,000; e. none of these.


6. If 7,500 units are sold during a month, then the expected net income for the month would be: a. $166,000; b. $325,500; c. $119,000; d. $142,500; e. none of these.


7. Which of the following is not a step in preparing a flexible budget? a. Determine the relevant range over which activity is expected to fluctuate during the period; b. Analyze the costs that will be incurred over the relevant range in terms of cost behavior patterns; c. Analyze the costs that will be incurred outside the relevant range in terms of cost behavior patterns; d. Separate costs by behavior and find the formulas for the variable and mixed costs; e. Use the formula for each cost to prepare a budget showing the costs that will be incurred at various points over the relevant range.


8. Which of the following is not a proper use of the master flexible budget? a. Determining the expected revenues at the budgeted activity level for a period; b. Determining the expected costs at the budgeted activity level for a period; c. Determining the expected net income at the budgeted activity level for a period; d. comparing the actual results for a period against the appropriate budget figures within the relevant range; e. All of the above are proper uses of the master flexible budget.


The following data relate to questions 9 and 10.


The Lehman Company makes and sells a single product. The company's Master Flexible Budget formula for October is:


Net income = $52 contribution margin per unit times number

of units sold, less $192,000 fixed costs per month


In October, the company actually produced and sold 5,000 units of product and had a net income of $77,200. The company budgeted 5,500 units to be sold in October.


Note: U means unfavorable and F means favorable


9. The selling price/cost variance for October is: a. $9,200 U; b. $9,200 F; c. $16,800 U; d. $16,800 F; e. none of these.


10. The activity variance for October is: a. $22,000 U.; b. $22,000 F; c. $26,000 U; d. $18,000 F; e. none of these.


11. Which of the following is not a problem in overhead cost control? a. Manufacturing overhead is usually made up of many separate costs; b. The separate manufacturing overhead costs are often small in dollar amount; c. The separate manufacturing overhead costs are often the responsibility of different managers; d. The separate fixed manufacturing overhead costs usually require several different activity bases in order to allocate these costs to production departments; e. Manufacturing overhead costs vary in behavior with some variable, some fixed, and some mixed in nature.


12. Consider the following factors:

I. The avoidance of dollars in the activity base itself.

II. The selection of an activity base should be simple and easily understood.

III. The existence of a casual relationship between the activity base and overhead costs.


Which of the above factors should be considered in selecting an activity base for an overhead flexible budget? a. Only I; b. Only III; c. Only I and III; d. Only II and III; e. I, II and III.


The following data relate to questions 13 through 16.


The Van Patten Company makes and sells a single product. The company employs the following Master Flexible Budget formulas, which cover a relevant range from 12,000 to 14,000 units sold each month:


Cost Budget Formula

Direct materials . $36 per unit

Direct labor .. $18 per unit

Manufacturing overhead . $5 per unit plus $170,000 per month

Selling & admin. expense .. $1 per unit plus $40,000 per month


The product sells for $90 per unit.

13. If 12,000 units are sold during June, total budgeted variable expenses will be: a. $60; b. $780,000; c. $720,000; d. $840,000; e. none of these.


14. If 12,000 units are sold during June, total budgeted fixed expenses will be: a. $210,000; b. $170,000; c. $250,000; d. $785,000; e. none of these.


15. If 14,000 units are sold during July, the budgeted contribution margin for the month will be: a. $840,000; b. $350,000; c. $390,000; d. $420,000; e. none of these.


16. Suppose that during July the company budgets to sell 14,000 units, but actually sells only 13,000 units. The activity variance for the month of July will be: a. $18,000 unfavorable; b. $18,000 favorable; c. $30,000 unfavorable; d. $30,000 favorable; e. impossible to determine from the information given.


17. During March, the Cobb Company used the following Overhead Flexible Budget formula:


$102,400 plus $1.40 per Direct Labor Hour


If the company budgeted $207,400 in total overhead cost for March, then the budgeted direct labor hours for the month must have been: a. 148,143 hours; b. 115,700 hours; c. 74,000 hours; d. 75,000 hours; e. none of these.


18. During April, the Huff Company used a denominator activity of 5,000 machine hours to compute overhead rates. The company's Overhead Flexible Budget is based on the following cost formula:


F plus $2.60 per Machine Hour


where F denotes the total budgeted fixed overhead costs. The company's flexible budget showed $35,500 in total overhead cost at the 5,000 ‑ hour denominator activity level. Given these data, the fixed portion of the predetermined overhead rate for the month is: a. $7.10; b. $4.50; c. $5.20; d. $6.00; e. none of these.



The following data relate to questions 19 and 20.


The Chuba Company makes and sells a single product and uses standard costing. During December, the company actually used 7,200 direct labor hours and made 1,900 units of finished product. The standard cost card for one unit of product includes the following:


Variable factory overhead: 4 hours @ $5.25 per direct labor hour

Fixed factory overhead: 4 hours @ $2.00 per direct labor hour


For December, the company incurred $16,550 in fixed overhead costs and recorded an $800 unfavorable volume variance.


19. The amount of fixed overhead cost contained in the company's flexible budget for December is: a. $15,200; b. $16,000; c. $16,550; d. $13,700; e. $14,400.


20. The denominator activity level in direct labor hours used by Chuba in setting the predetermined overhead rate for December is: a. 7,600 hours; b. 7,800 hours; c. 8,000 hours; d. 7,200 hours; e. impossible to determine from the information given.


21. The Rowe Company makes and sells a single product and uses a standard cost system. During January, the company budgeted to incur $225,000 in manufacturing overhead cost and to operate at a denominator activity level of 25,000 machine hours. At standard, each unit of finished product requires 3 machine hours. The following cost and activity was recorded during January:


Total actual manufacturing overhead cost incurred . $217,750

Units of product completed 8,000

Actual machine hours worked .. 23,000


The amount of overhead cost that the company applied to Work in Process for January was: a. $217,750; b. $225,000; c. $221,600; d. $216,000; e. none of these.


For February, Edwards Company's master flexible budget shows expected sales of 16,000 units, a contribution margin of $28 per unit sold, and total budgeted variable costs of $512,000. If sales actually total 14,500 units for the month, the company's activity variance will be: a. $42,000 unfavorable; b. $42,000 favorable; c. $36,000 unfavorable; d. $36,000 favorable; e. impossible to determine because total budgeted fixed costs for February are not given.



The following data relate to questions 23 through 25.


The Fisk Company makes and sells a single product and uses a standard cost system. The company's income statement for May includes the following data:


3,000 units sold

Actual Budgeted

Sales $258,000 $255,000

Cost of goods sold (standard variable

cost of $58 per unit sold, plus $50,000

fixed overhead cost) 224,000 224,000


Selling & admin. exp. ($1.75 variable cost

per unit sold, plus $15,000 fixed cost) 22,000 20,250


23. The total budgeted variable costs for May at 3,000 units sold is: a. $246,000; b. $179,250; c. $244,250; d. $195,800; e. none of these.


24. The total budgeted fixed costs for May is: a. $85,000; b. $70,000; c. $65,000; d. $50,000; e. none of these.


25. The selling price/cost variance for May is: a. $1,250 F; b. $1,250 U; c. $1,550 F; d. $1,550 U; e. none of these.


26. The Nelson Company makes and sells only one product and employs standard costing. Overhead cost is applied to products on a basis of machine hours. During April, the company had underapplied overhead of $4,516. The following additional information is available relating to April production:


Budgeted fixed overhead cost .. $21,000*

Applied fixed overhead cost $18,900

Budgeted variable overhead cost .. $15,750*


*As contained in the flexible budget for the month.


Based on these data, the total actual fixed overhead cost for April was: a. $30,134; b. $41,086; c. $39,166; d. $35,812; e. none of these.

27. Consider an Overhead Flexible Budget which has machine hours as an activity base. If the machine hours of activity decrease within the relevant range, then there will be: a. a decrease in total budgeted fixed costs; b. an increase in total budgeted variable costs; c. an increase in the budgeted fixed costs per machine hour; d. an increase in the budgeted variable cost per machine hour; e. no change in total budgeted variable costs.


28. The Grib Company makes and sells a single product. For 19x2, the company's Master Flexible Budget shows expected sales of 4,000 units, a variable cost of $60 per unit sold, total fixed costs of $170,000, and an expected net income of $110,000. Given these data, the expected selling price per unit of product is: a. $200; b. $180; c. $120; d. $130; e. none of these.


29. A flexible budget is appropriate for a:

Selling and

Direct‑labor budget administrative expense budget

a. No No

b. No Yes

c. Yes No

d. Yes Yes


30. When a flexible budget is used, a decrease in production levels within a relevant range would: a. Decrease variable cost per unit; b. Decrease total costs; c. Increase total fixed costs; d. Increase variable cost per unit; e. none of these.


31. When a flexible budget is used, an increase in production levels within a relevant range would: a. not change fixed costs per unit; b. change total fixed costs; c. not change variable costs per unit; d. not change total variable costs; e. none of these.


32. A flexible budget is: a. not appropriate when costs and expenses are affected by fluctuations in volume limits; b. appropriate for any relevant level of activity; c. appropriate for control of factory overhead but not for control of direct materials and direct labor; d. Appropriate for control of direct materials and direct labor but not for control of selling and administrative expenses; e. responses b and d are both correct.


XII. STANDARD COSTS


1. Suppose a company wishes to isolate cost variances as soon as possible. What would be the best time to isolate and give recognition to the materials price variance? a. When the purchase order is created; b. When the material is used in manufacturing; c. When the material is purchased; d. When the material leaves the stock room; e. none of these.


2. If the labor efficiency variance is unfavorable, then: a. actual hours exceeded standard hours; b. standard hours exceeded actual hours; c. standard rate exceeded actual rate; d. actual rate exceeded standard rate; e. none of these.

3. Which of the following would not be found on a standard cost card for a unit of finished product? a. Standard direct materials to be used; b. Standard direct labor rate; c. Standard sales commission; d. Standard direct materials price; e. Standard variable manufacturing overhead rate.



The following data relate to questions 4 through 8.


The Roth Company uses standard costing. The company makes and sells a single product called a Blid. The following data are for the month of October:


I. Direct materials purchased and used: 10,500 feet at a total cost of $60,900

II. Materials price variance: $2,100 favorable

III. Total materials variance: $6,300 favorable

IV. Standard cost of materials per Blid: $24

V. Standard time for direct labor per Blid: 1.5 hours

VI. Standard direct labor cost charged to Work in Process: $42,000

VII. Labor efficiency variance: $1,400 unfavorable

VIII. Total labor variance: $1,204 favorable


Note: U denotes unfavorable and F denotes favorable.


4. The actual cost of materials per foot for October is: a. $5.70; b. $5.80; c. $5.90; d. $6.00; e. none of these.


5. The total number of Blids produced in October is: a. 11,586 units; b. 12,000 units; c. 4,500 units; d. 2,800 units; e. none of these.


6. The material quantity variance for October is: a. $4,200 F; b. $4,200 U; c. $8,400 F; d. $8,400 U; e. none of these.


7. The actual direct labor hours worked during October are: a. 4,200 hours; b. 4,060 hours; c. 4,340 hours; d. 4,560 hours; e. none of these.


8. The labor rate variance for October is: a. $2,604 U; b. $2,604 F; c. $196 U; d. $196 F; e. none of these.


The following data relate to questions 9 and 10.


The Porter Company had the following standard cost data for July:


I. Direct materials purchased and used: 22,500 lbs. at an actual cost of $53,000.

II. Materials quantity variance: $1,875 unfavorable.

III. Standard quantity of materials allowed for July production: 21,750 lbs.

Note: U denotes unfavorable and F denotes favorable.


9. The materials price variance for July is: a. $2,725 F.; b. $2,725 U.; c. $3,250 F; d. $3,250 U; e. none of these.


10. The total materials variance for July is: a. $1,375 F; b. $1,875 U; c. $5,125 F; d. $5,125 U; e. none of these.


11. The Wright Company has a standard costing system. The following data are available for September:


I. Actual quantity of direct materials purchased: 25,000 lbs.

II. Standard price of direct materials: $2 per pound.

III. Material price variance: $2,500 unfavorable.


The actual price per pound of direct materials purchased in September is: a. $1.85; b. $2.00; c. $2.10; d. $2.15; e. none of these.


12. The Fletcher Company uses standard costing. The following data are available for October:


I. Actual quantity of direct materials used: 23,500 lbs.

II. Standard price of direct materials: $2 per pound.

III. Material quantity variance: $1,000 favorable.


The standard quantity of material allowed for October production is: a. 23,000 lbs; b. 24,000 lbs; c. 24,500 lbs; d. 25,000 lbs; e. impossible to determine from the data given.


13. Which of the following factors is not likely to be considered by management in determining whether a variance should be regarded as an exception? a. Consistency of occurrence over time; b. Ability to control the item related to the variance; c. Nature of the item related to the variance; d. Costing method used to calculate the variance; e. Materiality of the variance.


The following data relate to questions 14 and 15.


Borden Enterprises uses standard costing. For the month of April, the company reported the following data:


I. Standard direct labor rate: $10 per hour

II. Standard hours allowed for actual production: 8,000

III. Actual direct labor rate: $9.50 per hour

IV. Labor efficiency variance: $4,800 favorable


Note: F denotes favorable and U denotes unfavorable


14. The labor rate variance for April is: a. $3,760 U; b. $3,760 F; c. $2,850 F; d. $2,850 U; e. none of these.


15. The total labor variance for April is: a. $1,040 U; b. $1,040 F; c. $8,560 F; d. $8,560 U; e. none of these.


16. A favorable labor rate variance indicates that: a. actual hours exceed standard hours; b. standard hours exceed actual hours; c. actual rate exceeds standard rate; d. standard rate exceeds actual rate; e. none of these.


17. The standard cost card for one unit of a certain finished product shows the following:

Standard Quantity Standard Price

Item or Hours or Rate

Direct materials . 12 feet $ ? per foot

Direct labor 1.5 hours $12 per hour

Variable overhead . 1.5 hours $8 per hour


If the total standard variable cost for one unit of finished product is $78, then the standard price per foot for direct materials is: a. $2; b. $3; c. $4; d. $5; e. none of these.


The following data relate to questions 18 and 19.


The Haney Company has a standard costing system. The following data are available for January:


I. Actual variable manufacturing overhead: $25,500

II. Actual direct labor hours worked: 5,800

III. Variable overhead spending variance: $600 favorable

IV. Variable overhead efficiency variance: $2,475 unfavorable

V. Variable overhead is assigned to production on the basis of direct labor hours.


Note: F denotes favorable and U denotes unfavorable


18. The total variable overhead variance for January is: a. $3,075 F; b. $3,075 U; c. $1,875 F; d. $1,875 U; e. none of these.


19. The standard hours allowed for January production is: a. 5,975 hours; b. 5,800 hours; c. 5,425 hours; d. 5,250 hours; e. none of these.


20. The standards that allow for no machine breakdowns or other work interruptions and that require peak efficiency at all times are referred to as: a. normal standards; b. practical standards; c. ideal standards; d. budgeted standards; e. none of these.


The following data relate to questions 21 through 25.


Longview Hospital performs blood tests in its laboratory. The following standards have been set for each blood test performed:


Standard Price Standard Rate

Item or Quantity or Hours

Direct materials 2 plates $2.75 per plate

Direct labor . 0.2 hours $15 per hour

Variable overhead .. 0.2 hours $7 per hour


During May, the laboratory performed 1,500 blood tests. On May 1 there were no direct materials (plates) on hand; after a plate is used for a blood test it is discarded. Variable overhead is assigned to blood tests on the basis of direct labor hours. The following events occurred during May:


I. 3,600 plates were purchased at at cost of $9,540

II. 3,200 plates were used for blood tests

III. 340 actual direct labor hours were worked at a cost of $5,100


Note: F denotes favorable and U denotes unfavorable


21. The materials price variance for May is: a. $360 F; b. $360 U; c. $740 F; d. $740 U; e. none of these.


22. The materials quantity variance for May is: a. $1,650 F; b. $1,650 U; c. $550 U; d. $720 F; e. none of these.


23. The labor rate variance for May is: a. $225 F; b. $225 U; c. $450 F; d. $450 U; e. none of these.


24. The labor efficiency variance for May is: a. $600 F; b. $600 U; c. $515 U; d. $515 F; e. none of these.


25. The variable overhead efficiency variance for May is: a. $350 F; b. $350 U; c. $280 U; d. $280 F; e. impossible to determine from the information given.


26. The Reedy Company employs a standard costing system. The following data are available for November:


I. Actual direct labor hours worked: 5,800

II. Standard direct labor rate: $9 per hour

III. Labor rate variance: $1,160 favorable


The actual direct labor rate for November is: a. $8.80; b. $8.90; c. $9.00; d. $9.20; e. none of these.

27. The Fischer Company uses a standard costing system. For the month of December, the following data have been assembled:


I. Actual direct labor hours worked: 5,800

II. Standard direct labor rate: $9 per hour

III. Labor efficiency variance: $1,800 unfavorable


The standard hours allowed for December production is: a. 5,400 hours; b. 5,600 hours; c. 5,800 hours; d. 6,000 hours; e. impossible to determine from the data given.


28. Which of the following is not an advantage of using standard costs? a. They facilitate cash planning and inventory planning; b. They can assist in the implementation of 'responsibility accounting'; c. They provide an objective basis in determining costs for tax purposes; d. They may be more economical and simpler to use in income determination; e. They make possible the utilization of the concept of management by exception.



The following data relate to questions 29 through 33.


Cole laboratories makes and sells a lawn fertilizer called Fastgro. The company has developed standard costs for one bag of Fastgro as follows:


Standard

Item Standard Quantity Cost per Bag

Direct material . 20 pounds $8.00

Direct labor . 0.1 hours 1.10

Variable overhead .. 0.1 hours .40


The company had no beginning inventories of any kind on Jan. 1. Variable overhead is applied to production on the basis of direct labor hours. During January, the following activity was recorded by the company:


I. Direct materials purchased: 85,000 pounds at a cost of $32,300

II. Direct labor worked: 390 hours at a cost of $4,875

III. Variable manufacturing overhead incurred: $1,475

IV. Production of Fastgro: 4,000 bags

V. Inventory of direct materials on Jan. 31: 3,000 pounds


Note: F denotes favorable and U denotes unfavorable


29. The materials price variance for January is: a. $1,640 F; b. $1,640 U; c. $1,700 F; d. $1,300 U; e. none of these.


30. The materials quantity variance for January is: a. $800 U; b. $300 U; c. $300 F; d. $750 F; e. none of these.


31. The labor rate variance for January is: a. $475 F; b. $475 U; c. $585 F; d. $585 U; e. none of these.


32. The labor efficiency variance for January is: a. $475 F; b. $350 U; c. $130 U; d. $110 F; e. none of these.


33. The total variance for variable overhead for January is: a. $85 F; b. $40 F; c. $100 U; d. $125 F; e. none of these.



The following data relate to questions 34 through 36.


The Upton Company employs a standard costing system. Data for the month of February include the following:


I. Variable overhead cost incurred: $48,700.

II. Variable overhead is assigned to production on a basis of direct labor hours.

III. Total variable overhead variance: $300 favorable

IV. Standard hours allowed for actual production: 7,000

V. Actual direct labor hours worked: 6,840


Note: F denotes favorable and U denotes unfavorable


34. The standard variable overhead rate per direct labor hour is: a. $6.91; b. $6.95; c. $7.00; d. $7.12; e. impossible to determine from the information given.


35. The variable overhead spending variance is: a. $820 F; b. $820 U; c. $740 F; d. $740 U; e. impossible to determine from the information given.


36. The variable overhead efficiency variance is: a. $430 U; b. $740 F; c. $1,120 F; d. $950 U; e. none of these.


The following data relate to questions 37 through 41.


The Dexon Company makes and sells a single product called a Mip and employs a standard costing system. The following standards have been established for one Mip:

Standard Quantity Standard Cost

Item or Hours per Mip

Direct materials .. 6 board feet $9.00

Direct labor 0.8 hours 9.60


There were no inventories of any kind on August 1. During August, the following events occurred:


I. Purchased 15,000 board feet at the total cost of $24,000.

II. Used 12,000 board feet to produce 2,100 Mips.

III. Used 1,700 hours of direct labor time at a total cost of $20,060.


37. The inventory of direct materials on August 31 in board feet is: a. 0; b. 12,000; c. 15,000; d. 3,000; e. none of these.


38. To record the purchase of direct materials, the general ledger would include what kind of entry to the Materials Price Variance Account? a. $1,500 credit; b. $1,500 debit; c. $6,000 credit; d. $6,000 debit; e. none of these.


39. To record the use of direct materials in production, the general ledger would include what kind of entry to the Materials Quantity Variance account? a. $3,600 debit; b. $3,600 credit; c. $900 debit; d. $900 credit; e. none of these.


40. To record the incurrence of direct labor cost and its use in production, the general ledger would include what kind of entry to the Labor Rate Variance account? a. $240 credit; b. $240 debit; c. $340 debit; d. $480 credit; e. none of these.


41. To record the incurrence of direct labor costs and its use in production, the general ledger would include what kind of entry to the Labor Efficiency Variance account? a. $480 credit; b. $240 debit; c. $1,200 debit; d. $1,200 credit; e. none of these.


The following data relate to questions 42 through 45.


The Johnson Company makes a single product called a Pef. The company uses a standard cost system and has established the following standards for one Pef:


Standard Quantity Standard Cost

Item or Hours per Pef

Direct materials .. 4 gallons $10

Direct labor .. 0.5 hours $5

Variable overhead 0.5 hours $2


There was no direct materials inventory on June 1. Variable overhead is assigned to products on a basis of direct labor hours. The following events occurred in June:


I. Purchased 5,000 gallons of direct materials at a cost of $13,000.

II. Used 4,700 gallons of direct materials to produce 1,200 Pefs.

III. Used 670 hours of direct labor at a cost of $6,365.

IV. Incurred $2,510 in variable overhead cost.


Note: F denotes favorable and U denotes unfavorable


42. The material quantity variance for June is: a. $500 U; b. $250 F; c. $250 U; d. $185 F; e. none of these.


43. The labor efficiency variance for June is: a. $700 F; b. $700 U; c. $430 U; d. $980 F; e. none of these.


44. The amount debited to Work in Process to record the use of direct materials in production in June is: a. $13,000; b. $11,750; c. $12,000; d. $13,520; e. none of these.


45. The amount credited to Direct Materials Inventory to record the use of direct materials in June is: a. $12,500; b. $11,750; c. $12,000; d. $10,750; e. none of these.


46. Tub Co. uses a standard cost system. The following information pertains to direct labor for product B for the month of October:


Actual rate paid .. $8.40 per hour

Standard rate .. $8.00 per hour

Standard hours allowed for actual production 2,000 hours

Labor efficiency variance .. $1,600 unfavorable


What were the actual hours worked? a. 1,800; b. 1,810; c. 2,190; d. 2,200; e. none of these.


47. For the month of April, Thorp Co.'s records disclosed the following data relating to direct labor:


Actual cost $10,000

Rate variance . 1,000 favorable

Efficiency variance . 1,500 unfavorable

Standard cost . $ 9,500


For the month of April, actual direct labor hours amounted to 2,000. In April, Thorp's standard direct labor rate per hour was: a. $5.50; b. $5.00; c. $4.75; d. $4.50; e. none of these.


48. Information on Rex Co.'s direct material costs for May 19x5 is as follows:


Actual quantity of direct materials purchased and used . 30,000 lbs.

Actual cost of direct materials $84,000

Unfavorable direct materials quantity variance $3,000

Standard quantity of direct materials allowed for production . 29,000 lbs.


For the month of May, what was Rex's direct materials price variance? a. $2,800 favorable; b. $2,800 unfavorable; c. $6,000 unfavorable; d. $6,000 favorable; e. none of these.



The following data relate to questions 49 and 50.


The following information pertains to Nell Company's production of one unit of its manufactured product during the month of June:


Standard quantity of materials .. 5 lbs.

Standard cost per lb. .. $.20

Standard direct labor hours .. .4

Standard wage rate per hour .. $7.00

Materials purchased . 100,000 lbs.

Cost of materials purchased .. $.17 per lb.

Materials consumed for manufacture of 10,000 units 60,000 lbs.

Actual direct labor hours required for 10,000 units .. 3,900

Actual direct labor cost per hour .. $7.20


The materials price variance is recognized when materials are purchased.


49. Nell's materials price variance for June was: a. $3,000 favorable; b. $3,000 unfavorable; c. $2,000 favorable; d. $2,000 unfavorable; e. none of these.


50. Nell's labor efficiency variance for June was: a. $780 favorable; b. $780 unfavorable; c. $700 favorable; d. $700 unfavorable; e. none of these.


51. If a company follows a practice of isolating variances at the earliest point in time, what would be the appropriate time to isolate and recognize a direct material price variance? a. When material is issued; b. When material is purchased; c. When material is used in production; d. When purchase order is originated; e. none of these.


52. Which department is customarily held responsible or an unfavorable materials quantity variance? a. Quality control; b. Purchasing; c. Engineering; d. Production; e. none of these.


53. Information on Hanley's direct labor costs for the month of January 19x2 is as follows:


Actual direct labor rate .. $ 7.50

Standard direct labor hours allowed 11,000

Actual direct labor hours . 10,000

Direct labor rate variance‑‑favorable . $ 5,500


What was the standard direct labor rate in effect for the month of January 19x2? a. $6.95; b. $7.00; c. $8.00; d. $8.05; e. none of these.


54. Information on Cox Company's direct material costs for the month of January 19x3 was as follows:


Actual quantity purchased .. 18,000

Actual unit purchase price . $ 3.60

Materials price variance‑‑unfavorable (based on purchases) .. $ 3,600

Standard quantity allowed for actual production . 16,000

Actual quantity used . 15,000


For January 19x3 there was a favorable direct material quantity variance of: a. $3,360; b. $3,375; c. $3,400; d. $3,800; e. none of these.


55. Harper Company uses a standard cost system. Data relating to direct labor for the month of August 19x3 is as follows:


Direct labor efficiency variance‑‑favorable . $5,250

Standard direct labor rate $ 7.00

Actual direct labor rate .. $ 7.50

Standard hours allowed for actual production 9,000


What are the actual hours worked for the month of August 19x3? a. 9,750; b. 8,400; c. 8,300; d. 8,250; e. none of these.


The following data relate to questions 56 through 59.


Arrow Industries employs a standard cost system in which direct materials inventory is carried at standard cost. Arrow has established the following standards for the prime costs of one unit of product.


Standard Standard Standard

Quantity Price Cost

Direct materials .. 8 pounds $1.80 per pound $14.40

Direct labor .25 hour $8.00 per hour 2.00

$16.40


During May, Arrow purchased 160,000 pounds of direct material at a total cost of $304,000. The total factory wages for May were $42,000, 90 percent of which were for direct labor. Arrow manufactured 19,000 units of product during May using 142,500 pounds of direct material and 5,000 direct labor hours.


56. The direct material price variance for May is: a. $16,000 favorable; b. $16,000 unfavorable; c. $14,250 favorable; d. $14,250 unfavorable; e. none of these.


57. The direct material quantity variance for May is: a. $14,400 unfavorable; b. $1,100 favorable; c. $17,100 unfavorable; d. $17,100 favorable; e. none of these.


58. The direct labor rate variance for May is: a. $2,200 favorable; b. $1,900 unfavorable; c. $2,000 unfavorable; d. $2,090 favorable; e. none of these.


59. The direct labor efficiency variance for May is: a. $2,200 favorable; b. $2,000 favorable; c. $2,000 unfavorable; d. $1,800 unfavorable; e. none of these.


The following data relate to questions 60 and 61.


The following processing standards have been set for Duo Co.'s clerical workers:


Number of hours per 1,000 papers processed . 150

Normal number of papers processed per year . 1,500,000

Wage rate per 1,000 papers .. $600

Standard variable cost of processing

1,500,000 papers.. $900,000

Fixed costs per year .. $150,000


The following information pertains to the 1,200,000 papers that were processed during 1990:


Total cost . $915,000

Labor cost . $760,000

Labor hours 190,000


60. For 1990, Duo's expected total cost to process the 1,200,000 papers, assuming standard performance, should be: a. $910,000; b. $900,000; c. $870,000; d. $840,000; e. none of these.


61. For 1990, Duo's labor rate variance would be: a. $40,000 unfavorable; b. $32,000 favorable; c. $10,000 unfavorable; d. $0; e. none of these.


62. Information on Barber Company's direct labor costs for the month of January 19x3 is as follows:


Actual direct labor hours . 34,500

Standard direct labor hours .. 35,000

Total direct labor payroll $241,500

Direct labor efficiency variance ‑favorable . 3,200


What is Barber's direct labor rate variance? a. $17,250 unfavorable; b. $20,700 unfavorable; c. $21,000 unfavorable; d. $21,000 favorable; e. none of these.

63. During March 19x4 Younger Company's direct material costs for the manufacture of product T were as follows:


Actual unit purchase price . $6.50

Standard quantity allowed for actual production . 2,100

Quantity purchased and used for actual production .. 2,300

Standard unit price .. $6.25


Younger's material quantity variance for March 19x4 was: a. $1,250 unfavorable; b. $1,250 favorable; c. $1,300 unfavorable; d. $1,300 favorable; e. none of these.


64. If the actual hours worked exceed the standard hours allowed, what type of variance will occur? a. Favorable labor efficiency variance; b. Favorable labor rate variance; c. Unfavorable labor efficiency variance; d. Unfavorable labor rate variance; e. none of these.


65. An unfavorable materials price variance occurs because of: a. price increases on raw materials; b. price decreases on raw materials; c. less than anticipated levels of waste in the manufacturing process; d. more than anticipated levels of waste in the manufacturing process; e. both responses (a) and (b) are correct.


66. Consider the following:


I. Price increases for variable overhead items.

II. Setting the fixed portion of the predetermined overhead rate too low.

III. A waste of excessive usage of overhead materials.


Which of the above can affect the overhead spending variance? a. Only I; b. Only II; c. Only I and II; d. Only I and III; e. I, II and III.


67. Which of the following statements is not correct, assuming the denominator activity is in hours? a. If the denominator activity and the standard hours allowed for the output of a period are the same, then there is no volume variance; b. If there are no budgeted (or actual) fixed overhead costs for the period, then there is no volume variance for the period; c. If the denominator activity is greater than the standard hours allowed for the output of the period, then the volume variance is favorable; d. The volume variance is a measure of utilization of plant facilities; e. The volume variance equals the total budgeted fixed overhead cost for a period less the fixed overhead cost that is applied to work in process for the period.



The following data relate to questions 68 through 71.


The Ferris Company makes and sells a single product and uses an Overhead Flexible Budget to plan and control overhead costs. These costs are applied to products on a basis of direct labor hours. The standard cost card shows that 3 direct labor hours are required per unit of product. For August, the company budgeted to work 90,000 direct labor hours and incur the following costs:


Variable costs: $99,000 Fixed costs: $118,800


During August, the company completed 28,000 units of product, worked 86,000 direct labor hours, and incurred the following overhead costs:


Variable costs: $98,900 Fixed costs: $115,300


Note: U means unfavorable and F means favorable.


68. For August, the variable overhead spending variance is: a. $4,300 F; b. $4,300 U; c. $6,500 F; d. $6,500 U; e. none of these.


69. For August, the variable overhead efficiency variance is: a. $1,800 F; b. $0; c. $2,200 U; d. $2,200 F; e. none of these.


70. For August, the fixed overhead spending variance is: a. $3,500 F; b. $3,500 U; c. $3,200 F; d. $3,200 U; e. none of these.


71. For August, the fixed overhead volume variance is: a. $4,300 U; b. $7,920 U; c. $4,980 F; d. $4,980 U; e. none of these.


The following data relate to questions 72 and 73.


The Skoda Company makes and sells a single product called a Yig. Overhead costs are applied to products on a basis of direct labor hours. The following data applies to the company's activities for the month of November:


Actual fixed overhead cost incurred $161,450

Budgeted direct labor hours (denominator activity) 40,000

Number of Yigs completed .. 21,000

Fixed overhead spending variance $ 11,450 U

Standard direct labor hours allowed per Yig . 2


Note: U means unfavorable and F means favorable.


72. The fixed portion of the predetermined overhead rate for November is: a. $4.32; b. $4.12; c. $3.57; d. $3.75; e. $4.50.


73. The volume variance for November is: a. $7,500 F; b. $7,500 U; c. $6,800 F; d. $6,800 U; e. none of these.

74. During February, the Web Company used a denominator activity of 80,000 machine hours in computing its predetermined overhead rate. However, only 75,000 standard machine hours were allowed for the month's actual production. If the overhead volume variance for February was $6,400 unfavorable, then the total budgeted fixed overhead cost for the month was: a. $96,000; b. $102,400; c. $100,000; d. $98,600; e. impossible to determine from the information given.



The following data relate to questions 75 through 77.


The Tate Company makes and sells a single product. The company recorded the following cost and activity for September:


Cost:

Actual fixed overhead costs incurred . $61,400

Volume variance . $2,850 U

Activity:

Number of units completed 22,800

Standard direct labor hours allowed per unit of product 2.5

Fixed portion of the predetermined overhead rate . $0.95 per DLH

Budgeted direct labor hours (denominator activity) .. 60,000


Note: U means unfavorable and F means favorable.


75. The amount of fixed manufacturing overhead cost applied to work in process during September was: a. $61,400; b. $57,000; c. $54,150; d. $59,850; e. none of these.


76. The amount of fixed overhead cost contained in the company's overhead flexible budget for September was: a. $61,400; b. $57,000; c. $60,000; d. $58,550; e. none of these.


77. The fixed overhead spending variance for September was: a. $4,400 U; b. $4,400 F; c. $3,150 U; d. $3,150 F; e. none of these.



The following data relate to questions 78 through 82.


The Pohl Company makes and sells a single product and uses a standard cost system. Overhead costs are applied to products on a basis of machine hours. For June, the company's overhead flexible budget showed the following budgeted costs at a denominator activity level of 20,000 machine hours:


Variable costs (total): Fixed costs (total):

Maintenance $16,000 Supervision $20,500

Utilities $10,000 Depreciation $ 9,500


During June, 17,000 machine hours were used to complete 13,000 units of product, and the following actual overhead costs were incurred:


Variable costs (total): Fixed costs (total):

Maintenance $14,620 Supervision $19,320

Utilities $10,710 Depreciation $ 9,500


At standard, each unit of finished product requires 1.4 hours of machine time.


Note: U means unfavorable and F means favorable.


78. The variable overhead spending variance for maintenance cost for June was: a. $1,020 F; b. $1,020 U; c. $3,230 F; d. $3,230 U; e. none of these.


79. The variable overhead efficiency variance for utilities cost for June was: a. $960 F; b. $960 U; c. $600 F; d. $600 U; e. none of these.


80. The total predetermined overhead rate per machine hour for June was: a. $2.57; b. $1.30; c. $2.80; d. $3.15; e. none of these.


81. The fixed overhead spending variance (in total) for June was: a. $3,230 F; b. $3,230 U; c. $1,180 F; d. $1,180 U; e. none of these.


82. The total amount of overhead cost applied to Work in Process during June was: a. $54,150; b. $52,100; c. $53,660; d. $50,960; e. none of these.


83. RedRock Company uses flexible budgeting for cost control. RedRock produced 10,800 units of product during March, incurring an indirect materials cost of $13,000. Its master budget for the year reflected an indirect materials cost of $180,000 at a production volume of 144,000 units. In order to compute a variable overhead spending variance for March, the performance report for the month would show a budgeted cost for indirect materials of: a. $13,000; b. $13,500; c. $13,975; d. $11,700; e. none of these.


The following data relate to questions 84 and 85.


Based on a monthly normal volume of 50,000 units (100,000 direct labor hours), Raff Co.'s standard cost system contains the following overhead costs:


Variable . $6 per unit

Fixed . 8 per unit


The following information pertains to the month of March 19x9:

Units actually produced 38,000

Actual direct labor hours worked 80,000


Actual overhead incurred:

Variable . $250,000

Fixed . 384,000


84. For March 19x9 the unfavorable variable overhead spending variance was: a. $6,000; b. $10,000; c. $12,000; d. $22,000; e. none of these.


85. For March 19x9 the fixed overhead volume variance was: a. $96,000 unfavorable; b. $96,000 favorable; c. $80,000 unfavorable; d. $80,000 favorable; e. none of these.


86. King Company estimates that it will operate its manufacturing facilities at 800,000 direct labor hours for the year. The estimate for total budgeted overhead is $2,000,000, of which $1,600,000 is variable and $400,000 is fixed. The standard variable overhead rate is $2 per direct labor hour. The standard direct labor time is 3 hours per unit. The actual data for the year are presented below:


Actual finished units .. 250,000

Actual direct labor hours . 764,000

Actual variable overhead .. $1,610,000

Actual fixed overhead .. $ 392,000


The fixed overhead volume variance for the year is: a. $7,000 U; b. $25,000 U; c. $41,667 U; d. $18,000 F; e. none of these.



The following data relate to questions 87 through 89.


Dori Castings is a job order shop that uses a standard cost system to account for its production costs. The overhead costs are applied to production on a direct labor hour basis.


87. Dori's choice of a production volume as a denominator for calculating its predetermined overhead rate: a. has no effect on the fixed portion of this rate which is used for applying costs to production; b. has an effect on the variable portion of this rate which is used for applying costs to production; c. has no effect on the fixed overhead spending variance; d. has no effect on the fixed overhead volume variance; e. has no effect on the overall (net) fixed overhead variance.


88. A volume variance will exist for Dori in a month where: a. production volume differs from sales volume; b. actual direct labor hours differ from standard hours allowed; c. there is a spending variance in fixed overhead costs; d. the fixed overhead applied on the basis of standard hours allowed differs from actual fixed overhead; e. the fixed overhead applied on the basis of standard hours allowed differs from the budgeted fixed overhead.

89. The amount of fixed overhead that Dori would apply to finished production would be: a. the actual direct labor hours times the standard fixed overhead rate per direct labor hour; b. the standard hours allowed for the actual units of finished output times the standard fixed overhead rate per direct labor hour; c. the standard units of output for the actual direct labor hours worked times the standard fixed overhead rate per unit of output; d. the actual fixed overhead cost per direct labor hour times the standard hours allowed; e. the actual fixed overhead cost per direct labor hour times the actual direct labor hours worked on finished production.


XIII. JOINT COSTING



1. In a sell or process further decision, consider the following costs. Which of the above costs is (are) not relevant in a decision regarding whether the product should be processed further? a. Only I; b. Only III; c. Only I and II; d. Only I and III; e. I, II and III.


I. A variable production cost incurred prior to split‑off.

II. A variable production cost incurred after split‑off.

III. An avoidable fixed production cost incurred after split‑off.


The following data relate to questions 2 through 5.


Recyclers, Inc., reprocesses newspapers obtaining two main‑products, a by‑ product, and waste. By‑product revenues are treated as a reduction of the cost of obtaining the main‑products. During the period, 1,000 tons were processed at a cost of $12,000 for materials and processing, resulting in the following:


Sales Value Costs after Final Sales

Product Tons at Separation* Separation* Value*

Main‑1 200 $4,000 $2,000 $10,000

Main‑2 400 5,000 6,000 12,000

By‑prod 300 2,000 ‑0‑ 2,000

Waste 100 ‑0‑ ‑0‑ ‑0‑


*These are total, rather than per ton, figures.


2. If the firm allocates joint costs to the main‑products according to a physical basis, how much will be allocated to Main‑1? a. 2,000; b. 2,222; c. 2,400; d. 2,667; e. 3,333.


3. If the firm allocates joint costs to the main‑products according to sales value at separation, how much will be allocated to Main‑1? a. 3,636; b. 4,364; c. 4,444; d. 5,333; e. something else.


4. If the firm allocates joint costs to the main‑products according to final sales value, how much will be allocated to Main‑1? a. 5,000; b. 5,714; c. 6,000; d. 6,857; e. something else.

5. Main‑1 can be processed into Main‑2. What is the opportunity cost per ton to be assigned to Main‑1 for purposes of this decision? a. zero; b. 20; c. 30; d. 40; e. 50.


The following data relate to questions 6 and 7.


A processing plant has fixed joint costs of $20,000 per month and variable joint costs of $300 per input unit processed. Data regarding the three joint products are as follows:

Further Selling

Selling Price Processing Price after

Product at Separation Costs Further Proc.

A $100 $100 $300

B 200 200 500

C 200 100 500



6. What is the monthly breakeven point if all products are sold at the point of separation? a. 15.38; b. 20.00; c. 22.22; d. 33.33; e. 100.00.


7. What is the monthly breakeven point if all products are further processed? a. 15.38; b. 20.00; c. 22.22; d. 33.33; e. 100.00.


8. When a product emerges from a joint process, the decision to process it further depends partly upon: a. the size of the total joint costs; b. the number of different main‑ and by‑products; c. the methods by which joint products are costed, for both main and by‑products; d. more than one of the above; e. none of the above.


9. The net realizable value of joint cost allocation can be used except when there are no ordinary markets at the split‑off point for the joint products: a. True; b. False



The following data relate to questions 10 through 13.


A joint input costing $520 results in three distinct products at the separation point. Relevant data are as follows:


Sales Value Additional Cost Sales Value

Product at separation of processing after processing

W $100 $ 50 $200

X 200 100 500

Y 100 ‑‑‑ 100


All three products are treated as main‑products.


10. If joint costs are allocated according to the sales value at separation, at what cost will an X be inventoried at separation? a. 173; b. 260; c. 320; d. 325; e. none of the above.


11. If joint costs are allocated according to the sales value of the final or end products, at what cost will an X be inventoried at separation? a. 173; b. 260; c. 320; d. 325; e. none of the above.


12. If joint costs are allocated according to contribution in best alternative use (net realizable value), at what cost will an X be inventoried at separation? a. 173; b. 260; c. 320; d. 325; e. none of the above.


13. If joint costs are allocated according to contribution in best alternative use (net realizable value), at what cost will a finished Y be inventoried? a. 65; b. 80; c. 100; d. 130; e. none of the above.



The following data relate to questions 14 through 17.


A joint input costing $500 results in four distinct products at the point of split‑off. Relevant data follows:


Sales Value Additional Cost Sales Value

Product at split‑off of processing after processing

J $200 $100 $400

K 300 200 600

L 100 50 140

M 20 10 40



14. Which of the four products should not be further processed? a. J; b. K; c. L; d. M; e. They should all be further processed.


15. Management is considering an alternative to that shown for processing K. The separable cost of processing would be $250. If the firm is to be no worse‑ off, the new product must sell for: a. 850; b. 750; c. 650; d. 550; e. 450.


16. Assume that after split‑off, a J can be processed directly into a K and then processed further as K is shown. For purposes of making this decision, what is the opportunity cost to be assigned to a J? a. 100; b. 200; c. 300; d. 400; e. something else.


17. If an L is further processed as shown, it can be further processed again at a cost of $150 to obtain product XX. If the firm is not to be any worse‑off than if it made optimal processing decisions, XX must sell for at least: a. 200; b. 250; c. 260; d. 290; e. 300.


The following data relate to questions 18 through 21.

A joint input costing $500 results in four distinct products at the point of split‑off. Relevant data follows:


Sales Value Additional Cost Sales Value

Product at split‑off of processing after processing

J $200 $100 $400

K 300 200 600

L 100 ­­ ­­‑‑

M 20 10 40


Products J, K, and L are main‑products; product M is a by‑product.

18. If the other income approach is used for M, at what cost will it be inventoried at split‑off? a. zero; b. 10; c. 20; d. 30; e. 40.


19. If the other income approach is used for M, at what cost will it be inventoried after further processing? a. zero; b. 10; c. 20; d. 30; e. 40.


20. If the cost reduction approach is used for M, at what cost will it be inventoried at split‑off, if no profit or loss is to be recognized when the by‑ product is sold? a. zero; b. 10; c. 20; d. 30; e. 40.


21. If the cost reduction approach is used for M, at what cost will it be inventoried after further processing, if no profit or loss is to be recognized when the by‑product is sold? a. zero; b. 10; c. 20; d. 30; e. 40.



The following data relate to questions 22 through 28.


Multiprod, Inc., begins with a common input costing $2,000. This input includes materials, labor, and overhead. One‑half of the overhead is fixed.


At split‑off three products emerge: A, B, and C. A can be sold as is for $1,000. B can be sold as is for $500. C cannot be sold.


A can be processed further at a cost of $400 sold as a D for $1500. B is not processed further. C can be processed further at a cost of $600; this results in an E which can be sold for $1,000 and an F which can be sold for $100.


B is a by‑product; the cost‑reduction method is used. F is a by‑product; the other‑income method is used. D and E are main‑products; the net realizable value method applies.


22. The firm is considering using a biological process whereby an E will become a D. If it is not to be any worse off as a result, the cost of processing should not exceed: a. 1,500; b. 1,000; c. 600; d. 500; e. 400.

23. The firm has the technology to turn a B into an E. If the firm is not to be any worse off as a result, the cost of processing should not exceed: a. 400; b. 500; c. 600; d. 1000; e. none of the above.


24. The firm is seriously considering converting an F into a B. The opportunity cost to be assigned to an F for this purpose is: a. 100; b. 200; c. 300; d. 400; e. 500.


25. For product costing purposes, an F in inventory would be carried at: a. 0; b. $1‑25; c. $26‑50; d. $51‑75; e. $76‑100.


26. For product costing purposes, a B in inventory would be carried at: a. less than $500; b. exactly $500; c. more than $500.


27. Given the firm's product costing policy, the dollar amount of common cost to be allocated to the two main‑products is: a. $3,000; b. $2,600; c. $2,400; d. $2,000; e. $1,500.


28. Given the firm's costing policy, the proportion of common cost to be allocated to Product A is: a. 0‑20%; c. 21‑40%; c. 41‑60%; d. 61‑80%; e. 81‑ 100%.



The following data relate to questions 29 through 34.


Reprocessors, Inc., purchases a by‑product which results from refining castor oil. It pays $200 per ton for the material and it costs $400 to process a ton of the material. The following results are obtained from a ton of material:


Selling Price

Product Yield Pounds per pound

A 40% 800 $.50

B 30 600 $.20

C 10 200 $.20

D 20 400 $.30


For the product costing purposes, A and B are main‑products and C and D are by‑ products. Relative sales value is used to cost the main‑products, C and D.


29. If both C & D are treated as a reduction in the cost of main‑products, and C is treated as other income, how much of the joint cost will go to A & B? a. $420; b. $440; c. $476; d. $520; e. $600.


30. If D is treated as a reduction in the cost of main‑products, and C is treated as other income, how much of the joint cost will go to A & B? a. $420; b. $467; c. $476; d. $480; e. $520.


31. Refer to the two questions, above. What proportion of the joint cost will be allocated to A & B, respectively? a. 4/7 to A and 3/7 to B; b. 5/7 to A and 2/7 to B; c. 10/13 to A and 3/13 to B; d. 4/10 to A and 3/10 to B; e. The proportions to be allocated to A & B will depend on which of the two methods (above) is selected for the treatment of by‑products.


32. Product A can be further processed at a cost $.10 per pound and sold for $.65 per pound. However, as a result of processing, 10% of A will be lost as a result (down the drain). Should A be further processed? a. Yes; b. No.


33. Product B can be further processed at a cost of $.01 per pound. The result is that 1/4 of the weight will become A and 3/4 of the weight will still be B. If this is done, the firm will be: a. worse off by $45; b. worse off by $30; c. better off by $39; d. better off by $45; e. Cannot be determined without knowing how the joint or common cost has been allocated among main‑products and by‑ products.


34. C & D can be combined and sold to the natural foods industry at a price of $.30 per pound. If the firm is to be no worse off, the total cost of combining and processing C & D should not exceed: a. $20; b. $30; c. $160; d. $180; e. It depends on whether any joint cost has been allocated to C & D and, if so, how much.


The following data relate to questions 35 through 40.


Hallejuah, Inc., manufactures drug components from animal by‑products. One of the firm's processes results in several products. The physical flow chart with the cost of producing is as follows:


­­ ­­ M (30 pounds)

‑‑‑ J (50 pounds) Cost $200 ­­ ­­|

| ­­ ­­ N (20 pounds)

|

Cost $1,000 ­­ ­­|‑‑‑ K (10 pounds)

(1000 pounds) |

|--- L (40 pounds) Cost $100 ­­ ­­­­ ­­‑ O (40 pounds)

|

|

‑‑‑ Waste (900 pounds)


The single input, above, yields three products (J, K, and L) and waste at the point of split‑off. J is processed further to yield M and N. K is sold at split‑off. L is processed further to yield O. The waste is flushed down the drain.


Total Revenues: J K L M N O

$600 $100 $300 $900 $100 $500


N is a by‑product; the 'other income method' is used. K is a by‑product; the 'cost‑reduction method' is used. M and O are main‑products; the 'net realizable value method' is used.


35. The firm is considering a process whereby an O can be processed into an M (40 pounds of O will become 30 pounds of M). If it is not to be any worse‑off as a result, the cost of processing should not exceed: a. $100; b. $200; c. $300; d. $400; e. something else.


36. For purposes of the preceding question, the opportunity cost to be assigned to an O is: a. $100; b. $200; c. $300; d. $400; e. $500.


37. For product costing purposes, a K in inventory would be carried at: a. less than $50; b. $ 50‑100; c. $101‑150; d. $151‑200; e. more than $200.


38. For product costing purposes, a N would be carried in inventory at: a. less than $50; b. $ 50‑100; c. $101‑150; d. $151‑200; e. more than $200.


39. For product costing purposes, the dollar amount of the initial common or joint cost to be allocated to the two main‑products is: a. less than $800; b. $800‑850; c. $851‑900; d. $901‑950; e. $951‑1000.


40. For product costing purposes, the proportion of the initial common or joint cost to be allocated to product L is: a. .01 to .10; b. .11 to .20; c. .21 to .30; d. .31 to .40; e. .41 to .50.


XV. SERVICE DEPARTMENT COST ALLOCATION



1. A company has one service department, G, and two operating departments, Department I and Department II. Forty percent of G's services goes to I and the rest goes to II. Service department costs are allocated by the direct method. During a recent period, G incurred $38,000 in departmental costs (no distinction is made between fixed and variable service department costs). The amount of G's departmental cost allocated to II for this period is: a. $15,200; b. $19,000; c. $22,800; d. $38,000; e. none of these.


2. How should fixed costs of service departments be allocated to consuming departments? : a. Use budgeted costs based on actual service provided; b. Use budgeted costs based on peak‑period service needed; c. Use actual costs based on actual service provided; d. Use actual costs based on peak‑period service needed; e. none of these.


3. Consider the following statements concerning service department allocation methods:


I. Under the direct method, service department costs are allocated to other service departments.

II. To use the step method, at least two service departments are needed.

III. The direct method and the step method will generally yield different cost allocations to the consuming departments involved.


Which statement(s) is (are) correct? a. Only I; b. Only II; c. Only III; d. Only I and III; e. Only II and III.


The following data relate to questions 4 through 8.


The Delta Railroad has two operating divisions ‑ Freight and Passenger. The railroad has only one maintenance department which serves both divisions. Variable service department costs are budgeted at $12 per labor hour. The fixed service department costs are budgeted at $800,000 per year. Pertinent data for last year are as follows:

Labor Hours

Division Budgeted Peak‑period Actual

Passenger 19,200 24,750 20,500

Freight 28,800 30,250 29,500


During last year, the maintenance department actually incurred $650,000 in variable costs and $815,000 in fixed costs.


4. At the beginning of last year, the total amount of budgeted cost that should have been allocated from the maintenance department to the Passenger Division for flexible budget purposes is: a. $590,400; b. $550,400; c. $657,000; d. $600,000; e. none of these.


5. At the beginning of last year, the amount of variable service department cost that should have been allocated from the maintenance department to the freight division for flexible budget purposes is: a. $363,000; b. $360,000; c. $230,400; d. $345,600; e. none of these.


6. At the end of last year, the amount of fixed cost that should have been allocated from the maintenance department to the Freight Division for variance computation purposes was: a. $480,000; b. $440,000; c. $472,000; d. $50,000; e. none of these.


7. The amount of the maintenance department's actual variable costs that would be retained in the department as a spending variance is: a. $74,000; b. $0; c. $50,000; d. $60,000; e. none of these.


8. The amount of the maintenance department's actual fixed costs that would be retained in the department as a spending variance is: a. $0; b. $15,000; c. $12,000; d. $18,000; e. none of these.


9. Which of the following is not a key point to consider when allocating service department costs? a. The distinction between fixed and variable costs should be maintained, if possible; b. Fixed costs should be allocated in predetermined lump‑sum amounts; c. Variable costs should be allocated at the actual rate, based on the activity measure that controls the incurrence of the cost involved; d. If it is not feasible to distinguish between fixed and variable costs in a service department, then the department's costs should be allocated according to the base that appears to give the best measure of benefits received in the consuming departments; e. Reciprocal services between departments should be recognized where possible.


10. A firm has one service department, H, and two operating departments, I and II. Thirty percent of H's service goes to I and the rest goes to II. Service department costs are allocated by the direct method (no distinction is made between fixed and variable costs of H). During March, H allocated $7,200 in cost to I. The amount of cost allocated from H to II for March would have been: a. $7,200; b. $16,800; c. $12,400; d. $20,500; e. impossible to determine from the information given.


The following data relate to questions 11 and 12.


The Thomas Company has two service departments and two operating departments. The following data are available for a recent period:


Service Depts. Operating Depts.

P Q 1 2

Overhead costs $80,000 $60,000 $75,000 $92,000

Number of employees . 30 60 360 180

Square ft. of space . 18,000 25,000 60,000 90,000


Thomas allocates service department costs by the step method in the following order: P is allocated first, based on number of employees; Q is allocated second, based on space occupied. No distinction is made between fixed and variable costs.


11. After all allocations, the total amount of overhead cost contained in Department 1 will be: a. $152,333; b. $168,550; c. $150,200; d. $155,700; e. none of these.


12. After the allocations, the total amount of overhead cost contained in Department 2 will be: a. $156,800; b. $151,300; c. $154,667; d. $138,450; e. none of these.



The following data relate to questions 13 through 16.


The Toville City Hospital has two service departments ‑ Energy and Housekeeping. These two departments provide services to the three operating departments ‑ Pediatrics, Surgery, and Health Care. The fixed costs of the Energy Department are allocated on a basis of Kilowatt hours, and the fixed costs of the Housekeeping Department are allocated on a basis of square footage of space. Data for August are as follows:


House Health

Energy keeping Pediatrics Surgery Care

Budgeted fixed costs . $160,000 $200,000 ­­ ­­ ‑‑

Actual fixed costs . $162,000 $207,000 ­­ ­­ ‑‑

Square footage .. 5,000 5,000 20,000 10,000 70,000

Peak‑period KWH used .. 2,000 36,000 90,000 90,000 144,000

Actual KWH used ‑ August . 2,000 30,000 75,000 60,000 135,000


The hospital allocates service department costs by the step method, starting with the Energy Department.


13. The amount of fixed cost allocated from Housekeeping to Pediatrics for August will be: a. $40,000; b. $43,200; d. $40,500; c. $44,640; e. none of these.


14. The amount of fixed cost allocated from Energy to Surgery for August will be: a. $40,500; b. $38,900; c. $42,000; d. $40,000; e. none of these.


15. The amount of fixed cost allocated from Housekeeping to Surgery for August will be: a. $22,320; b. $28,740; c. $35,980; d. $25,180; e. none of these.


16. The total amount of fixed cost allocated to Health Care from all service departments for August will be: a. $221,040; b. $61,600; c. $215,200; d. $175,800; e. none of these.


17. Consider the following statements:


I. The direct method is simpler than the step method.

II. The direct method ignores interdepartmental services.

III. The direct method always leaves some service department

costs unallocated.


Which of these statements is (are) not true? a. Only I; b. Only II; c. Only III; d. Only II and III; e. Only I and II; e. none of these.


18. Consider the following statements:


I. The step method does recognize the interaction between service departments.

II. The step method requires that an order of allocation be established before service department costs can be allocated.

III. The step method will give the same allocations to the operating departments regardless of the order in which allocations are made.


Which of these statements is (are) true? a. Only I; b. Only II; c. Only III; d. Only I and II; e. I, II and III.



The following data relate to questions 19 through 23.


The Hudson Block Company has a trucking department that delivers crushed stone from the company's quarry to its two cement block production facilities ‑ the West Plant and the East Plant. The trucking department has adequate capacity to handle peak‑period requirements of 80,000 tons per year at the West Plant and 120,000 tons at the East Plant. At this activity level, budgeted costs for the trucking department are $400,000 per year, consisting of $340,000 in fixed costs and $0.30 per ton variable cost. During 19x2, 70,000 tons of crushed stone were budgeted to be delivered to the West Plant and 100,000 tons of crushed stone to the East Plant. The company allocates fixed and variable costs separately.


During 19x2, the trucking department actually delivered 75,000 tons of crushed stone to the West Plant and 90,000 tons to the East Plant. Its actual costs for the year were $65,000 variable, and $350,000 fixed.


19. The amount of actual fixed trucking department cost that should be allocated to the West Plant for 19x2 is: a. $160,000; b. $204,000; c. $140,000; d. $136,000; e. none of these.


20. The amount of actual fixed trucking department cost that should be allocated to the East Plant for 19x2 is: a. $240,000; b. $160,000; c. $184,000; d. $210,000; e. none of these.


21. The amount of actual variable trucking department cost that should be allocated to the West Plant for 19x2 is: a. $22,500; b. $24,000; c. $21,000; d. $32,000; e. none of these.


22. The amount of actual variable trucking department cost that should be allocated to the East Plant for 19x2 is: a. $30,000; b. $36,000; c. $40,000; d. $27,000; e. none of these.


23. The total amount of actual trucking department cost not allocated to the plants for 19x2 is: a. $10,000; b. $25,500; c. $0; d. $15,700; e. none of these.


The following data relate to questions 24 and 25.


The Bolton Company operates a Health Care service department for its employees. The variable costs of this department are allocated to the company's two operating departments Assembly and Finishing‑based on the number of employees in each department. Data for the past year are as follows:


Department

Health Care Assembly Finishing

Budgeted number of employees .. ‑‑ 300 200

Actual number of employees . ‑‑ 304 197

Budgeted total variable costs . $55,000 $400,000 $250,000

Actual total variable costs $56,112 $412,700 $256,100


The Health Care service department allocates its variable costs so as to be able to compare actual performance against planned performance.


24. The amount of actual Health Care variable cost allocated to Assembly for the past year would be: a. $33,000; b. $33,600; c. $33,440; d. $34,048; e. none of these.


25. The amount of actual Health Care variable cost allocated to Finishing for the past year would be: a. $21,670; b. $22,000; c. $22,064; d.$22,400; e. none of these.



The following data relate to questions 26 through 30.


The Conway Company has budgeted overhead costs in its factory departments for the coming year as follows:


Service departments:

Factory Administration $140,000

Custodial . 32,000


Operating departments:

Machining $160,000

Finishing 96,000


The company does not distinguish between fixed and variable service department costs and uses the direct method of allocation. Factory Administration costs are allocated on the basis of total labor hours in the operating departments, and Custodial costs are allocated on the basis of square footage of space occupied. Other pertinent budgeted data for the coming year are as follows:


Departments

Factory Adm. Custodial Machining Finishing

Total labor hours . ‑‑ 1,500 25,200 34,800

Sq. footage occupied . 4,000 2,000 40,000 40,000

Direct labor hours ­­ ­­ 10,000 30,000

Machine‑hours .. ­­ ­­ 35,000 15,000


In the Machining Department, machine‑hours are used as a basis for computing predetermined overhead rates; in the Finishing department, direct labor hours are used as a basis for computing predetermined overhead rates.


26. After all allocations, how much of the company's total overhead costs will be charged to the Machining Department for the coming year? a. $234,800; b. $217,400; c. $235,073; d. $205,100; e. $272,900.


27. After all allocations, how much of the company's total overhead costs will be charged to the company's Finishing Department for the coming year? a. $210,600; b. $215,100; c. $193,200; d. $192,927; e. $222,900.


28. The predetermined overhead rate in the Machining Department for the coming year is (to the nearest cent): a. $5.87; b. $6.21; c. $6.71; d. $5.44; e. none of these.


29. The predetermined overhead rate in the Finishing Department for the coming year is (to the nearest cent): a. $7.02; b. $6.44; c. $4.83; d. $5.79; e. none of these.


30. Suppose Conway decides not to allocate service department costs, but rather chooses to use a single plantwide predetermined overhead rate based on machine‑ hours. This rate, based on all factory overhead costs for the coming year, is (to the nearest cent): a. $5.35; b. $10.70; c. $7.28; d. $8.56; e. none of these.



The following data relate to questions 31 and 32.


The Sherman Company manufactures and sells rowboats and canoes. The company has two operating departments, Carpentry and Finishing. The company also has a Maintenance department which serves both operating departments. The costs in the Maintenance department are budgeted at $120,000 per month plus $0.25 per labor hour. The Maintenance department has a capacity to service peak period needs of 150,000 labor hours per month in Carpentry and 90,000 labor hours per month in Finishing. For May, Carpentry has estimated that it will operate 100,000 labor hours and Finishing has estimated that it will operate 80,000 labor hours.


31. The amount of Maintenance department cost that should be allocated to Carpentry at the beginning of May for flexible budget planning purposes is: a. $112,500; b. $100,000; c. $74,537; d. $92,400; e. none of these.


32. The amount of Maintenance department cost that should be allocated to Finishing at the beginning of May for flexible budget planning purposes is: a. $67,500; b. $65,400; c. $70,000; d. $65,000; e. none of these.


The following data relate to questions 33 through 35.


Lakeside Nursing Home has two operating departments, Custodial Care and Rehabilitation. It also has a Housekeeping department that serves the two operating departments. The costs of the Housekeeping department are all variable and are allocated to the operating department on a basis of labor hours. Data for September follow:


Department

Labor Hours Custodial Care Rehabilitation

Budgeted .. 3,000 1,000

Actual . 3,200 1,600


The budgeted costs of the Housekeeping department for September were $24,000 and the actual costs were $29,760.


33. The amount of Housekeeping department cost allocated to Custodial Care at the start of September for flexible budget planning purposes would be: a. $16,000; b. $6,000; c. $8,000; d. $18,000; e. none of these.


34. The amount of actual Housekeeping department cost allocated to Rehabilitation at the end of September would be: a. $19,840; b. $9,920; c. $9,600; d. $7,440; e. none of these.


35. The portion of the $29,760 actual Housekeeping service department costs for September that should not be allocated to the operating departments is: a. $960; b. $5,760; c. $0; d. $1,240; e. none of these.


36. Consider the following statements regarding the use of sales dollars as an allocation base:


I. Sales dollars often constitute a very poor allocation base.

II. Using sales dollars as an allocation base can cause one department to be allocated costs because of what happens in other departments.

III. Sales dollars represents a good allocation base because sales dollars often give a good indication of ability to pay.

Which of the above statements is (are) correct? a. Only I; b. Only III; c. Only I and II; d. Only I and III; e. I, II and III.


The following data relate to questions 37 and 38.


Allury Enterprises allocates its fixed administrative expenses on the basis of sales dollars. Last year the fixed administrative expenses were $90,000. Allury has two sales departments, Coats and Hats. Last year Coats had sales of $320,000 and Hats had sales of $80,000. This year Allury had fixed administrative expenses of $90,000, sales of $320,000 in the Coats department, and sales of $180,000 in the Hats department.


37. The fixed administrative expenses allocated to Hats last year was: a. $32,400; b. $18,000; c. $20,000; d. $24,000; e. none of these.


38. Consider the following statements:


I. The fixed administrative expense allocated to Coats decreased $12,600 this year from what it was last year.

II. The fixed administrative expense allocated to Hats increased $14,400 this year over what it was last year.

III. There is good reason for the manager of Hats to be upset over the allocation of fixed administrative expense this year as compared to last year's allocation.


Which of the above statements is (are) correct? a. Only I; b. Only II; c. Only I and III; d. Only II and III; e. I, II and III.



The following data relate to questions 39 through 43.


The Mohawk‑Hudson Company is an electric utility which has two service departments, Accounting and Maintenance. It has two operating departments, Generation and Transmission. The company does not distinguish between fixed and variable service department costs. Service department costs are allocated to operating departments on the basis of hours of service provided. Budgeted costs and other data for the coming year are as follows:


Departments

Accounting Maintenance Generation Transmission

Budgeted overhead .. $100,000 $200,000 $600,000 $400,000

Maintenance hours of service ‑‑ 600 7,200 4,800

Sq. footage occupied .. 5,000 2,000 50,000 23,000

Accounting hours of service .. 500 2,000 4,500 3,500


The company's accountant has prepared the following partial schedule of service department cost allocations by the step method for flexible budget planning purposes for the coming year:

Departments

Accounting Maintenance Generation Transmission

Dept. cost before allocation $ 100,000 $200,000 $600,000 $400,000

Allocation:

Accounting costs $(100,000) U V W

Maintenance costs X $(220,000) Y Z


39. In the above schedule, the value of U to the nearest dollar should be: a. $0; b. $20,000; c. $19,048; d. $18,000; e. none of these.


40. In the above schedule, the value of V to the nearest dollar should be: a. $42,857; b. $57,143; c. $38,000; d. $45,000; e. none of these.


41. In the above schedule, the value of W to the nearest dollar should be: a. $38,095; b. $34,000; c. $33,333; d. $39,000; e. none of these.


42. In the above schedule, the value of X to the nearest dollar should be: a. $0; b. $69,315; c. $75,000; d. $88,000; e. none of these.


43. In the above schedule, the value of Y to the nearest dollar should be: a. $132,000; b. $123,750; c. $150,685; d. $140,000; e. none of these.



The following data relate to questions 44 and 45.


The James Company has four departments with data as follows:


Service Depts. Operating Dept.

Cafeteria Maintenance Milling Finishing

Budgeted overhead costs $12,000 $10,000 $42,000 $38,000

Number of employees 12 10 84 66

Labor hours 1,500 1,250 5,250 4,750


44. The Maintenance costs are allocated on the basis of labor hours. The amount of cost allocated to Milling from Maintenance under the direct method would be: a. $5,600; b. $6,720; c. $5,250; d. $5,700; e. none of these.


45. Cafeteria costs are allocated on the basis of number of employees. If the step method is used with costs of the Cafeteria allocated first, the amount of cost allocated from the Cafeteria to Maintenance would be: a. $0; b. $625; c. $698; d. $750; e. none of these.


46. Boa Corp. distributes service department overhead costs directly to producing departments without allocation to the other service department. Information for the month of June 19x5 is as follows:


Service Departments

Maintenance Utilities

Overhead costs incurred $ 20,000 $10,000

======== ========

Service provided to

departments:

Maintenance 10 Percent

Utilities 20 Percent

Producing ‑ A 40 Percent 30 Percent

Producing ‑ B 40 Percent 60 Percent

Totals 100 Percent 100 Percent


The amount of maintenance department costs distributed to Producing Department A for June 19x5 was: a. $8,000; b. $8,800; c. 10,000; d. $12,000; e. none of these.


XV ANSWERS




I. INTRODUCTION


1. E 5. D 9. C 13. B

2. C 6. C 10. C 14. A

3. B 7. D 11. A 15. B

4. A 8. B 12. D 16. B



II. MANUFACTURING ENVIRONMENT


1. A 13. D 25. A 37. C 49. A

2. D 14. B 26. C 38. D 50. A

3. C 15. A 27. B 39. A 51. B

4. A 16. C 28. D 40. B 52. C

5. D 17. B 29. C 41. D 53. D

6. B 18. D 30. B 42. D 54. B

7. C 19. C 31. D 43. A 55. B

8. B 20. B 32. C 44. C 56. C

9. C 21. A 33. C 45. A

10. B 22. B 34. B 46. B

11. D 23. A 35. B 47. A

12. B 24. B 36. B 48. C



III. COST BEHAVIOR


1. C 11. A 21. C 31. C 41. C

2. B 12. B 22. B 32. D 42. C

3. D 13. B 23. C 33. B 43. B

4. C 14. D 24. D 34. C 44. A

5. A 15. D 25. D 35. A 45. D

6. D 16. A 26. B 36. D 46. B

7. C 17. C 27. B 37. A 47. B

8. D 18. D 28. D 38. A 48. C

9. A 19. B 29. A 39. C 49. B

10. C 20. C 30. C 40. B



IV. COST VOLUME PROFIT


1. A 15. C 29. B 43. C 57. C

2. D 16. B 30. D 44. E 58. C

3. D 17. A 31. A 45. C 59. B

4. B 18. A 32. C 46. A 60. E

5. C 19. C 33. B 47. D 61. A

6. B 20. D 34. C 48. A 62. C

7. A 21. B 35. D 49. C 63. C

8. C 22. B 36. D 50. D 64. B

9. B 23. B 37. A 51. A 65. D

10. D 24. C 38. C 52. D 66. D

11. B 25. C 39. B 53. C

12. C 26. A 40. D 54. C

13. B 27. D 41. B 55. D

14. A 28. C 42. A 56. B



V. VARIABLE COSTING


1. A 8. A 15. C 22. B 29. B

2. E 9. B 16. A 23. C 30. A

3. B 10. B 17. D 24. A 31. B

4. C 11. C 18. C 25. C 32. D

5. D 12. A 19. B 26. B 33. C

6. B 13. C 20. B 27. B

7. C 14. C 21. C 28. C



VI. SEGMENT ANALYSIS


1. C 6. B 11. D 16. B 21. A

2. A 7. C 12. A 17. D 22. D

3. B 8. C 13. A 18. B

4. E 9. D 14. C 19. D

5. B 10. B 15. A 20. C



VII. DECISION MAKING


1. C 13. E 25. B 37. A 49. C

2. D 14. B 26. D 38. B 50. C

3. C 15. A 27. C 39. A 51. A

4. A 16. C 28. A 40. D

5. D 17. D 29. C 41. C

6. B 18. C 30. B 42. E

7. B 19. D 31. A 43. E

8. C 20. C 32. C 44. C

9. A 21. A 33. A 45. C

10. B 22. B 34. B 46. B

11. D 23. B 35. D 47. D

12. D 24. C 36. C 48. B



VIII. JOB ORDER COSTING


1. A 13. B 25. C 37. A 49. D

2. C 14. A 26. D 38. C 50. B

3. D 15. A 27. A 39. B 51. A

4. B 16. B 28. C 40. B 52. B

5. B 17. D 29. B 41. D 53. D

6. D 18. C 30. B 42. B 54. C

7. C 19. D 31. D 43. D 55. A

8. A 20. C 32. D 44. C 56. C

9. A 21. D 33. A 45. A 57. C

10. C 22. A 34. C 46. A 58. C

11. D 23. B 35. B 47. B

12. A 24. B 36. C 48. C



IX. PROCESS COSTING


1. A 13. D 25. B 37. D 49. A

2. B 14. B 26. C 38. C 50. C

3. C 15. A 27. B 39. B 51. D

4. D 16. A 28. D 40. C 52. D

5. A 17. D 29. B 41. B 53. D

6. B 18. B 30. C 42. E 54. A

7. C 19. B 31. A 43. C 55. B

8. D 20. C 32. C 44. D

9. C 21. A 33. A 45. B

10. B 22. A 34. B 46. C

11. C 23. C 35. A 47. A

12. D 24. D 36. B 48. C



X. BUDGETING


1. D 12. B 23. D 34. C 45. D

2. C 13. A 24. D 35. B 46. C

3. D 14. C 25. B 36. B 47. B

4. B 15. D 26. D 37. B 48. A

5. D 16. B 27. C 38. A 49. D

6. C 17. D 28. E 39. B 50. C

7. C 18. C 29. B 40. D 51. B

8. A 19. D 30. A 41. B 52. D

9. D 20. C 31. C 42. C 53. C

10. E 21. E 32. C 43. D

11. D 22. B 33. C 44. C



XI. FLEXIBLE BUDGETING


1. D 9. B 17. D 25. A

2. A 10. C 18. B 26. C

3. C 11. D 19. B 27. C

4. B 12. E 20. C 28. D

5. B 13. C 21. D 29. D

6. D 14. A 22. A 30. B

7. C 15. D 23. B 31. C

8. E 16. C 24. C 32. B



XII. STANDARD COSTS


1. C 21. A 41. B 61. D 81. C

2. A 22. C 42. B 62. B 82. D

3. C 23. E 43. B 63. A 83. B

4. B 24. B 44. C 64. C 84. B

5. D 25. C 45. B 65. A 85. A

6. A 26. A 46. D 66. D 86. B

7. C 27. B 47. A 67. C 87. C

8. B 28. C 48. D 68. B 88. E

9. C 28. C 49. A 69. C 89. B

10. A 30. A 50. C 70. A

11. C 31. D 51. B 71. B

12. B 32. D 52. D 72. D

13. D 33. D 53. D 73. A

14. B 34. C 54. C 74. B

15. C 35. B 55. D 75. C

16. D 36. C 56. B 76. B

17. C 37. D 57. D 77. A

18. D 38. B 58. A 78. B

19. D 39. D 59. C 79. C

20. C 40. E 60. C 80. C



XIII. JOINT COSTING


1. A 11. D 21. E 31. C

2. E 12. C 22. D 32. B

3. C 13. B 23. B 33. C

4. E 14. C 24. A 34. A

5. D 15. C 25. A 35. D

6. E 16. C 26. B 36. E

7. D 17. E 27. E 37. B

8. E 18. A 28. D 38. A

9. B 19. B 29. B 39. C

10. B 20. C 30. D 40. D


XIV. SERVICE DEPARTMENT COST ALLOCATION


1. C 11. C 21. A 31. B 41. E

2. B 12. A 22. D 32. D 42. A

3. B 13. B 23. B 33. D 43. A

4. A 14. D 24. C 34. C 44. C

5. D 15. E 25. A 35. A 45. D

6. B 16. C 26. A 36. C 46. C

7. C 17. C 27. C 37. B

8. B 18. D 28. C 38. D

9. C 19. D 29. B 39. B

10. B 20. E 30. D 40. D







Politica de confidentialitate | Termeni si conditii de utilizare



DISTRIBUIE DOCUMENTUL

Comentarii


Vizualizari: 26901
Importanta: rank

Comenteaza documentul:

Te rugam sa te autentifici sau sa iti faci cont pentru a putea comenta

Creaza cont nou

Termeni si conditii de utilizare | Contact
© SCRIGROUP 2024 . All rights reserved